From 5bae51d51824c2d960bc15d332ec69bd5df515bf Mon Sep 17 00:00:00 2001 From: Ulli Kehrle Date: Sun, 17 Dec 2017 20:14:11 +0100 Subject: Quellcode neu strukturiert, Zusätzliche Version für kleine Bildschirme hinzugefügt MIME-Version: 1.0 Content-Type: text/plain; charset=UTF-8 Content-Transfer-Encoding: 8bit --- .gitignore | 3 +- Makefile | 11 +- ch01-lineare-struktur.tex | 296 +++++++ ch02-topologie.tex | 566 ++++++++++++++ ch03-topologisch-lineare-raeume.tex | 1480 +++++++++++++++++++++++++++++++++++ ch04-unitaere-raeume.tex | 447 +++++++++++ ch05-hahn-banach.tex | 370 +++++++++ funkana-ebook.tex | 68 ++ funkana.tex | 35 +- inhalt.tex | 972 ++++++++++++----------- latexmkrc | 4 +- motivation.tex | 68 ++ pdf/funkana.pdf | Bin 654737 -> 0 bytes ref.bib | 153 ++++ skript.cls | 67 +- 15 files changed, 4029 insertions(+), 511 deletions(-) create mode 100644 ch01-lineare-struktur.tex create mode 100644 ch02-topologie.tex create mode 100644 ch03-topologisch-lineare-raeume.tex create mode 100644 ch04-unitaere-raeume.tex create mode 100644 ch05-hahn-banach.tex create mode 100644 funkana-ebook.tex create mode 100644 motivation.tex delete mode 100644 pdf/funkana.pdf create mode 100644 ref.bib diff --git a/.gitignore b/.gitignore index 0583d8e..83c0976 100644 --- a/.gitignore +++ b/.gitignore @@ -1,3 +1,4 @@ auto/* build/* -*.pdf \ No newline at end of file +*.pdf +gitHeadInfo.gin \ No newline at end of file diff --git a/Makefile b/Makefile index a2933d0..f653328 100644 --- a/Makefile +++ b/Makefile @@ -1,4 +1,13 @@ -all: pdf/funkana.pdf +n: all publish + +all: pdf/funkana.pdf \ + pdf/funkana-ebook.pdf pdf/funkana.pdf: funkana.tex inhalt.tex skript.cls latexmk funkana.tex + +pdf/funkana-ebook.pdf: funkana-ebook.tex inhalt.tex skript.cls + latexmk funkana-ebook.tex + +publish: all + scp -r pdf/* hrnz.li:public_html/files/funkana/ diff --git a/ch01-lineare-struktur.tex b/ch01-lineare-struktur.tex new file mode 100644 index 0000000..9f151c6 --- /dev/null +++ b/ch01-lineare-struktur.tex @@ -0,0 +1,296 @@ +\chapter{Die lineare Struktur} +\section{Der lineare Raum} +Sei im Folgenden stets $\K = ℝ$ oder $\K = ℂ$. Zunächst die +\begin{definition}[Vektorraum] + Sei $\K$ ein Körper. Eine Abelsche Gruppe $(X,+)$ zusammen mit einer Abbildung + \[ + \cdot : \K × X → X + \] + heißt $\K$-Vektorraum, falls für alle $\alpha , β ∈ \K$ und $x, y ∈ X$ gilt: + \begin{enumerate}[label=(V\arabic*)] + \item $\alpha x+y) = \alpha x + βy$ + \item $(\alpha +β)x = \alpha x + βx$ + \item $(\alpha β)x = \alpha (βx)$ + \item $1 \cdot x = x$ + \end{enumerate} +\end{definition} + +\begin{bemerkung-nn} + Je nachdem, ob $\K = ℂ$ oder $\K = ℝ$ gilt, heißt $X$ ein \emph{komplexer} oder ein \emph{reeller} Vektorraum. +\end{bemerkung-nn} + +\begin{bemerkung-nn} + Eine nichtleere Teilmenge $Y ⊂ X$ ist bereits dann ein linearer Raum, falls aus $\alpha , β ∈ \K$, $x, y ∈ Y$ bereits $\alpha x + βy ∈ Y$ folgt, also $Y$ abgeschlossen unter den Vektorraumoperationen ist. + $Y$ heißt dann \emph{linearer Teilraum} oder auch \emph{linearer Unterraum}. +\end{bemerkung-nn} + +\begin{bemerkung-nn} + Zu jeder Teilmenge $M ⊂ X$ bildet die Menge aller Linearkombinationen von je endlich vieler Elemente einen linearen Teilraum von $X$. + Dieser heißt die \emph{lineare Hülle} von $M$ oder der \emph{Aufspann} von $M$ + \[ + \lspan M = \left\{ x ∈ X: ∃ l ∈ ℕ, \alpha _1,…,\alpha _l ∈ \K, m_1,…,m_l ∈ M \text{ mit } \sum_{i=1}^l \alpha _i m_i = x \right\}. + \] +\end{bemerkung-nn} + +\begin{bemerkung-nn} + $M = \{x_\lambda \}_{\lambda ∈ \Lambda } ⊂ X$ heißt \emph{Basis} oder \emph{Hamel-Basis} von $X$, falls $M$ \emph{linear unabhängig}, das heißt, + $0 ∈ X$ lässt sich nur auf triviale Art und Weise als Linearkombination endlich vieler der $x_\lambda $ schreiben, und $\lspan M = X$ ist. +\end{bemerkung-nn} + +\begin{bemerkung-nn} + Besitzt $X$ eine Basis von $n < \infty $ Elementen, dann heißt $n$ die \emph{Dimension} von $X$ und wir schreiben $\dim X = n$. + Andernfalls heißt $X$ \emph{unendlich-dimensional} ($\dim X = \infty $). +\end{bemerkung-nn} + +\begin{bemerkung-nn} + Seien $X_1, X_2 ⊂ X$ lineare Teilräume. Dann ist + \[ + X_1 + X_2 \coloneq \left\{ \alpha x_1 + βx_2: \alpha , β ∈ \K, x_1 ∈ X_1, x_2 ∈ X_2 \right\} + \] + ebenfalls ein linearer Teilraum. + Falls $X_1 ∩ X_2 = \{ 0\}$, schreiben wir $X_1 \oplus X_2$ und nennen die Summe \emph{direkt}. +\end{bemerkung-nn} + +\begin{bemerkung-nn} + Sei $Y$ ein linearer Teilraum von $X$. Definiere die Äquivalenzrelation $\sim$ auf $X$ durch + $x \sim y \Leftrightarrow x - y ∈ Y$. + Dann wird die Menge der Äquivalenzklassen mit vertreterweiser Addition und Multiplikation auch ein $\K$-Vektorraum. + Wir schreiben für diesen Vektorraum $X/Y$. +\end{bemerkung-nn} + +\begin{satz}\label{01-vr-besitzt-basis} + Jeder lineare Raum besitzt eine (Hamel-)Basis. +\end{satz} +\begin{proof} + Folgt unmittelbar aus \cref{01-basisergaenzungssatz}. +\end{proof} + +\begin{satz}[Basisergänzungssatz]\label{01-basisergaenzungssatz} + Sei $M ⊂ X$ eine linear unabhängige Teilmenge. + Dann gibt es eine Basis $B$ von $X$ mit $M ⊂ B$. +\end{satz} +\begin{proof} + Sei $P$ die durch Inklusion geordnete Menge aller linear unabhängigen Teilmengen von $X$, die $M$ umfassen. + Wegen $M ∈ P$ ist $P$ nichtleer. + Für jede totalgeordnete Teilmenge $T ⊂ P$ ist $\bigcup T ∈ P$, also $T$ durch $\bigcup T$ beschränkt. + Nach Zorn's Lemma besitzt $P$ ein maximales Element $B$. + Wäre $B$ keine Basis, gäbe es ein $x ∈ X \setminus \lspan B$. + Aber dann wäre $B ∪ \{x\}$ ebenfalls linear unabhängig im Widerspruch zur Maximalität von $B$. +\end{proof} + +\section{Beispiele} +\begin{beispiel} + Der $ℝ^n$ ist ein linearer Raum über dem Körper $ℝ$. Der $ℂ^n$ ist sowohl ein $ℂ$- als auch ein $ℝ$-Vektorraum. +\end{beispiel} + +\begin{beispiel} + Sei $[a,b] ⊂ ℝ$, $a < b$. Dann ist + \[ + C[a,b] = \{x: [a,b] → \K, x \text { ist stetig}\} + \] + ein $\K$-Vektorraum mit $\dim C[a,b] = \infty $. + Zum Beispiel sind die Monome $(t^k)_{k ∈ ℕ}$ ein unendliches linear unabhängiges System, jedoch keine Basis. + Tatsächlich ist jede Basis dieses Raumes überabzählbar. +\end{beispiel} + +\begin{beispiel}[Folgenräume] + Es ist + \[ + \ell^p = \{ x = (ξ_n)_{n ∈ ℕ} , ξ_n ∈ \K, \sum_{n=1}^∞ |ξ_n|^p < ∞ \} + \] + für $0 < p < ∞$ ein linearer Raum. + Die Menge der Einheitsvektoren $\{e_i\}_{i ∈ ℕ}$ mit $e_i = (0,…,0,1,0,…)$ ist eine unendliche linear unabhängige Teilmenge, aber ebenfalls keine Basis. + + Genauso ist + \[ + \ell^∞ = \left\{ x = (ξ_n)_{n ∈ ℕ} : ξ_n ∈ \K, \sup_{n=1} |ξ_n| < ∞ \right\} + \] + ein überabzählbar"=dimensionaler linearer Raum mit den Unterräumen + \[ + c = \left\{ (ξ_n)_{n ∈ ℕ} ∈ \ell^∞: \lim_{n → ∞} ξ_n \text{ existiert}\right\} + \] + und + \[ + c_0 = \left\{ (ξ_n)_{n ∈ ℕ} ∈ \ell^∞: \lim_{n → ∞} ξ_n = 0 \right\}. + \] +\end{beispiel} + +\begin{beispiel}[Lebesgue-integrierbare Funktionen]. + Sei $M ⊂ ℝ$ messbar und $0 < p < ∞$. + Dann ist + \[ + \L^p(M) = \left\{f : M → ℝ, f \text { messbar}, ∫_M |f|^p \dd μ < ∞ \right\} + \] + ein unendlich"=dimensionaler linearer Raum. + Offenkundig ist $\mathcal N \coloneq \{ f: M → ℝ,\; f = 0$ fast überall $\}$ ein Unterraum von $\L^p(M)$, also auch + \[ + L^p(M) = \L^p(M)/\mathcal N + \] + ein linearer Raum. +\end{beispiel} + +\section{Lineare Abbildungen} +\begin{definition} + Seien $X, Y$ lineare Räume über $\K$. $A: X → Y$ heißt \emph{linear}, falls für alle $x_1, x_2 ∈ X$ und $\alpha , β ∈ \K$ gilt: + \[ + A(\alpha x_1 + βx_2) = \alpha A(x_1) + βA(x_2). + \] + $A: X → \K$ heißt \emph{lineares Funktional}. + Für $A$ linear heißt $R(A) = \im A = \{A(x): x ∈ X\}$ der \emph{Bildraum} von $A$ und $N(A) = \ker A = \{ x ∈ X: A(x) = 0\}$ der \emph{Kern} von $A$. +\end{definition} + +\begin{bemerkung} + Sei $A: X → Y$ linear. + \begin{enumerate} + \item Sei $M ⊂ X $ ein linearer Unterraum. Dann ist $A(M) ⊂ Y$ wieder ein linearer Unterraum und es gilt $\dim A(M) \le \dim M$ mit Gleichheit bei Injektivität. + \item Es gilt + \[ + A \text{ injektiv} \Longleftrightarrow N(A) = \{ 0\}. + \] + Allgemeiner ist + \[ + X/(N(A)) \cong \im A. + \] + \item + Falls $\dim X = \dim Y = n < \infty $, dann ist $A$ genau dann injektiv, wenn $A$ surjektiv ist. + \item + $A: X → Y$ ist linear und bijektiv genau dann, wenn es eine lineare Umkehrabbildung $A^{-1}: Y → X$ gibt. + \item + Falls so ein $A: X → Y$ linear und bijektiv existiert, nennen wir $X$ und $Y$ \emph{linear isomorph.} + $A$ heißt dann ein \emph{linearer Isomorphismus}. + + Nur falls $\dim X = \dim Y < \infty $ sind $X$ und $Y$ auch „topologisch“ isomorph. + In diesem Fall erhält man die Prototypen $ℝ^n$ und $ℂ^n$ für endlich-dimensionale Vektorräume und andere gibt es nicht (die sie auch als Topologische Räume isomorph sind). + \end{enumerate} +\end{bemerkung} + +\begin{beispiel-nn} + $X = \{ x: [a,b] → ℝ, x, \dot x, \ddot x \text{ stetig},\; x(a) = \dot x(a) = 0\}$ ist ein linearer Raum. + Sei $Y = C[a,b]$ und $A: X → Y$ gegeben durch + \[ + (Ax)(t) \coloneq \ddot x(t) + c_1 (t) \dot x (t) + c_2 (t) x(t), \quad t ∈ [a,b], c_1,c_2 ∈ C[a,b]. + \] + Dann ist $A$ linear, weil differenzieren linear ist und $A$ ist injektiv: + Zunächst ist $x = 0$ eine Lösung der linearen Differentialgleichung zweiter Ordnung $Ax = 0$. + Die Theorie der Differentialgleichungen sagt uns, dass diese Differentialgleichung eine eindeutige Lösung des Anfangswertsproblems ist. + + $A$ ist aber auch surjektiv: Sei $y ∈ Y$ gegeben, dann suchen wir $x ∈ X$ mit $Ax = y$. + Also wollen wir eine inhomogene Differentialgleichung zweiter Ordnung lösen. + Auch diese ist nach der Theorie von gewöhnlichen Differentialgleichungen eindeutig lösbar. + + Also ist $A$ bijektiv, das heißt, es gibt eine lineare Abbildung $A^{-1}: Y → X$. + Diese Inverse ist in der Regel schlecht anzugeben. + Einen einfacheren Spezialfall dazu wird in der Übung behandelt. +\end{beispiel-nn} + +\begin{beispiel-nn} + Sei $X = Y = C[a,b]$, $A: X → X$ gegeben durch + \[ + (Ax)(t) \coloneq ∫_a^b k(s,t) x(s) ds, \quad t ∈ [a,b], + \] + wobei $k : [a,b] × [a,b] → ℝ$ stetig und gegeben ist. + Dann ist $A$ linear, da das Integral linear ist. + Auch ist, wenn $\lambda ∈ ℝ$ ein Parameter ist, die Abbildung + \[ + (A_\lambda x)(t) \coloneq \lambda x(t) - (Ax)t), \quad t ∈ [a,b] + \] + linear. + Die Probleme $Ax = y$ (bei gegebenem $y ∈ Y$ und gesuchtem $x ∈ X$) oder $A_\lambda x = 0$ (gesucht ist $\lambda ∈ ℝ$ und eine nichttriviale Lösung $x ∈ X \setminus \{ 0\}$) + heißen Integralgleichungen erster und zweiter Ordnung. +\end{beispiel-nn} + +\begin{beispiel-nn} + Sei $X = C[a,b]$, $A : X → ℝ$ mit + \[ + Ax = x(t_0), + \] + wobei $t_0 ∈ [a,b]$ fest gewählt sei. + Eine andere lineare Abbildung $A: X → ℝ$ ist gegeben durch + \[ + Ax = ∫_a^b x(t) dt + \] + Dann sind beide Abbildungen $A$ linear und nicht injektiv, aber surjektiv. +\end{beispiel-nn} + +\begin{beispiel-nn} + Sei $X = \ell^2$, $A: X → X$. Für $x = (ξ_n)_{n ∈ ℕ}$ sei + \[ + Ax = (0,ξ_1, ξ_2, \dots) ∈ \ell^2. + \] + $A$ heißt (Rechts-)Shiftoperator und ist linear und injektiv, jedoch nicht surjektiv. + Solche Abbildungen gibt es für $\dim X = \dim Y < \infty $ nicht. +\end{beispiel-nn} + +\section{Duale Räume} +$A: X → \K$ sei ein lineares Funktional, $X$ ein linearer Raum. Wir verwenden ein neues Symbol (statt $A$) +\[ + x': X → \K = \begin{cases} ℝ \\ ℂ \end{cases} \text{ linear}. +\] +Wir schreiben nun +\[ + x'(x) =: \langle x, x' \rangle = \langle x, x' \rangle_{X × X^f} ∈ \K. +\] +Wir setzen +\[ + X^f \coloneq \left\{ x': x' \text{ ist lineares Funktional auf } X \right\}. +\] +Hierbei sollte man nicht $x'$ nicht mit der Ableitung von $x$ verwechseln. +Auch ist $\langle -, - \rangle_{X × X^f}$ kein Skalarprodukt. + +Der Raum $X^f$ wird auf natürlicher Weise zum linearen Raum mit +\[ + (\alpha x_1' + βx_2')(x) \coloneq \alpha x_1'(x) + βx_2'(x), \quad x ∈ X, x_1', x_2' ∈ X^f, \alpha , β ∈ \K. +\] +So ist +\[ + \langle -,- \rangle_{X×X^f}: X × X^f → \K +\] +bilinear. +\begin{definition} + $X^f$ heißt der \emph{algebraische Dualraum} zu $X$. + $X^{ff} \coloneq (X^f)^f$ heißt der \emph{biduale Raum} zu $X$. +\end{definition} + +\begin{beispiel-nn} + $X^{ff}$ liefert die kanonische Abbildung + \[ + J: X → X^{ff}, \; x ↦ J(x) = x'' + \] + mit + \[ + \langle x', x'' \rangle \coloneq \langle x, x' \rangle \quad ∀x' ∈ X^f. + \] + Damit ist $x'': X^f → \K$ linear wohldefiniert. +\end{beispiel-nn} + +\begin{definition} + Der lineare Raum $X$ heißt \emph{algebraisch reflexiv}, falls $J$ bijektiv ist (und damit $X$ linear isomorph zu $X^{ff}$) ist. +\end{definition} + +\begin{bemerkung} + $X$ ist genau dann algebraisch reflexiv, wenn $\dim X < \infty $ ist. + + Im Fall $\dim X < \infty $ lässt sich leicht eine duale Basis angeben: + Sei dazu $M \coloneq \{x_1,…,x_n\}$ eine Basis von $X$. Dann wird durch + \[ + \langle x_i, x_k' \rangle \coloneq \delta _{i,k} + \] + und linearer Fortsetzung die Menge $ M \coloneq \{x_1',…,x_n'\} ⊂ X^f$ erklärt. + Dann ist $M'$ eine Basis von $X'$, die die \emph{duale Basis} von $M$ genannt wird. + Tatsächlich ist $X^f$ im Falle $\dim X = \infty $ wesentlich größer. + Man wählt deshalb eine (neue) Defintion des Dualraums: +\end{bemerkung} + +\begin{definition}[Dualraum] + Zu einem linearen Raum $X$ ist + \[ + X' \coloneq \left\{ x' : X → \K, x' \text{ linear und stetig} \right\} ⊂ X^f + \] + der Dualraum von $X$. +\end{definition} +Um Allerdings von Stetigkeit reden zu können, müssen wir zunächst \emph{Topologien} einführen. + +%%% Local Variables: +%%% mode: latex +%%% TeX-master: "funkana-ebook" +%%% End: diff --git a/ch02-topologie.tex b/ch02-topologie.tex new file mode 100644 index 0000000..b1e79f6 --- /dev/null +++ b/ch02-topologie.tex @@ -0,0 +1,566 @@ +\chapter{Topologie} +\section{Topologische Räume} +\begin{definition} + Sei $X$ eine Menge und $\mathcal T ⊂ \Pot X$ eine Menge von Teilmengen von $X$. + $\mathcal T$ heißt eine \emph{Topologie} auf $X$, falls $\mathcal T$ unter endlichen Durchschnitten und beliebigen Vereinigungen abgeschlossen ist. + Insbesondere muss $\mathcal T$ $\emptyset$ als leere Vereinigung und $X$ als leeren Schnitt enthalten. + $(X,\T)$ heißt dann \emph{topologischer Raum}. Die Elemente von $\T$ heißen \emph{offene Mengen} +\end{definition} +\begin{beispiele-nn} + \begin{enumerate}[label=(\alph*)] + \item + Für alle Mengen $X$ ist $\T = \{ ∅, X\}$ eine Topologie auf $X$, die sogenannte \emph{indiskrete Topologie}, \emph{gröbste Topologie} oder auch \emph{Klumpentopologie}. + \item + Für alle Mengen $X$ ist $\T = \Pot X$ eine Topologie, die sogenannte \emph{diskrete Topologie} oder \emph{feinste Topologie} auf $X$. + \item + In Analysis I wird eine Menge $U ⊂ ℝ$ für offen erklärt, wenn es zu jedem $x ∈ U$ ein $\epsilon > 0$ gibt, so dass für alle $ y ∈ ℝ$ mit $|x - y| < \epsilon $ auch $y ∈ U$ gilt. + Aus der Analysis ist bekannt, dass die so definierten offenen Mengen den Axiomen genügen. + Diese Topologie $\Tnat$ wird \emph{natürliche Topologie} genannt. + \item + Sei $X$ eine beliebige Menge. Die \emph{cofinite Topologie} auf + $X$ wird definiert als + \[ + \Tcof = \{ Y ⊂ X: Y = ∅\; \text{oder}\; \complement_X Y\, \text{ist endlich}\} + \] + \item + Der \emph{Sierpinski-Raum} ist die Menge $\{0,1\}$ versehen mit der Topologie $\{ ∅, \{0\}, \{0,1\}\}$. + \end{enumerate} +\end{beispiele-nn} + +\begin{definition} + Sei $M ⊂ X$. + \begin{enumerate} + \item + $M$ heißt \emph{abgeschlossen}, wenn $X \setminus M$ offen ist. + \item + $U ⊂ X$ heißt \emph{Umgebung von $A$}, wenn es eine offene Menge $V$ gibt mit $A ⊂ V ⊂ U$. Wir setzen + \[ + \U_A \coloneq \U_A (\T) \coloneq \{ U ⊂ X : U\; \text{Umgebung von $A$}\}. + \] + $\U_A$ heißt \emph{Umgebungssystem} oder \emph{Umgebungsfilter} von $A ⊂ X$. + Für $x ∈ X$ setzen wir $\U_x \coloneq \U_{\{x\}}$. $x$ heißt dann \emph{innerer Punkt} von $U$ für alle $U ∈ \U_x$. + \item + $x ∈ X$ heißt \emph{Häufungspunkt} von $M$, falls jede Umgebung von $x_0$ ein $y ∈ M$ enthält mit $y \ne x$.k + \item + Das \emph{Innere von M} ist + \[ + M^\circ \coloneq \bigcup \left\{ U ∈ \T: U ⊂ M \right\} + \] + die größte offene Menge, die in $M$ enthalten ist. + \item + Der \emph{Abschluss von} M ist + \[ + \cl M \coloneq \bigcap \left\{ U ⊂ M: U \text{ abgeschlossen} \right\} + \] + die kleinste abgeschlossene Menge, die $M$ enthält. + \item + $M$ heißt \emph{kompakt}, falls jede offene Überdeckung von $M$ eine endliche Teilüberdeckung besitzt. + \item + $M$ heißt \emph{dicht}, falls $\cl M = X$. + \item + $M$ heißt \emph{nirgends dicht}, falls $(\cl M)^\circ = \emptyset$. + \end{enumerate} +\end{definition} +\begin{bemerkung} + \begin{enumerate} + \item $M^\circ ⊂ M ⊂ \cl M$. + \item + $M^\circ$ ist die Menge der inneren Punkte von $M$. + \item + $M$ ist genau dann abgeschlossen, wenn $M = \cl M$. + \end{enumerate} +\end{bemerkung} + + +\begin{definition}[Hausdorff-Raum] + Sei $(X,\T)$ eine topologischer Raum. + Für alle $x,y \in X$ mit $x \neq y$ + existieren $U \in \U_x, V \in \U_x$ mit $U \cap V = \emptyset$. + Dann heißt $(X,\T)$ Hausdorff-Raum bzw. genügt dem $T_2$-Axiom. +\end{definition} + +\begin{beispiele-nn} + \begin{enumerate} + \item + Ein Pseudometrischer Raum $(X,d)$ ist Hausdorff"=Raum genau dann, wenn + $d$ eine Metrik ist. + \item + Der Sierpinski"=Raum $(\{0,1\}),\{\emptyset, \{0\}, \{0,1\}\})$ ist kein Hausdorff"=Raum. + \item + Sei $X = \prod_{i ∈ I} X_i$ ausgestattet mit dem Produkt $\T$ der Topologien + $(T_i)_{i ∈ I}$. $(X,\T)$ ist hausdorffsch genau dann, wenn alle $(X_i, + \T_i)$ hausdorffsch sind. + \item + Ist $(X,\T)$ ein Hausdorff Raum und $Y ⊂ X$, dann ist auch $(Y,\T|Y)$ hausdorffsch. + \end{enumerate} +\end{beispiele-nn} + +\begin{definition}[Konvergenz] + Eine Folge $\{x_{n}\}_{n \in \N} \subset X$ heißt konvergent gegen $x_{0} \in X$, + falls zu jeder Umgebung $U \in \U_{x_{0}}$ ein $n_{0} \in \N$ existiert, + sodass $x_{n} \in U$ für alle $n \geq n_{0}$. +\end{definition} +\begin{bemerkung-nn} + Man überlegt sich leicht, dass der Grenzwert $x_{0}$ in der Regel nicht eindeutig ist. + Bsp: In $\T=\{X,\emptyset\}$ konvergiert jede Folge gegen jeden Punkt. + Ist $(X,\T)$ jedoch ein Hausdorff-Raum, so ist jeder Grenzwert eindeutig. +\end{bemerkung-nn} +\begin{beweis} + Seien $x_{0} \neq x'_{0}$ Grenzwerte von $(x_{n})_{n \in \N} \subset X$. + Dann existieren disjunkte Umgebungen $U,U' ∈ \U_{x_0}$. + Weiterhin gibt es ein $n_{0} \in \N$, so dass $x_{n} \in U$ für alle $n \geq n_{0}$ + und $n_0' \in \N$, so dass $x_{n} \in U'$ für alle $n \geq n_0'$. + Also gilt $x_{\max\{n_0,n'_0\}} \in U \cap U'$ + Das ist ein Widerspruch zur Disjunktheit der Umgebungen. +\end{beweis} + +\begin{definition}[Häufungspunkt] + $x_{0} \in X$ heißt Häufungspunkt von $\{x_{n}\}_{n \in \N} \subset X$, + falls zu jeder Umgebung $U \in \U_{x_{0}}$ und für alle $k \in \N$ + ein $n \geq k \in \N$ existiert, so dass $x_{n} \in U$. +\end{definition} +\begin{beispiel-nn} + Wir betrachten $\R$ mit der natürlichen Topologie. + $(x_n)_{n ∈ ℕ}$ mit $x_n=(-1)^n$ hat zwei Häufungspunkte $\pm 1$. + Die Menge aller Folgenglieder $M=\{x_{n}:n \in \N\}=\{-1,1\}$ hat als Menge jedoch keine Häufungspunkte. +\end{beispiel-nn} +\begin{bemerkung-nn} + Für die indiskrete Topologie ist jeder Punkt in $X$ Häufungspunkt jeder Folge. +\end{bemerkung-nn} + +\begin{definition}[Stetigkeit] + Seien $(X, \T_X)$ und $(Y, \T_Y)$ topologische Räume, $f: X → Y$. + \begin{enumerate} + \item + Sei $x_0 ∈ X$. + $f$ heißt \emph{stetig in $x_0$}, falls für jede Umgebung $V$ von $f(x_0)$ das Urbild $f^{-1}(V)$ eine Umgebung von $x_0$ ist. + \item + $f$ heißt \emph{stetig}, falls für alle $V \in \T_{Y}$ gilt, dass $f^{-1}(V) \in \T_{X}$. + \end{enumerate} +\end{definition} +\begin{bemerkung-nn} + $f$ ist genau dann stetig, wenn $f$ in jedem Punkt stetig ist. +\end{bemerkung-nn} + +\begin{definition}[Homöomorphismus] + Ist $f : (X,\T_{X}) \rightarrow (Y,\T_{Y})$ bijektiv, stetig, + und $f^{-1} : (Y,\T_{Y}) \rightarrow (X,\T_{X})$ auch stetig, + dann heißt $f$ (und $f^{-1}$) \emph{Homöomorphismus}. + $X$ und $Y$ heißen \emph{homöomorph}, falls so ein Homöomorphismus +existiert. +\end{definition} + +\begin{definition}[Basis von Topologien und Umgebungen] + \begin{enumerate} + \item + Eine Familie $B \subset \T$ heißt Basis der Topologie in $(X,\T)$, falls $T= \{\bigcup M: M \subset B\}$. + \item + Eine Familie $B \subset \U_{x}$ von $x \in X$ heißt Umgebungsbasis des Punktes $x$, falls für alle $U \in \T, x \in U$ existiert ein $V \in B$ mit $x \in V \in U$. + \end{enumerate} +\end{definition} +\begin{beispiel-nn} + Für die natürliche Topologie auf $\R^n$ ist eine Basis der Topologie gegeben durch + ${B_{\eps}(x): x \in X, \eps > 0}$ + mit den offenen Kugeln $B_{\eps}(x)={y \in R^n : \norm{x-y}<\eps}$. + Sei $x \in \R^n$ fest. + Dann ist ${B_{1/n}(x):n \in \N}$ eine abzählbare Umgebungsbasis von x +\end{beispiel-nn} + +\begin{definition}[Relativtopologie oder Spurtopologie] + $M \subset \T$ eines topologischen Raumes $(X,\T)$ lässt sich in natürlicher Weise + zu einem topologischen Raum machen, nämlich mit $\T' \coloneq \{M \cap V : V \in \T\}$. +\end{definition} +\begin{bemerkung-nn} + $M = M \cap X \in \T'$ da $X \in \T$, d.h. $M$ ist offen in der Spurtopologie. + Achtung: $M$ muss nicht offen in $X$ sein. +\end{bemerkung-nn} + +\begin{definition} + Seien zwei Topologien $\T_{1},\T_{2}$ auf X gegeben. + Wir sagen $\T_{1}$ ist feiner als $\T_{2}$, falls $\T_{1} \supset \T_{2}$. + Wir sagen $\T_{1}$ ist gröber als $\T_{2}$, falls $\T_{1} \subset \T_{2}$. + Wir sagen die Topologien sind gleich, falls $\T_{1}=\T_{2}$. +\end{definition} +\begin{bemerkung-nn} + Sei $\T_{1}$ feiner als $\T_{2}$. + Die feinere Topologie $\T_{1}$ enthält mehr offene Mengen, + und damit zu jedem Grenzwert $x_{0}$ weniger konvergte Folgen. + + Man zeigt leicht: + $\T_{1}$ ist feiner als $\T_{2}$ $\Longleftrightarrow$ + Für alle $x \in X$ gilt: Seien $B_{1} \subset T_{1},B_{2} \subset T_{2}$ Umgebungsbasen von $x$, + dann gilt für alle $U \in B_{1}$, dass ein $V \in B_{2}$ existiert mit $V \subset U$. +\end{bemerkung-nn} + +\begin{beispiel-nn} + Folgende Topolgien auf $\R^n$ sind gleich. + $\T_{1}$ sei die Topologie, die durch die Kugeln + $B_{\eps}(x)=\{y \in R^n : \norm{x-y}<\eps\}$ erzeugt wird. + $\T_{2}$ sei die Topologie, die durch die Quader + $B_{\eps}(x)=\{y \in R^n : \max_{1 \leq i \leq n} |y_{i}-x_{i}|<\eps\}$ erzeugt wird. +\end{beispiel-nn} + +\begin{definition}[Produkttopologie] + Seien $(X,\T_{X}),(Y,\T_{Y})$ topologische Räume. + Dann ist die Familie von Mengen + \[ + \{U_{X} \times U_{Y} : U_{X} \in \T_{X}, U_{Y} \in \T_{Y} \} \subset \Pot{X \times Y} + \] + eine Basis der Topologie $\T_{X \times Y}$ im kartesischen Produkt $X \times Y$. + Bemerkung: Es genügt auch wenn $U_{X},U_{Y}$ über Basen von $\T_{X},\T_{Y}$ genommen werden. +\end{definition} + +\section{Metrische Räume} +\begin{definition}[Pseudometrik, Metrik] +\label{defi:metrik} + Sei $X$ eine Menge. $d: X × X → \R$ heißt \emph{Pseudometrik}, wenn $d$ den + folgenden Axiomen genügt: + \begin{enumerate}[series=metrik,label=(\textbf{M.\arabic*}),ref=M.\arabic*] + \item \label{defi:metrik:m1} + Für alle $x, y ∈ X$ gilt $d(x,y) \ge 0$ und $d(x,x) = 0$. + \item \label{defi:metrik:m2:symmetrie} + \emph{Symmetrie:} Für alle $ x, y ∈ X$ gilt $d(x,y) = d(y,x)$. + \item \label{defi:metrik:m3:dreiecksungleichung} + \emph{Dreiecksungleichung:} Für alle $x, y, z ∈ X$ gilt $d(x,y) + \le d(x,y) + d(z,y)$. + \end{enumerate} + $d$ heißt \emph{Metrik}, falls es zusätzlich + \begin{enumerate}[resume=metrik,label=(\textbf{M.\arabic*}),ref=M.\arabic*] + \item \label{defi:metrik:m4:posdef} + $d(x,y) = 0 \implies x = y$ + \end{enumerate} + erfüllt. $(X,d)$ heißt dann (pseudo-)metrischer Raum. Zu $x ∈ X$ und $r > 0$ + definieren wir die \emph{offene Kugel um $x$ mit Radius $r$} als + \[ + B_r(x) \coloneq B^d_r(x) \coloneq \{ y ∈ X: d(x,y) < r\}. + \] + Die Menge + \[ + \cl{B_r}(x) \coloneq \{y ∈ X: d(x,y) \le r\} + \] + heißt \emph{abgeschlossene Kugel}. +\end{definition} +\begin{satz} + Sei $(X,d)$ pseudometrischer Raum. Dann wird durch + \[ + U ∈ \T_d :\Longleftrightarrow ∀ x ∈ U ∃ ε > 0: B_ε(x) ⊂ U + \] + eine Topologie $\T_d$ definiert, die \emph{von $d$ induzierte Topologie} auf + $X$. Die Kugeln $B_r(x)$ für $x ∈ X$ und $r > 0$ sind offen bezüglich dieser + Topologie. Ein topologischer Raum $(X,\T)$ heißt + \emph{(pseudometrisierbar}, wenn es eine (Pseudo-)Metrik $d$ mit $\T = + \T_d$ gibt. +\end{satz} +\begin{proof} + Sei zunächst $\mathfrak M ⊂ \T_d$ und $x ∈ \bigcup \mathfrak M$. Dann gibt es + $x ∈ M ∈ \mathfrak M$, da $M$ offen ist, enthält $M$ einen Ball $B_ε(x)$. Somit auch + $B_ε(x) ⊂ M ⊂ \bigcup \mathfrak M$. Da dies für alle $x ∈ \bigcup \mathfrak M$ gilt, ist $\bigcup \mathfrak M$ offen. + + Sei nun $\mathfrak M ⊂ \T_d$ endlich und $x ∈ \bigcap \mathfrak M$. Dann + gibt es zu jedem $M_i ∈ \mathfrak M$ ein $ε_i > 0$ mit $B_{ε_i}(x) ⊂ M_i$. Somit ist + für $ε \coloneq \min\limits_{i} ε_i $ auch $B_ε(x) ⊂ \bigcup \mathfrak M$. + Also ist $\T_d$ eine Topologie. + + Jetzt zu $B_r(x) ∈ \T_d$. Sei dazu $y ∈ B_r(x)$, also $δ \coloneq r - d(x,y) > 0$ + Sei $z ∈ B_δ(y)$, also $d(z,y) < δ$. Mit der Dreiecksungleichung ist dann + $d(x,z) < d(x,y) + d(y,z) = d(x,y) + r - d(x,y) = r$. Also $y ∈ B_δ(y) ⊂ B_r(x)$. +\end{proof} +\begin{bemerkung-nn} + Die abgeschlossene Kugel $\cl B_r (x)$ ist im Allgemeinen nicht der Abschluss der offenen Kugel $B_r(x)$, aber es gilt immer + \[ + \cl{B_r(x)} ⊂ \cl{B_r}(x). + \] +\end{bemerkung-nn} +\begin{satz} + Sei $(X,d)$ ein metrischer Raum. + Dann genügt $(X,\T_d)$ dem $T_2$-Axiom. +\end{satz} +\begin{proof} + Seien $x \ne y ∈ X$. Dann ist $δ \coloneq d(x,y) > 0$. + Dann sind $B_{δ/2}(x)$ und $B_{δ/2}(y)$ disjunkte Umgebungen von $x$ bzw $y$: + Sei $z ∈ B_{δ/2}(x)$. Dann ist + \[ + d(z,y) \ge d(y,x) - d(x,z) > δ - \frac δ 2 = \frac δ 2. + \] +\end{proof} +\begin{lemma-nn}[Eigenschaften metrischer Räume] + Sei $(X,d)$ ein metrischer Raum. + \begin{enumerate} + \item Jeder Punkt $x ∈ X$ besitzt eine abzählbare Umgebungsbasis + \[ + \{ B_{1/n} (x), n ∈ ℕ\}. + \] + + \item + Es gilt + \[ + \lim_{n \to \infty } x_n = x \; \Longleftrightarrow \; \lim_{n→\infty } d(x,x_n) = 0. + \] + \item + Es ist $x_0 ∈ M$ genau dann ein innerer Punkt von $M ⊂ X$, wenn ein $\epsilon > 0$ existiert mit $B_\epsilon (x_0) ⊂ M$. + \item + $M$ ist nirgends dicht in $X$ genau dann, wenn es zu jeder Kugel +$B_\epsilon (x_0)$ mit $x_0 ∈ X, \epsilon > 0$ eine Kugel $B_\delta (x_1) ⊂ +B_\epsilon (x_0)$ mit $B_\delta(x_1) ∩ M = \emptyset$ gibt. + \item + Seien $(X,d_X)$ und $(Y,d_Y)$ metrische Räume. + Dann ist auch $(X×Y,d_{X×Y})$ ein metrischer Raum vermöge der Metrik + \[ + d_{X×Y}((x_1,y_1),(x_2,y_2)) \coloneq \max\{d_x(x_1,x_2),d_y(y_1,y_2)\} + \] + oder auch mit + \[ + d_{X×Y}((x_1,y_1),(x_2,y_2)) \coloneq \sqrt{d_x^2(x_1,x_2)+d_y^2(y_1,y_2)}. + \] + Tatsächlich induzieren diese beiden Metriken die gleiche Topologie (nämlich die Produkttopologie) + \item + Homöomorphismen $f: X → Y$ (für metrische Räume $X, Y$), die die Metrik respektieren, das heißt + \[ + d_X(x_1,x_2) = d_Y(f(x_1),f(x_2)) \quad ∀x_1, x_2 ∈ X + \] + heißen \emph{Isometrien}. + \item + Ein metrischer Raum muss im allgemeinen keine lineare Struktur haben. + Man betrachte hierzu die Menge $X \coloneq \{1,2,3,4,5,6\}$ mit der diskreten Metrik. + Diese kann keine Vektorraumstruktur haben, da $|X| = 6$ keine Primzahlpotenz ist. + \end{enumerate} +\end{lemma-nn} +\begin{proof} +Der Beweis wird aufgrund seiner Trivialität den Lesern zur Übung überlassen, da er wirklich nur Einsetzen der Definitionen ist. +\end{proof} + +Nun ein paar Charakterisierungen von kompakten Mengen in metrischen Räumen. +\begin{satz} + Im metrischen Raum $(X,d)$ sind äquivalent: + \begin{enumerate} + \item + $K ⊂ X$ ist kompakt (überdeckungskompakt) + \item + Jede Folge in $K$ besitzt mindestens einen Häufungspunkt in $K$ (abzählbar kompakt) + \item + Jede Folge in $K$ besitzt eine konvergente Teilfolge mit Grenzwert in $K$ (folgenkompakt) + \end{enumerate} +\end{satz} + +\begin{bemerkung} + Der Satz gilt so im allgemeinen Hausdorff-Raum \emph{nicht}. + Für „$(b) \Rightarrow (a)$“ benötigt man zusätzlich das zweite Abzählbarkeitsaxiom, also die Existenz einer abzählbaren Basis der Topologie. + Für „$(b) \Rightarrow (c)$“ benötigt man das erste Abzählbarkeitsaxiom, also die Existenz von abzählbaren Umgebungsbasen für jeden Punkt. +\end{bemerkung} + +\section{Vollständigkeit in metrischen Räumen und der Satz von Baire} +\begin{definition} + Eine Folge $(x_n)_{n ∈ ℕ} ⊂ X$ in $(X,d)$ heißt \emph{Cauchy-Folge}, falls zu jedem $\epsilon > 0$ ein $N = N(\epsilon )$ existiert mit $d(x_m,x_n) < \epsilon $ für alle $n,m \ge N$. +\end{definition} + +\begin{lemma} + Jede konvergente Folge $(x_n)_{n ∈ ℕ} ⊂ X$ ist auch eine Cauchy-Folge. +\end{lemma} +\begin{proof} + Sei etwa $\lim_{n→∞} x_n = x$. Sei $ε > 0$. + Da $(x_n)_{n ∈ ℕ}$ gegen $x$ konvergiert, gibt es $N ∈ ℕ$ mit $d(x_n,x)< ε/2$ für alle $n ≥ N$, also mit der Dreiecksungleichung + \[ + ∀n,m ≥ N: d(x_n,x_m) ≤ d(x_n,x) + d(x, x_m) < \frac ε 2 + \frac ε 2 = ε. + \] +\end{proof} + +\begin{definition} + Der metrische Raum $(X,d)$ heißt \emph{vollständig}, falls jede Cauchy-Folge in $(X,d)$ konvergiert. +\end{definition} + +Nicht jeder metrische Raum braucht vollständig zu sein (man betrachte hierfür z.B. $ℚ$ und die Folge der Partialsummen der Dezimalbruchentwicklung von $\sqrt 2$), +jedoch lässt sich jeder metrische Raum zu einem vollständigen Erweitern. + +\begin{satz} + Jeder metrische Raum $(X,d)$ lässt sich in einen bis auf Isometrie eindeutig bestimmten kleinsten vollständigen metrischen Raum $(\tilde X, \tilde d)$ einbetten. + Dieser Raum $(\tilde X, \tilde d)$ heißt die Vervollständigung von $(X,d)$. +\end{satz} +\begin{proof} + Zwei Cauchyfolgen $(x_n)_{n ∈ ℕ}$ und $(y_n)_{n ∈ ℕ}$ seien äquivalent, falls $d(x_n,y_n) \xrightarrow[n → \infty ]{} 0$. + Hierdurch ist eine Äquivalenzrelation definiiert. Sei $[(x_n)_{n ∈ ℕ}]$ die vom Repräsententaten $(x_n)_{n ∈ ℕ}$ erzeugte Klasse. Man setzt + \[ + \tilde X \coloneq \{ [ (x_n)_{n ∈ ℕ}] : (x_n)_{n ∈ ℕ} \text{ ist Cauchy-Folge in }(X,d)\} + \] + und + \[ + \tilde d([(x_n)_{n ∈ ℕ}],[(y_n)_{n ∈ ℕ}]) \coloneq \lim_{n → \infty } d(x_n,y_n). + \] + Dann ist $(d(x_n,y_n))_{n ∈ ℕ}$ eine Cauchy-Folge in $ℝ$, da + \[ + |d(x_n,x_m) - d(y_m,y_m) | \le \underbrace{d(x_n,x_m)}_{→ 0} + \underbrace{d(y_n,y_m)}_{→ 0}. + \] + Da $ℝ$ bekanntlich vollständig ist, existiert somit der Grenzwert. + Ferner ist $\tilde d$ Repräsentatenunabhängig, also wohldefiniert: + Seien $(\tilde x_n)$ und $(\tilde y_n)$ andere Repräsentaten. Dann ist + \[ + d(x_n,y_n) \le \underbrace{d(x_n,\tilde x_n)}_{→ 0} + d(\tilde x_n,\tilde y_n) + \underbrace{d(\tilde y_n, y_n)}_{→ 0}. + \] + Die umgekehrte Ungleichung ergibt sich aus Vertauschung der Rollen. Man rechnet leicht nach, dass $(\tilde X, \tilde d)$ ein vollständiger Raum ist. + Wir können $(X,d)$ durch die entsprechenden konstanten Folgen isometrisch in $\tilde X$ einbetten. +\end{proof} + +\begin{bemerkung-nn} + Wendet man diese Technik auf $ℚ$ mit der natürlichen Metrik an, dann erhält man $(ℝ,d)$ als vollständige Hülle. + Man beachte jedoch, dass dies nicht für die Konstruktion von $ℝ$ ausreicht, da hier schon die Existenz von $ℝ$ verwenden wird -- Aber das funktioniert größtenteils analog. +\end{bemerkung-nn} + + +\begin{satz}[Schachtelsatz]\label{schachtelsatz} + Sei $(X,d)$ ein vollständiger metrischer Raum und seien + $(x_n)_{n * ℕ} ⊂ X$ und $(r_n)_{n ∈ ℕ} ⊂ (0,\infty ) $ Folgen mit der Eigenschaft + \begin{enumerate} + \item $\cl B_{r_{n+1}}(x_{n+1}) ⊂ B_{r_n} (x_n)$ + \item $\lim_{n \to \infty } r_n = 0$. + \end{enumerate} + Dann gibt es genau ein $x_0 ∈ X$ mit $x_0 ∈ \bigcap_{n ∈ ℕ} \cl B_{r_n} (x_n)$. +\end{satz} +\begin{proof} + + Für $p ∈ ℕ$ beliebig gilt + \[ + \cl B_{r_{n+p}} (x_{n+p}) ⊂ \cl B_{r_n} (x_n). + \] + Also + \[ + d(x_{n+p},x_n) \le r_n \xrightarrow[n → \infty ]{} 0. + \] + Damit ist $(x_n){n ∈ ℕ}$ eine Cauchyfolge und damit konvergiert gegen ein $x_0 ∈ X$. + Außerdem gilt + \[ + d(x_p,x_n) \le \underbrace{d(x_0, x_{n+p})}_{→ 0 (p → \infty )} + \underbrace{d(x_{n+p},x_n)}_{ \le r_n}. + \] + Damit folgt für $p → \infty $ + \[ + d(x_0, x_n) \le r_n \quad ∀ n ∈ ℕ + \] + also $x_0 ∈ \bigcap_{n ∈ ℕ} \cl B_{r_n}(x_n)$. + Für die Eindeutigkeit sei $\tilde x_0$ ebenfalls in $\bigcap_{n ∈ ℕ} \cl B_{r_n}(x_n)$. + Dann folgt + \[ + d(x_0,\tilde x_0) \le \underbrace{d(x_0,x_n)}_{\le r_n} + \underbrace{d(x_n, \tilde x_0)}_{\le r_n} \le 2r_n \xrightarrow[n → \infty ]{} 0. + \] + Doch damit war bereits $x_0 = \tilde x_0$. +\end{proof} + +\begin{definition} + Eine Teilmenge $M$ eines metrischen Raumes $(X,d)$ heißt \emph{von erster Kategorie} oder \emph{mager}, falls sie + die Vereinigung abzählbar vieler in $X$ nirgends dichter Mengen ist. Andernfalls heißt $M$ \emph{von zweiter Kategorie}. +\end{definition} + +Der folgende Satz wird beim Beweis mehrerer fundamentaler Sätze benötigt, z.B beim Prinzip der gleichmäßigen Beschränktheit oder dem Open-Mapping-Theorem. + + +\begin{satz}[Baire]\label{baire} + Jede nichtleere offene Menge eines vollständigen metrischen Raumes $(X,d)$ ist von zweiter Kategorie (insbesondere $X$ selbst) +\end{satz} +\begin{proof} + Sei $ \emptyset \ne M ⊂ X$ offen. Wir nehmen umgekehrt an, $M$ wäre von erster Kategorie, das heißt + \[ + M ⊂ \bigcup_{n ∈ ℕ} M_n + \] + mit $M_n ⊂ X$ nirgends dicht. Wähle $x_0 ∈ M$. Da $M$ offen ist, gibt es ein $r = r_0 > 0$ mit $B_{r_0}(x_0) ⊂ M$. + Da $M_1$ nirgends dicht ist, gibt es $r_1 > 0$ und $x_1 ∈ X$ mit + \[ + B_{r_1}(x_1) ⊂ B_{r_0/2} (x_0) + \] + und $B_{r_1}(x_1) ∩ M_1 = \emptyset$. + Analog finden wir, da $M_2$ nirgends dicht ist, $r_2 > 0$ und $x_2 ∈ X$ mit + \[ + B_{r_2}(x_2) ⊂ B_{r_1/2} (x_1) + \] + und $B_{r_2}(x_2) ∩ M_2 = \emptyset$. + Durch Fortsetzen dieses Schemas finden wir eine Folge $(x_n)_{n ∈ ℕ} ⊂ X$ und Radien $(r_n)_{n ∈ ℕ} ⊂ (0,\infty )$ mit $r_n \le r/2^n \xrightarrow[n → \infty ]{} 0$. + Damit sind alle Voraussetzungen von \cref{schachtelsatz} erfüllt. Folglich existiert genau ein + \[ + \tilde x ∈ \bigcap_{n ∈ ℕ} B_{r_n} (x_n) ⊂ B_r(x_0) ⊂ M. + \] + Aber $\tilde x \not\in M_n$ für alle $n ∈ ℕ$ Folglich ist auch $\tilde x$ nicht in $\bigcup_{n ∈ ℕ} M_n = M$. Das ist ein Widerspruch. Also ist $M$ von zweiter Kategorie. +\end{proof} + +% \begin{satz}[Satz von Baire]\label{44-baire} +% Sei $(X,\T)$ ein vollständig metrisierbarer oder lokalkompakter Hausdorff\hyp{}Raum +% \begin{enumerate} +% \item +% Sei $(U_n)_{n ∈ ℕ}$ eine Folge offener, dichter Teilmengen von $X$. +% Dann ist auch $\bigcap_{n ∈ ℕ} U_n ⊂ X$ dicht. +% \item +% Sei $(A_n)_{n ∈ ℕ}$ eine Folge nirgends dichter Teilmengen. Dann ist +% $\bigcup_{n ∈ ℕ} A_n \ne X$. +% \item +% Sei $(A_n)_{n ∈ ℕ}$ eine Folge abgeschlossener Teilmengen mit +% $\bigcup_{n ∈ ℕ} A_n = X$. Dann gilt für mindestens ein $n ∈ ℕ$, dass $A_n^\circ +% \ne \emptyset$. +% \end{enumerate} +% \end{satz} +% \begin{proof} +% \begin{enumerate} +% \item +% Sei $W ⊂ X$ offen und nichtleer. Zu zeigen: $\bigcap_{n ∈ ℕ} U_n ∩ W \ne +% \emptyset$. Sei zunächst $X$ vollständig metrisierbar durch die Metrik +% $d$. Da $U_1 ∩ W$ offen und nichtleer nach Annahme gibt es $x_1 ∈ U ∩ W$ +% und $0 < r_1 < 1$ mit $B_{r_1}(x_1) ⊂ U_1 ∩ W$. Wir wählen nun induktiv +% Punkte $x_n ∈ X$ und Zahlen $0 < r_n < 1$ (für $n \ge 2$) mit folgenden +% Eigenschaften: +% \begin{enumerate}[label=(\roman*)] +% \item +% $0 < r_n < \frac 1 n$ +% \item +% $\cl{B_{r_n}(x_n)} ⊂ U_n ∩ B_{r_{n-1}} (x_{n-1})$ +% \end{enumerate} +% Dazu beachte man, dass $U_n ∩ B_{r_{n-1}}(x_{n-1})$ nichtleer und offen +% ist, also existiert $x_n$ und $\frac 1 n > \epsilon > 0$ mit $B_\epsilon (x_n) ⊂ U_n ∩ +% B_{r_{n-1}} (x_{n-1})$ und $r_n = \frac \epsilon 2$ ist wie gewünscht. Für $m +% \ge n$ impliziert (ii), dass $x_m ∈ B_{r_n}(x_n)$ und aus (i) folgt, +% dass die Folge $(x_n)_{n ∈ ℕ}$ damit eine Cauchyfolge ist. Damit +% konvergiert $(x_n)_{n ∈ ℕ}$ gegen ein $x ∈X$. Sei nun $N ∈ ℕ$ und $m > +% N$. Dann folgt aus $x_m ∈ B_{r_N}(x_N)$, dass +% \begin{align*} +% x &= \lim_{m → \infty } x_m ∈ \cl{B_{r_N}(x_n)} ⊂ U_N ∩ B_{r_{N-1}}(x_{N-1}) \\ +% & ⊂ U_N ∩ B_{r_1}(x_1) ⊂ U_N ∩ W, +% \end{align*} +% also $x ∈ \bigcap_{n ∈ ℕ} U_N ∩ W$. + +% Sei Nun $X$ lokalkompakt. Da $U_1 ∩ W$ offen und nichtleer ist, gibt es +% $x ∈ U_1 ∩ W$, und es ist $U_1 ∩ W ∈ \U_x$. Wähle $B_1 ∈ \U_x$ kompakt +% mit $B_1 ⊂ U_1 ∩ W$. Wir konstruieren nun sukzessive kompakte Mengen +% $B_k$ (zu $k \ge 2$) mit folgenden Eigenschaften: +% \begin{enumerate}[label=(\roman*)] +% \item +% $B_k ⊂ B_{k-1}$ +% \item +% $\emptyset \ne B_k^\circ ⊂ B_k ⊂ U_k$. +% \end{enumerate} +% Ist $B_{k-1}$ gegeben, so ist wegen $B_{k-1}^\circ \ne \emptyset$ und +% der Dichtheit von $U_k$ der Schnitt $B_{k-1}^\circ ∩ U_k$ offen und +% nichtleer. Es gibt also ein $x ∈ B_{k-1}^\circ ∩ U_k$ und damit auch +% eine kompakte $x$-Umgebung $B_k ⊂ U_k ∩ B_{k-1}$. +% Die Familie $(B_k)_{k ∈ ℕ}$ nichtleerer Teilmengen der kompakten Menge +% $B_1$ ist absteigend, besitzt also die endliche Durschnittseigenschaft. +% Da $X$ hausdorffsch und die $B_k$ kompakt sind, ist zudem jedes $B_k$ +% abgeschlossen, somit folgt +% \[ +% \emptyset \ne \bigcap_{k ∈ ℕ} B_k ⊂ \bigcap _{k ∈ ℕ}U_k +% \] +% sowie +% \[ +% \bigcap_{k ∈ ℕ} B_k ⊂ B_1 ⊂ W. +% \] +% Insgesamt also +% \[ +% \emptyset \ne \bigcap_{k ∈ ℕ} B_k ⊂ \bigcap_{k ∈ ℕ} U_k ∩ W. +% \] +% \item +% Für $n ∈ ℕ$ Sei $U_n = \complement_X \cl{A_n}$. Dann ist $U_n$ offen und +% dicht. Mit Teil (a) folgt, dass auch $\bigcap_{n ∈ ℕ U_n }$ dicht, und +% somit insbesondere nicht leer, ist. Also ist $\bigcup_{n ∈ ℕ} A_n +% ⊂ \bigcup_{n ∈ ℕ} \cl{A_n} = (\bigcap_{n ∈ ℕ} U_n)^\complement \ne X$. +% \item +% Das ist eine direkte Konsequenz aus (b). +% \end{enumerate} +% \end{proof} + + +%%% Local Variables: +%%% mode: latex +%%% TeX-master: "funkana-ebook" +%%% End: diff --git a/ch03-topologisch-lineare-raeume.tex b/ch03-topologisch-lineare-raeume.tex new file mode 100644 index 0000000..d58b433 --- /dev/null +++ b/ch03-topologisch-lineare-raeume.tex @@ -0,0 +1,1480 @@ +\chapter{Topologische lineare Räume} +Erklärtes Ziel dieses Kapitels wird sein, die beiden Strukturen aus den vorherigen beiden Kapiteln, also die Topologie und den linearen Raum zusammenzuführen. +\begin{definition} + Ein linearer Raum $X$ über dem Körper $\K$ mit Topologie $\T$ heißt \emph{topologischer linearer Raum}, falls die Vektorraumoperationen ($+ : X×X → X$ und $\cdot: \K×X → X$) stetig sind. +\end{definition} + +\begin{bemerkung-nn} + Stetigkeit der Vektorraumoperationen sollte als minimales Kompatibilitätskriterium der beiden Strukturen gefordert werden. + Tatsächlich ist es im Allgemeinen gar nicht erfüllt. Erst im normierten Raum bekommt man diese Stetigkeit geschenkt. +\end{bemerkung-nn} + +\section{Normierte Räume} +\begin{definition} + Sei $X$ ein linearer Raum über $\K$. Die Abbildung $\norm\cdot: X → [0,\infty )$ + heißt \emph{Norm} auf $X$, falls für alle $x, y ∈ X, \alpha ∈ K$ gilt: + \begin{enumerate} + \item $\norm x = 0 \Longleftrightarrow x = 0$ (Definitheit) + \item + $\norm{\alpha x} = |\alpha | \norm x$ (Homogenität) + \item + $\norm{x+y} \le \norm x + \norm y$ (Dreiecksungleichung) + \end{enumerate} + $(X,\norm\cdot)$ heißt dann \emph{normierter Raum}. +\end{definition} + +\begin{bemerkung} + Durch $d(x,y) \coloneq \norm{x-y}$ wird ein normierter Raum auch ein metrischer, also insbesondere auch ein topologischer Raum. + Diese induzierte Topologie auf $(X, \norm\cdot)$ heißt \emph{Normtopologie}. + + Ohne die lineare Struktur macht der normierte Raum gar keinen Sinn, da für die Definition einiger der Normaxiome die Vektorraumoperationen verwendet werden. +\end{bemerkung} + +\begin{beispiele} + \begin{enumerate} + \item + Betrachte den $ℝ^n$ mit $\norm x _{p} \coloneq \left( \sum_{i=1}^n |x_i|^p \right)^{1/p}$ mit $1 \le p < \infty $ ist ein normierter Raum, + genauso wie mit $\norm{x}_{\infty } \coloneq \max_{1 \le i \le n} |x_i|$. + Insbesondere gibt es im $ℝ^n$ überabzählbar viele verschiedene Normen. + Wir werden jedoch später sehen, dass diese Normen alle die gleiche Topologie erzeugen. + \item + Der Raum aller stetigen Funktionen auf einem kompaktem Intervall $C[a,b]$ mit $\norm{x}_{\infty } \coloneq \max_{t ∈ [a,b]} |x(t)|$ ist ein normierter Raum. + Außerdem wird durch + \[ + \norm x \coloneq ∫_a^b |x(t)| dt + \] + ebenfalls eine Norm definiert. + \item + Sei $\Omega ⊂ ℝ^n$ offen und beschränkt. Dann wird $C(\cl{\Omega})$ mit + \[ + \norm{x}_{\infty } \coloneq \max_{t ∈ \cl \Omega} |x(t)| + \] + auch zu einem normierten Raum. + \item + $L^p(\Omega) = \L^p(\Omega)/\mathcal N$, wobei $\mathcal N = \{ f: \Omega → \R, f(t) = 0 \text{ fast überall}\}$ ist mit + \[ + \norm x \coloneq \left(∫_{\Omega} |x(t)|^p dt \right)^{1/p} + \] + ein normierter Raum, wobei $1 \le p < \infty $. + \item + $\ell^p$ mit + \[ + \norm x _{p} \coloneq \left( \sum_{i=1}^n |x_i|^p \right)^{1/p} + \] + ist ebenfalls ein normierter Raum, wobei $1 \le p < \infty $. + \end{enumerate} +\end{beispiele} + +\begin{lemma} + Sei $(X, \norm\cdot)$ ein normierter Raum. Dann sind die Abbildungen $+$, $\cdot$ und $\norm\cdot$ stetig. +\end{lemma} +\begin{proof} + Für beliebige Folgen $(x_n)_{n ∈ ℕ},(y_n)_{n ∈ ℕ} ⊂ X, (\alpha _n)_{n ∈ ℕ}$ mit $\lim x_n = x$, $\lim y_n = y$, $\lim \alpha _n = \alpha $ gelten + \[ + \norm{(x_n + y_n) - (x+y)} \le \norm{x-x_n} + \norm{y -y_n} + \] + sowie + \[ + \norm{\alpha _nx_n - \alpha x} \le |\alpha _n| \norm{x_n-x} + \norm{x} |\alpha _n - \alpha | + \] + und + \[ + |\norm{x_n} - \norm{x}| \le \norm{x_n - x} + \] + nach der umgekehrten Dreiecksungleichung. + Folglich sind die zu betrachtenden Abbildungen alle folgenstetig, und, da metrische Räume stets dem ersten Abzähhlbarkeitsaxiom genügen, auch stetig. +\end{proof} + +\begin{korollar} + Jeder normierte Raum versehen mit der Normtopologie ist ein topologischer linearer Raum. + Deshalb ist auch keine Unterscheidung zwischen normierten Räumen und normierten topologischen linearen Räumen nötig. +\end{korollar} + +\section{Topologische lineare Räume} +\begin{bemerkung-nn} + Hierbei sei stets die Topologie von $X×X$ die Produktopologie, bei den Körpern $\K = \begin{cases} ℝ \\ ℂ \end{cases}$ die übliche Topologie. + Wir schreiben im Folgenden für Mengen $M, M_1, M_2 ⊂ X$ und $\alpha ⊂ \K$ nun + \[ + M_1 + M_2 \coloneq s(M_1,M_2) \coloneq \{x+y : x ∈ M_1, y ∈ M_2\}, + \] + \[ + A \cdot M \coloneq m(A,M) \coloneq \{ \alpha x: \alpha ∈ A, x ∈ M\}. + \] +\end{bemerkung-nn} + +\begin{lemma} + Hat der topologische Raum $(X,\T)$ auch eine lineare Struktur, so sind äquivalent: + \begin{enumerate} + \item Die Addition $s$ ist stetig. + \item + Für beliebiges $x, y ∈ X$ gilt: Zu jeder Umgebung $O_{x+y} ∈ \T$ existieren Umgebungen $O_x ∈ \T$ von $x$ und $O_y ∈ \T$ von $y$ mit $O_x + O_y ⊂ O_{x+y}$ + \end{enumerate} +\end{lemma} +\begin{proof} + $s$ ist stetig in $(x,y)$ genau dann, wenn zu jeder Umgebung $O_{x,y} ∈ \T_X$ + von $(x,y)$ existiert eine Umgebung $U ⊂ \T_{X×X}$ von $(x,y)$ mit $s(U) ⊂ O_{x+y}$. + Nach Definition der Produkttopologie existieren dann Umgebungen $O_x ∈ \U_x$ und $O_y ∈ \U_y$ mit $O_x × O_y ⊂ U$. + Damit ist + \[ + O_x + O_y = s(O_x, O_y) = s(O_x × O_y) ⊂ s(U) ⊂ O_{x+y}. + \] + Analog zeigt man die entsprechende Aussage für die skalare Multiplikation: +\end{proof} +\begin{lemma} + Hat der topologische Raum $(X,\T)$ auch eine lineare Struktur, so sind äquivalent: + \begin{enumerate} + \item Die Addition $m$ ist stetig. + \item + Für beliebiges $\alpha ∈ \K, x ∈ X$ gilt: Zu jeder Umgebung $O_{\alpha x} ∈ \T$ existieren Umgebungen $O_x ∈ \T$ von $x$ und $O_\alpha ∈ \T$ von $y$ mit $O_\alpha × O_x ⊂ O_{\alpha x}$. + \end{enumerate} +\end{lemma} + +Betrachtet man insbesondere die Stetigkeit am Punke $\alpha =0$ und $x ∈ X$ beliebig, dann gilt also: +Für jede Umgebung $O_0 ∈ \U_0 ⊂ X$ existiert eine Umgebung $O_x ∈ \U_x$ und ein $r > 0$, so dass +\[ + ∀β: |β| 0 ∃ \delta > 0 ∃ r> 0 ∀β ∈ \K ∀y ∈ X: + \begin{rcases} + |β - \alpha | < r \\ + d(x,y) < \delta + \end{rcases} + \implies d(βy,\alpha x) < \epsilon +\] + + +\begin{lemma} + \label{lemma-metrischer-linearer-raum-charak} + Sei $(X,d)$ ein metrischer Raum mit linearer Struktur und mit einer translationinvarianten Metrik. + Dann ist $X$ mit der von $d$ erzeugten Topologie ein \emph{metrischer linearer Raum} genau dann, wenn für alle $\alpha ∈ \K, x ∈ X$ und beliebige Nullfolgen $(x_n)_{n ∈ ℕ} ⊂ X, (\alpha _n)_{n ∈ ℕ)} ⊂ \K$ gilt + \begin{gather*} + \alpha x_n \xrightarrow[n → \infty ]{} 0 \\ + \alpha x_n \xrightarrow[n → \infty ]{} 0 \\ + \alpha _nx_n \xrightarrow[n → \infty ]{} 0 + \end{gather*} +\end{lemma} +\begin{proof} + „$⇒$”: Skalare Multiplikation ist im metrischen linearen Raum stetig, also folgen die Aussagen sofort. + + „$⇐$“: Wegen der Äquivalenz von Stetigkeit und Folgenstetigkeit ist zu zeigen + \[ + \begin{rcases} + \alpha _n \xrightarrow[n → \infty ]{} \alpha ∈ \K \\ + x_n \xrightarrow[n → \infty ]{} x ∈ X + \end{rcases} + \implies \alpha _n x_n \xrightarrow[n → \infty ]{} \alpha x. + \] + + Sei dazu $z_n \coloneq x_n - x ∈ X$, $γ_n \coloneq \alpha _n - \alpha ∈ \K$. Dann ist + \[ + γ_n z_n + γ_n x + \alpha z_n = (\alpha _n - \alpha )(x_n-x) + (\alpha _n-\alpha ) x + \alpha (x_n-x) + = \alpha _n x_n - \alpha ×. + \] + Somit ist + \begin{align*} + d(\alpha _nx_n,\alpha x) &= d(\alpha nx_n - \alpha x,0) = d(γ_nz_n + γnx + \alpha z_n, 0) \\ + &\le \underbrace{d(γ_nz_n,0)}_{→ 0} + \underbrace{d(γ_nx, 0)}_{→ 0} + +\underbrace{d(\alpha z_n, 0)}_{→ 0} \xrightarrow{n → \infty } 0. + \end{align*} + Da die Addition ohnehin immer stetig ist, sind wir fertig. +\end{proof} + + +\begin{definition} + Eine Abbildung $|\cdot|: X → [0,\infty )$ heißt \emph{Quasi-Norm} auf dem Linearen + Raum $X$, falls gilt: + \begin{enumerate}[label=(Q\arabic*)] + \item + $|x| \ge 0$ für alle $x ∈ X$ und $|x| = 0$ genau dann, wenn $x = 0$. + \item + $|-x| = |x|$ für alle $x ∈ X$ + \item + $|x+y| \le |x| + |y|$ für alle $x,y ∈ X$ + \item + $|\alpha x_n| \xrightarrow[n → \infty ]{} 0$ für $\alpha ∈ \K$, falls $|x_n| → 0$ + \item + $|\alpha _nx| \xrightarrow[n → \infty ]{} 0$ für $x ∈ X$, falls $|\alpha _n| → 0$ + \item + $|\alpha _nx_n| \xrightarrow[n → \infty ]{} 0$ falls $|x_n| → 0$ und $|\alpha _nx_n| → 0$ + \end{enumerate} + $(X,|\cdot|)$ heißt dann \emph{quasi-normierter} Raum. +\end{definition} + +\begin{bemerkung} + Jeder normierte Raum ist auch ein quasi-normierter Raum. +\end{bemerkung} + +\begin{satz} + \begin{enumerate} + \item + Ist $|\cdot|$ eine Quasi-Norm auf $X$, so wird durch $d(x,y) \coloneq |x-y|$ eine translationsinvariante Metrik definiert, welche $X$ zu einem metrischen linearen Raum macht. + \item + Ist $(X,d)$ ein metrischer linearer Raum mit translationsinvarianter Metrik $d$, so ist + $(X,|\cdot|)$ mit $|x| \coloneq d(x,0)$ ein quasi-normierter Raum. + \end{enumerate} +\end{satz} +\begin{proof} + Das folgt direkt aus den Axiomen und \cref{lemma-metrischer-linearer-raum-charak}. +\end{proof} + + +Speziell für die Anwendung sehr wichtige metrische lineare Räume werden von Semi-Normen erzeugt. + +\begin{definition} + Sei $X$ ein linearer Raum. + Eine Abbildung $p: X → ℝ$ heißt \emph{Semi-Norm} oder \emph{Halbnorm}, falls folgendes gilt: + \begin{enumerate}[label=(S\arabic*)] + \item + $∀x ∈ X: p(x) \ge 0$ (positiv) + \item + $∀ x ∈ X, \alpha ∈ \K: p(\alpha x) = |\alpha | p(x)$ (Homogenität) + \item + $∀ x, y ∈ X: p(x+y) \le p(x) + p(y)$ (Dreiecksungleichung) + \end{enumerate} + $(X,p)$ heißt dann \emph{semi-normierter} Raum. +\end{definition} + +\begin{beispiel-nn} + $\L^p(\Omega)$ ist ein semi-normierter Raum. +\end{beispiel-nn} + +\begin{bemerkung} + Jeder semi-normierte Raum $(X,p)$ erzeugt einen normierten Raum $(X/N,p)$, wobei $N = \{ x ∈ X: p(x) = 0\}$ ein linearer Unterraum ist. +\end{bemerkung} + +\begin{satz} + \label{satz-abzaehlbares-prod-seminormierter-raeume} + Es seien $p_n: X → ℝ, n ∈ ℕ$ abzählbar viele Semi-Normen auf einem linearen Raum mit der Eigenschaft + \begin{equation} + p_n(x) = 0 \text{ für alle n ∈ ℕ } \implies x = 0. \label{eq:seminorm-folge-blub} + \end{equation} + Dann ist + \[ + d(x,y) \coloneq \sum_{n = 1}^\infty 2^{-n} \frac{p_n(x-y)}{1+p_n(x-y)} + \] + eine translationsinvariante Metrik auf $X$, welche $X$ zum metrischen linearen Raum macht. +\end{satz} + +\begin{bemerkung} + $p_n: X → ℝ$ sind auf $(X,d)$ stetig. Das folgt aus (für $x_i → x_0$ in $X$) + \[ + |p_n(x_i) - p_n(x_0)| \le p_n(x_i-x_0) \xrightarrow{} 0 + \] + und einer Übungsaufgabe. +\end{bemerkung} + +\begin{satz} + \label{satz-umgebungsbasis-produkt-von-seminorm} + Sei $(X,d)$ der in \cref{satz-abzaehlbares-prod-seminormierter-raeume} gegebene metrische lineare Raum (mit der von der Metrik erzeugten Topologie). + Dann bilden die Mengen ($\epsilon _n > 0$) + \[ + U (p_n,\epsilon _n) \coloneq \bigcup B^{p_n}_{\epsilon _n}(0) + = \{ x ∈ X: p_n(x) < \epsilon _n\} + \] + und deren endliche Durchschnitte eine Umgebungsbasis von $0 ∈ X$ +\end{satz} + +\begin{bemerkung-nn} + Nach dem Invarianzprinzip ist damit durch $\bigcup B^{p_n}_{\epsilon _n}$ die ganze Topologie bestimmt. + Mit anderen Worten: Die Topologie welche über die Metrik bestimmt ist, ist dieselbe wie die, welche von den + $U(p_n,\epsilon _n)$ und endlichen Schnitten davon erzeugt wird. +\end{bemerkung-nn} +\begin{proof}[\cref{satz-umgebungsbasis-produkt-von-seminorm}] + Zunächst ist $U (p_n,\epsilon _n) ∈ \T$: + Sei $n ∈ ℕ$ und $\epsilon _n > 0$ fest und $y ∈ U(p_n,\epsilon _n)$ beliebig gegeben. + Dann ist $p_n(y) < \epsilon _n$. Dann wähle $ρ = ρ(y) > 0$, so dass $p_n(y) + ρ < \epsilon _n$. + Dann gilt für $r \coloneq 2^{-n} \frac{ρ}{1+ρ} > 0$: + \[ + x ∈ B_r(y) \implies p_n(x+r) < ρ. + \] + Dazu ist + \[ + \frac{p_n(x-y)}{1+p_n(x-y)} \le 2^n \underbrace{d(x,y)}_{< r} < 2^n r = \frac{ρ}{1+ρ}, + \] + also $p_n(x-y) < ρ$. Mit diesem $r$ gilt $B_r(y) ⊂ U(p_n,\epsilon _n)$: + Sei $x ∈ B_r(y)$. Dann gilt + \[ + p_n(x) \le \underbrace{p_n(x-y)}_{< ρ} + p_n(y) < p_n(y) + ρ = \epsilon _n + \] + wie gewünscht. + + + Sei $ B_r(0), r > 0$ gegeben. + Wähle $n_0 ∈ ℕ$ mit + \[ + \sum_{n=n_0}^\infty 2^{-n} < \frac r 2. + \] + + mit $\epsilon \coloneq \frac r 2 $ gilt dann + \[ + \bigcap_{n=1}^{n_0} U(p_(,\epsilon ) ⊂ B_r(0). + \] + Sei dazu $x ∈ \bigcap_{n=1}^{n_0} U(p_n,\epsilon )$ beliebig. + Dann ist + \[ + d(x,0) \le \sum_{n=1}^{n_0} 2^{-n} \frac{p_n(x)}{1+p_n(x)} + \sum_{n=n_0}^\infty 2^{-n} < \epsilon \sum_{n=1}^{n_0} 2^{-n} + \frac r 2 < \epsilon + \frac r 2 = r, + \] + somit also $x ∈ B_r(0)$. +\end{proof} + +\begin{bemerkung} + Die Mengen $U(p_n,\epsilon _n)$ und deren endlichen Schnitte sind konvexe Mengen, das heißt + \[ + x, y ∈ U(p_n,\epsilon _n),\alpha ∈ [0,1] \implies \alpha x+(1-\alpha )y ∈ U(p_n,\epsilon _n) + \] +\end{bemerkung} +\begin{proof} + Es ist + \[ + p_n(\alpha x + (1-\alpha )y) \le |\alpha | \underbrace{p_n(x)}_{< \epsilon _n} + |1-\alpha |\underbrace{p_n(y)}_{< \epsilon _n} = \epsilon _n. + \] +\end{proof} + +Also besitzt der in \cref{satz-abzaehlbares-prod-seminormierter-raeume} gewonne metrische lineare Raum $(X,d)$ eine Umgebungsbasis von $0$, die nur aus konvexen elementen besteht. + +\begin{definition} + Ein topologischer linearer Raum $(X,\T)$, in dem jedes $x ∈ X$ eine Umgebungsbasis besitzt, die nur aus konvexen Mengen besteht, heißt \emph{lokalkonvex}. +\end{definition} + +\begin{satz} + Sei $X$ ein linearer Raum mit Semi-Normen $p_i, i ∈ I$, wobei $I$ eine beliebige Indexmenge ist, mit der Eigenschaft + \[ + p_i(x) = 0 \text { für alle } i ∈ I \implies x = 0. + \] + Dann sind die Mengen + \[ + U(p_i,\epsilon _i) = \{ x ∈ X: p_{(x) < \epsilon _i}\}, \quad \epsilon _i > 0, i ∈ I + \] + und deren endliche Schnitte eine konvexe Umgebungsbasis von $0 ∈ X$. + Die dadurch gewonne Topologie $\T$ macht $X$ zu einem \emph{lokalkonvexen Hausdorff"=Raum}. +\end{satz} + +\section{Beispiele} +Wir werden die unten angegebenen Beispiele auch gleich auf Vollständigkeit untersuchen. + +\begin{definition} + \begin{enumerate} + \item + Ein metrischer linearer Raum $(X,d)$ der vollständig ist, heißt \emph{Fréchet"=Raum}. + \item + Ein normierter Raum $(X,\norm\cdot)$, der vollständig ist, heißt \emph{Banach"=Raum}. + + \end{enumerate} +\end{definition} + +\begin{beispiel-nn}[$\ell^p$-Räume] + \begin{enumerate} + \item + $(\ell^p,\norm\cdot_p)$, $1 \le p < \infty $ ist normierter Raum mit + \[ + \norm x _p = \left( \sum_{i=1}^\infty |x_i|^p \right)^{1/p}. + \] + \item + $(\ell^\infty ,\norm\cdot_\infty)$, ist normierter Raum mit $\norm x _\infty = \sup_{i ∈ ℕ} |x_i|$. + \item + $(\ell^p,|\cdot|_p = \norm\cdot_p^p)$, $0 \le p < 1$ ist quasi-normierter Raum. + \end{enumerate} +\end{beispiel-nn} + +\begin{bemerkung} + Für $0 < p < q \le \infty $ gilt $\ell^p ⊂ \ell^q ⊂ \ell^\infty $. +\end{bemerkung} +\begin{beweis} + Sei $x ∈ \ell^p$ mit $|x| = 1 = \sum_{i ∈ ℕ} |x_i|^p$. + Dann ist für alle $i ∈ ℕ$ $|x_i|^p \le 1$, also auch $|x_i| < 1$. + Dann folgt auch $\sum_{i ∈ ℕ} |x_i|^q < 1$, also $x ∈ \ell^q$ und $\sup_{i ∈ ℕ} |x_i| \le 1$, also $x ∈ \ell^\infty $. +\end{beweis} + + +\begin{satz} + Für $1 \le p \le \infty $ ist $(\ell^p,\norm\cdot_p)$ ein Banachraum. + Für $0 < p < \infty $ ist $(\ell^p,|\cdot|_p)$ ein Fréchet-Raum. +\end{satz} +\begin{proof} + Nur für $1 \le p < \infty $. + Sei dazu $(x_n)_{n ∈ ℕ} ⊂ \ell^p$ eine Cauchy-Folge, also + $x_n = (ξ_k^n)_{k ∈ ℕ}$ und für jedes $\epsilon > 0$ gibt es ein $n_0$ mit + \[ + ∀n,m > n_0: \norm{x_n-x_m}_p = \left( \sum_{k=1}^\infty |ξ_k^n-ξ_k^m|^p \right)^{1/p} < \epsilon . + \] + Sei $k_0 ∈ ℕ$ beliebig. Dann ist $(ξ_k^n)_{n ∈ ℕ}$ + eine Cauchy-Folge in $\K$, besitzt also einen Grenzwert $ξ_{k_0}$. + Setze nun $x \coloneq (ξ_k)_{k ∈ ℕ} ∈ \K^\infty = s$. Wir vermuten $x$ als Grenzwert unserer Cauchy-Folge. + Also müssen wir zeigen, dass $x ∈ \ell^p$, und dass unsere Folge tatsächlich gegen $x$ konvergiert. + + Es gilt + \[ + \norm{x_n}_! \le \underbrace{\norm{x_n-x_{n_0}}}_{< \epsilon } + \norm{x_{n_0}} \quad \forall n \ge n_0 + \] + Deshalb existiert ein $M > 0$ mit $\norm{x_n}_p < M$ für alle $n ∈ ℕ$, also + \[ + \sum_{k=1}^N |ξ_k^n|p < \sum_{k =1}^\infty |ξ_k^n|^p \le M^p < \infty . + \] + Also haben wir + \[ + \sum_{k=1}^N |ξ_k^p| \le M^p \quad ∀ n ∈ ℕ, + \] + also durch Grenzwertbildung $N → \infty $ auch $\norm{x}_p^p \le M^p$ bzw. $x ∈ \ell^p$. + + + Ferner haben wir + \[ + \sum_{k=1}^N |ξ_k^n-ξ_k^m|^p < \epsilon ^p \quad ∀ N ∈ ℕ, n, m \ge n_0(\epsilon ). + \] + Für $n → \infty $ folgt + \[ + \sum_{k=1}^N |ξ_k-ξ_k^m|^p < \epsilon ^p \quad ∀N ∈ ℕ, m \ge n_0, + \] + und mit $N → \infty $ + \[ + \sum_{k=1}^\infty |ξ_k-ξ_k^m|^p < \epsilon ^p \quad ∀m \ge n_0, + \] + also die Konvergenz. +\end{proof} +\begin{beispiel-nn} + Betrachte den Folgenraum $S = \K^\infty = \{x = (ξ_n)_{n ∈ ℕ}, ξ_n ∈ \K\}$. + Dann ist + \[ + p_n(x) \coloneq |ξ_n|, \quad p_n: \K^\infty → ℝ + \] + eine abzählbare Familie von Halbnormen mit + \[ + p_n(x) = 0 ∀n ∈ ℕ \implies x = 0 ∈ \K^\infty + \] + Nach \cref{satz-abzaehlbares-prod-seminormierter-raeume} folgt, dass $(\K^\infty , d)$ mit + \[ + d(x,y) \coloneq \sum_{n ∈ ℕ} 2^{-n} \frac{p_n(x-y)}{1+p_n(x-y)} + \] + ein metrischer linearer Raum ist. + Der Konvergenzbegriff entspricht gerade der komponentenweisen Konvergenz, das heißt, für eine Folge $(x_k)_{k ∈ ℕ}$ mit $x_k = (ξ^k_n)_{n ∈ ℕ}$ gilt + \begin{align*} + x_k \xrightarrow[k→\infty ]{} 0 + &\gdw d(x_n,0) \xrightarrow[k→\infty ]{} 0 \\ + &\gdw p_n(x_k) \xrightarrow[k→\infty ]{} ∀ n ∈ ℕ \\ + &\gdw |ξ_n^k| \xrightarrow[k→\infty ]{} 0 ∀ n ∈ ℕ. + \end{align*} + + Wir fragen uns nun, ob auf dem $\K^\infty $ auch eine Topologie existiert, so dass der induzierte Konvergenzbegriff der der gleichmäßigen Konvergenz in allen Komponenten entspricht. + Also + \[ + x_k \xrightarrow[k → \infty ]{\text{glm}} 0 ∈ \K^\infty \gdw ∀\epsilon > 0 ∃ k_0 ∈ ℕ: |ξ_n^k| < \epsilon ∀ k \ge k_0 ∀n ∈ ℕ. + \] + Wenn $\K^\infty $ ein topologischer linearer Raum sein soll, ist das nicht möglich. Notwendig wäre, dass für eine Folge $x  ∈ \K^\infty $ + \[ + \alpha _k \xrightarrow[k → \infty ]{} 0 \text{ in } \K \implies \alpha _k x \xrightarrow[k→\infty ]{} \text{ in } X = \K^\infty . + \] + Wähle dazu die Nullfolge $(\alpha _k)_{k ∈ ℕ} = (1/k)_{k ∈ ℕ} ⊂ ℝ$, $x= (n)_{n ∈ ℕ} ⊂ X$. Dann ist + \[ + \alpha _k x = (n/k)_{n ∈ ℕ} ∈ \K^\infty + \] + zwar eine Nullfolge in $\K^\infty$ ist, diese Konvergenz ist jedoch nicht gleichmäßig in $n$. + Man kann zeigen, dass $\K^\infty $ mit $d$ vollständig, also ein Fréchet-Raum, ist. + Ist $\K^\infty $ auch normierbar? + Also gibt es auf $\K^\infty $ eine Norm, welche die gleiche Topologie erzeugt wie die $d$? + Auch das ist nicht möglich: +\end{beispiel-nn} +\begin{lemma} + \label{lemma-s-metrikkugeln-enthalten-unterraeme} + In $(\K^\infty ,d)$ gilt: + \begin{enumerate} + \item + $B_1(0) = \K^\infty $ + \item + Betrachte den linearen Unterraum $M_{n_0} \coloneq \{ x = (ξ_n)_{n ∈ ℕ}$ mit $ξ_n = 0$ für $n = 1,…,n_0 \}$. + Dann gibt es für jeden Radius $r > 0$ ein $n_0 ∈ ℕ$, so dass $M_{n_0} ⊂ B_{r}(0)$. + Das heißt, jede noch so kleine Metrikkugel enthält einen nichttrivialen Unterraum. + \end{enumerate} +\end{lemma} +\begin{proof} + \begin{enumerate} + \item + Das ist trivial. + \item + Sei $r > 0$ gegeben. + Wähle nun $n_0$, so dass $\sum_{n=n_0+1}^\infty 2^{-n} < r$. + Dann gilt + \[ + ∀ x ∈ M_{n_0}: d(x,0) = + \sum_{n ∈ ℕ} 2^{-n} \frac{p_n(x)}{1+p_n(x)} = + \sum_{n=n_0}^\infty 2^{-n} \frac{p_n(x)}{1+p_n(x)} \le + \sum_{n=n_0}^\infty 2^{-n} < r. + \] + \end{enumerate} +\end{proof} + +Wäre nun die Topologie auf $(\K^\infty ,d)$ nun auch von einer Norm erzeugt, dann wären die Normkugeln +\[ + B_r^{\norm\cdot}(0) = \{ x ∈ \K^\infty : \norm x < \tilde r \} +\] +auch eine Umgebungsbasis der Null. +Das heißt insbesondere würden wir zu jedem $\tilde r$ ein $r$ finden, so dass $0 ∈ B_r^d(0) ⊂ B_r^{\norm\cdot} (0)$. +Mit \cref{lemma-s-metrikkugeln-enthalten-unterraeme} folgt also +\[ + M_{n_0} ⊂ B_r(0) ⊂ B_r^{\norm\cdot}(0) +\] +für ein geeignetes $n_0$. +Sei nun ein $0 \ne x ∈ M_{n_0}$. Dann ist, da $M_{n_0}$ ein Unterraum ist, auch $\alpha x ∈ M_{n_0}$ für alle $\alpha ∈ \K$. +Das heißt, +\[ + |\alpha | \cdot \norm x = \norm{\alpha x} < \tilde r \text{ für alle } \alpha ∈ \K, +\] +was bereits $\alpha = 0$ impliziert. Das ist ein Widerspruch. + + +\begin{beispiel-nn}[Räume beschränkter Funktionen] + Sei $S$ eine beliebige Menge und $B(S) \coloneq \{ f: S → \K, f(s)$ ist beschränkt $\}$. + Dann wird $B(S)$ mit + \[ + \norm f _{B(S)} \coloneq \sup_{x ∈ S} |f(x)| < \infty , + \] + der $\sup$-Norm, zu einem Banachraum. + Dabei ist offensichtlich, dass $\norm\cdot_{B(S)}$ tatsächlich eine Norm ist, und wir werden in einer Übung zeigen, dass die induzierte Metrik tatsächlich vollständig ist. +\end{beispiel-nn} + +\begin{lemma-nn} + \label{lemma-vollst-ubertragt-abgeschl-teilmengen} + Sei $(X,d)$ ein metrischer Raum, $Y ⊂ X$. Es gilt + \begin{enumerate} + \item + Wenn $(X,d)$ vollständig ist und $Y$ abgeschlossen, dann ist auch $(Y,d|_{Y×Y}$ vollständig. + \item + Wenn $(Y,d|_{Y×Y}$ vollständig ist, so ist $Y$ abgeschlossen in $(X,d)$. + \end{enumerate} +\end{lemma-nn} +\begin{proof} + Übungsaufgabe. +\end{proof} + + +\begin{beispiel-nn}[Räume stetiger Funktionen] + Sei $K ⊂ ℝ^n$ kompakt, also nach Heine-Borel abgeschlossen und beschränkt. + Dann ist + \[ + C(k) = \{ f: K → \K, f \text{ stetig} \} + \] + ein normierter Raum mit + \[ + \norm{f}_{C(K)} = \norm{f}_{\infty } = \max_{t ∈ K} |f(t)|, + \] + der Maximumsnorm. + Dieses Maximum wird tatsächlich immer angenommen, da $K$ kompakt ist (Satz von Minimum und Maximum). + Insbesondere sind alle stetigen Funktionen auf $K$ beschränkt. Damit gilt offensichtlich $C(K) ⊂ B(K)$ und $\norm{f}_{C(K)} = \norm{f}_{B(K)}$ für alle $f ∈ C(K)$. + Da jede stetige Funktion auf kompakten Teilmengen von metrischen Räumen auch gleichmäßig stetig ist, das heißt + \[ + ∀ \epsilon > 0 ∃ \delta > 0: \left( |t_1-t_2| < \delta \implies |f(t_1)-f(t_2)| < \epsilon \right) ∀ t_1,t_2 ∈ K + \] +\end{beispiel-nn} + +\begin{lemma} + $C(K)$ ist ein abgeschlossener Unterraum von $(B(K), \norm\cdot_{B(K)})$ und somit insbesondere auch (mit \cref{lemma-vollst-ubertragt-abgeschl-teilmengen}) vollständig. +\end{lemma} +\begin{proof} + Sei $(f_i)_{i ∈ ℕ}$ eine konvergente (in $(B(K),\norm\cdot_{B(K)})$) Folge in $C(K)$. + Dann existiert ein $f ∈ B(K)$ mit $f_i \xrightarrow[i → \infty ]{\norm{\cdot}_{B(K)}} f$. + Wir müssen zeigen, dass $f$ bereits stetig ist. + Für beliebige $t₁, t_2 ∈ K$ gilt + \[ + |f(t_1)-f(t_2) | \le \underbrace{|f_i(t_1)-f_i(t_2)|}_{< \epsilon /3 \text{ für } |t_1-t_2| < \delta ^{(i)}(\epsilon )} + 2 \underbrace{\norm{f_i - f}_{B(K)}}_{< \epsilon /3 \text{ für } i > i_0} < \epsilon . + \] + Damit ist $f$ auch gleichmäßig stetig, also insbesondere auch stetig und in $C(K)$. +\end{proof} +Das heißt, die Stetigkeit der Folgenglieder $(f_i)_{i ∈ ℕ} ⊂ C(K)$ überträgt sich auf die Grenzfunktion und Konvergenz in $(C(K),\norm\cdot_{\infty })$ ist „gleichmäßig auf $K$“. +Wegen dieser Eigenschaft ist die Maximumsnorm $\norm\cdot_\infty $ auch die natürliche Norm auf $C(K)$. +Andere mögliche Normen (und damit andere Topologien) auf $C(K)$ wären z.B. +\[ + \norm{f}_p = \left( \int_K |f(t)|^p dt \right)^{1/p}, \quad 1 \le p < \infty . +\] +Allerdings ist die Konvergenz in dieser Topologie impliziert keine Stetigkeit für die Grenzfunktion. + + +\begin{beispiel-nn} + Sei $\Omega ⊂ ℝ^n$ offen und analog + \[ + C(\Omega) \coloneq \{ f: \Omega → \K, f \text { stetig }\}. + \] + Hier können Funktionen aber auch unbeschränkt sein. Also braucht $\sup |f|$ nicht mehr zu existieren. +\end{beispiel-nn} + +\begin{definition} + Es sei $(K_m)_{m ∈ ℕ}$ eine \emph{Ausschöpfung} von $\Omega$ mit kompakten Mengen $K_= ⊂ \Omega$, das heißt, es gelte + \[ + \begin{cases} + \Omega = \bigcup_{m ∈ ℕ} K_m, \quad K_m ⊂ K_{m+1}, \\ + K ⊂ \Omega \text { kompakt } \implies K ⊂ K_m \text { f ür ein } m ∈ ℕ + \end{cases} + \] +\end{definition} +Man nehme z.B. +\[ + K_m = \{ x ∈ \Omega ⊂ ℝ^n: \norm{x} \le m, \operatorname{dist}(x,∂\Omega) \ge 1/m\}, +\] +wobei $\operatorname{dist}(x,∂\Omega) \coloneq \inf\{ \norm{x-y}: y ∈ ∂\Omega\}$ und $∂M = \cl \Omega \setminus \Omega$. + +Dann ist $C(\Omega)$ mit der Metrik +\[ + d(f,0) = \sum_{m ∈ ℕ} 2^{-m} \frac{\norm{f}_{C(K_m)}}{1+\norm{f}_{C(K_m)}} +\] +ein Fréchetraum, also ein metrisierbarer linearer Raum nach \cref{satz-abzaehlbares-prod-seminormierter-raeume}, da +\[ + \norm{f}_{C(K_m)} = 0 ∀ m ∈ ℕ \implies f = 0 ∈ C(\Omega). +\] + +Es gilt in diesem Raum +\[ + d(f_i,f) \xrightarrow[i → \infty ]{} 0 \gdw + \norm{f_i-f}_{C(K_m)} \xrightarrow[i → \infty ]{} ∀m ∈ ℕ, +\] +was ja gerade gleichmäßige Konvergenz auf jeder Kompakten Menge $K ⊂ \Omega$ bedeutet. +Damit ist Stetigkeit der Folgenglieder $(f_i)_{i ∈ ℕ} ⊂ C(\Omega)$ impliziert Stetigkeit der Grenzfunktion $f ∈ C(\Omega)$, da Stetigkeit nur eine lokale Eigenschaft ist. + +Wir werden in der Übung sehen, dass $C(\Omega)$ mit dieser Metrik $d_{C(\Omega)}$ nicht normierbar ist. + +\begin{beispiel-nn}[Räume differenzierbarer Funktionen] + \begin{enumerate} + \item + Betrachte die Menge $C^\ell(K) = \{ f: K → ℝ, D^\alpha f$ existiert und ist stetig für$|\alpha | < \ell \}$ der $\ell$-mal stetig differenzierbaren Funtktionen auf einer kompakten Menge $K ⊂ ℝ^n$ mit $\ell ∈ ℕ_0$ + Dabei ist $\alpha = (\alpha _1,…,\alpha _n) ∈ ℕ_0^n$ ein Multiindex, $|\alpha | = \sum_{i=1}^n \alpha _i$ und + \[ + D^\alpha f = \frac{∂^{|\alpha |} f}{∂x_1^{\alpha _1}\cdots∂x_n^{\alpha _n}}. + \] + Dann wird $C^\ell(K)$ mit der Norm + \[ + \norm{f}_{C^\ell(K)} = \max_{|\alpha | \le l} \max_{x ∈ K} | D^\alpha f(x)| + \] + zu einem Banachraum. Die meisten Eigenschaften sind klar, die Vollständigkeit folgt unmittelbar aus der Vollständigkeit von $C(K)$ + Konvergenz in $C^\ell(K)$ bedeutet gerade gleichmäßige Konvergenz aller partiellen Ableitungen bis zur Ordnung $\ell$ auf ganz $K$. + \item + Sei $\Omega ⊂ ℝ^n$ offen und + $\C^\ell(\Omega) = \{ f: \Omega → ℝ, D^\alpha f$ existiert und ist stetig für$|\alpha | < \ell \}$ + der Raum der $\ell$-mal stetig differenzierbaren Funtktionen auf $\Omega$ mit $\ell ∈ ℕ_0$. + $C^\ell(\Omega)$ wird mit der Metrik + \[ + d(f,g) \coloneq \sum_{m ∈ ℕ} 2^{-m} \frac{p_{m,l}(f-g)}{1+p_{m,l}(f-g)}, \quad p_{m,l}(f) = \max_{|\alpha | \le \ell} \norm{D^\alpha f}_{C(K_m)}, + \] + wobei die $K_m$ Ausschöpfungen von $\Omega$ mit kompakten Mengen sind, zu einem Fréchetraum. + Konvergenz in $C^\ell(\Omega)$ bedeutet gerade gleichmäßige Konvergenz aller partiellen Ableitungen bis zur Ordnung $\ell$ auf jedem Kompaktum, das in $\Omega$ enthalten ist. + Auch dieser Raum ist nicht normierbar mit einem analogem Argument wie bei den stetigen Funktionen auf $\Omega$. + \item + Wir betrachten nun einige Unterräume von $\C^\ell(\Omega)$: + \begin{enumerate}[label=(\roman*)] + \item + $\C^\ell_B(\Omega) = \{ f: \Omega → ℝ, D^\alpha f$ existiert, ist beschränkt und ist stetig für$|\alpha | < \ell \}$ + wird zum normierten Raum mit + \[ + \norm{f}_{C^\ell_B(\Omega)} = \max_{|\alpha | \le l} \sup_{x ∈ \Omega} | D^\alpha f(x)| + \] + Zwar gilt $C^\ell_B(\Omega) ⊂ C^\ell(\Omega)$ (als Mengen), jedoch besitzt $C^\ell_B(\Omega)$ nicht die Relativtopologie von $\C^\ell(\Omega)$, wie wir in einer Übung sehen werden. + \begin{definition} + \begin{enumerate} + \item + Für $\Omega ⊂ ℝ^n$ offen und $f: \Omega → ℝ$ heißt + \[ + \supp f \coloneq \cl{ \{ x ∈ \Omega, f(x) \ne 0 \}} + \] + der \emph{Träger} oder \emph{Support} von $f$. + \item + Wir sagen für eine Menge $M ⊂ \Omega$ \emph{$M$ liegt kompakt in $\Omega$}, wenn $\cl M $ kompakt ist und $\cl M ⊂ \Omega$. Wir schreiben dafür $M ⊂⊂ \Omega$. + \end{enumerate} + \end{definition} + \item + $C_0^\ell(\Omega) = \{ f ∈ C^\ell(\Omega) : \supp f ⊂⊂ M \}$ + Funktionen mit $\supp f ⊂⊂ M $ haben Luft zum Rand von $\Omega$: + \[ + \operatorname{dist}(\supp(f), ∂\Omega) > 0, + \] + denn sowohl $\supp f$ als auch $∂\Omega$ sind abgeschlossen. + Es gibt verschiedene Möglichkeiten, Topologien für $C_0^\ell(\Omega)$ zu wählen: + \begin{enumerate} + \item + $C_0^\ell(\Omega) ⊂ C^\ell(M)$ mit Spurtopologie von der Metrik. + \item + $C_0^\ell(\Omega) ⊂ C_B^\ell(M)$ mit Spurtopologie von der Norm. + \end{enumerate} + Diese Topologien sind jedoch nicht identisch. + \end{enumerate} + \item + Sei $\Omega ⊂ ℝ^n$ offen und + $C^\infty (\Omega) = \{ f: \Omega → ℝ, D^\alpha f $ existiert und ist stetig für alle $\alpha ∈ ℕ_0^n \} = \bigcap_{\ell ∈ ℕ}C^\ell(\Omega)$. + Wir definieren die Topologie wieder über eine Metrik durch Seminormen + \[ + d(f,g) \coloneq \sum_{m ∈ ℕ} 2^{-m} \frac{p_{m}(f-g)}{1+p_{m}(f-g)}, \quad p_{m}(f) = \max_{|\alpha | \le m} \norm{D^\alpha f}_{C(K_m)}. + \] + Mit dieser Metrik wird $C^\ell(\Omega)$ zum Fréchetraum. + Konvergenz in $C^\infty (\Omega)$ bedeutet gerade gleichmäßige Konvergenz aller partiellen Ableitungen auf jedem Kompaktum, das in $\Omega$ enthalten ist. + Auch dieser Raum ist nicht normierbar mit einem analogem Argument wie bei den stetigen Funktionen auf $\Omega$. + \item + Sei $\Omega ⊂ ℝ^n$ offen und $C_0^\infty (\Omega) = \{ f ∈ C^\infty (\Omega) : \supp f ⊂⊂ M \}$ der \emph{Raum der Testfunktionen}. + Ein Beispiel für so eine Funktion ist + \[ + f(x) = + \begin{cases} + c \exp \left( - \frac{1}{{1-|x|^2}} \right), & |x| < 1 \\ + 0, & |x| \ge 1 + \end{cases}, + \] + wobei $\Omega = B^{\norm\cdot_\infty}_2(0)$, $|\cdot| = \norm\cdot_2$ und $c ∈ ℝ$ konstant. + Offensichtlich ist $C_0^\infty (\Omega) ⊂ C^\infty (\Omega)$. + Wenn man auf $C_0^\infty (\Omega)$ jedoch die Spurtopologie wählt, bekommt man später Probleme (bestimmte Funktionale auf $C_0^\infty (\Omega)$ sind nicht mehr stetig, wie wir in einer Übungsaufgabe sehen werden. + Man nennt Funktionale auf $C_0^\infty (\Omega)$ auch Distributionen). + Außerdem wäre der $C_0^\infty (\Omega)$ mit dieser Metrik nicht vollständig -- der Träger der Grenzfunktion muss nicht mehr beschränkt sein. + \begin{definition-nn} + Sei $M ⊂ X$ und $X$ ein linearer Raum. Dann heißt + \[ + \conv (M) \coloneq \{ x: ∃\alpha _i > 0, x_i ∈ M, i ∈ \{1,…,k\}: \sum_{i=1}^k \alpha _i = 1, \sum_{i=1}^k \alpha _i x_i = x \} + \] + die \emph{konvexe Hülle} von $M$. + \end{definition-nn} + Aus Gründen, die erst später zu verstehen sind, wählt man auf $C^\infty _0(\Omega)$ folgende lokalkonvxe Topologie: + Setze + \[ + p(\xi) \coloneq \sum_{k ∈ ℕ} 2^{-k} \frac{\norm \xi _{C^k(\Omega)}}{1 + \norm \xi _{C^k(\Omega)}}, \quad \xi ∈ C_0^\infty (\Omega) + \] + Sei $(D_j)_{j ∈ ℕ}$ eine Ausschöpfung von $\Omega$ mit offenen Mengen, also $D_j ⊂ D_{j+1}, D_j ⊂⊂ \Omega, \bigcup_{j ∈ ℕ} D_j = \Omega$. + Eine mögliche Wahl wäre beispielsweise $D_j = K_j^\circ$, wobei die $K_j$ wie oben sind. + Für $\epsilon = (\epsilon _j)_{j ∈ ℕ} ∈ ℝ^\infty , \epsilon _j > 0$ für alle $ℕ$ definieren wir eine Umgebungsbasis der $0 ∈ C_0^\infty (\Omega)$ durch alle Mengen + \[ + U_\epsilon \coloneq \conv \left[ \bigcup_{j ∈ ℕ} \{ \xi ∈ C^\infty _0 : p(\xi) < \epsilon _j \} \right] ⊂ C_0^\infty (\Omega). + \] + mit $\epsilon = (\epsilon _j)_{j ∈ ℕ} ∈ ℝ^\infty , \epsilon _j > 0$ und endliche Schnitte davon. Andere Umgebungen umgeben sich durch Translation. + Die so definierte Topologie nennen wir $\T_\D$ und den Raum $C_1^\infty (\Omega)$ auch $\D(\Omega)$. + Es stellt sich heraus, dass diese Topologie tatsächlich unabhängig von der gewählten Ausschöpfung ist. + Außerdem ist $(\D(\Omega),\T_\D)$ ein topologischer linearer Raum, das heißt, die Vektorraumoperationen sind stetig. + \begin{lemma}[Charakterisierung offener Mengen in $\D(\Omega)$] + Es gilt + \[ + O ∈ \T_\D \iff ∀ ξ ∈ O ∃ \epsilon =(e_j)_{j ∈ ℕ} ∈ ℝ^\infty , e_j > 0: e+U_\epsilon ⊂ O. + \] + Das heißt, die Topologie $\T_\D$ und die Topologie + \[ + \tilde T_\D = \{ O ⊂ C_0^\infty (\Omega): ∀ ξ ∈ O ∃ \epsilon = (\epsilon _j)_{j ∈ ℕ} ∈ ℝ^\infty , \epsilon _j > 0: \epsilon + U_\epsilon ⊂ O \} + \] + sind gleich. + \end{lemma} + \begin{proof} + Übung. + \end{proof} + \begin{korollar} + Die Mengen $U_\epsilon$ sind bereits eine Umgebungsbasis der Null. + Nach Definition sind sie aber auch Konvex, das heißt $(\D(\Omega),\T_\D)$ ist ein lokalkonvexer Hausdorff-Raum. + \end{korollar} + \begin{satz} + $ξ_m \xrightarrow[m → \infty ]{} 0 \gdw$ + \[ + \begin{cases} + (i), & \text{Es existiert $D$ offen mit $D ⊂⊂ \Omega$ und + $ξ_m ∈ C_0^\infty (D)$ für alle $m ∈ ℕ$} \\ + (ii), & \text{Für jedes $k ∈ ℕ$ gilt: + $\norm{ξ_m}_{C^k(\cl{\Omega})} \xrightarrow[m → \infty ]{} 0$} + \end{cases} + \] + \end{satz} + \begin{proof} + Zeige nur „$\Leftarrow$“. Sei dazu $(ξ_m)_{m ∈ ℕ}$ eine Folge mit (i) und (ii). + Wähle nun $D_j$ von oben mit $D ⊂ D_j$ ($j$ ist fest). + Sei nun $\epsilon =(\epsilon _i)_{i ∈ ℕ}, \epsilon _i > 0$ gegeben. Dann müssen wir zeigen, dass für alle $m > m_0$ schon $ξ_m ∈ U_\epsilon $ gilt. + Zunächst sind nach (i) $ξ_m ∈ C^\infty _0(D_j)$ . + Außerdem gilt + \[ + p(\xi_m) \le \underbrace{\sum_{k=1}^N 2^{-k} \frac{ \norm{\xi_m}_{C^k(\cl \Omega)} } {1+ \norm{\xi_m}_{C^k(\cl \Omega)} }}_{\text{wegen (i)} < \epsilon _j/2 \text{ für $m \ge m_0(\epsilon _j,N)$}} + \underbrace{\sum_{k=N+1} 2^{-k}}_{<\epsilon _j/2 \text{ für $n$ groß genug}} < \epsilon _j. + \] + \end{proof} + \end{enumerate} + \end{beispiel-nn} + \begin{beispiel-nn}[Lebesgue-integrierbare Funktionen] + Betrachten wir nun Lebesgue-integrierbare Funktionen. + Bereits eingeführt wurden die Räume $\L^p(\Omega)$ und $L^p(\Omega)$, $0 < p < \infty $, wobei $\Omega ⊂ ℝ^n$ offen. + Diese sind für $1 \le p < \infty $ normiert, und für $0 < p < 1$ quasi-normiert. + Für $p = \infty $ setzen wir + \[ + \L^\infty (\Omega) \coloneq \{ f: \Omega → ℝ ∪ \{ -\infty , \infty \}, f \text{ messbar und fast überall beschränkt} \}. + \] + Damit haben wir offenbar + \[ + C(\Omega) ∩ B(\Omega) ⊂ \L^\infty (\omega). + \] + Sei + \[ + \norm f _{\L^\infty (\Omega)} \coloneq \supess_{t ∈ \Omega} |f(t)| \coloneq \inf_{M ⊂ \Omega \text{ NM}} \sup_{t ∈ \Omega \setminus M} |f(t)|. + \] + Dann gilt für $f ∈ \L^\infty (\Omega)$ + \[ + \norm f = 0 \gdw f = 0 \text{ fast überall} + \] + Mit $N \coloneq \{ f ∈ \L^\infty (\Omega) : \norm f = 0 \}$ wird + \[ + L^\infty (\Omega) \coloneq \left( \L^\infty (\Omega)/N, \norm\cdot_{L^\infty (\Omega)} \right) + \] + zu einem normiertem Raum. +\end{beispiel-nn} + +\begin{bemerkung-nn} + \begin{enumerate} + \item + Es gilt die \emph{Hölder'sche Ungleichung}. Für $f ∈ L^p(\Omega)$, $g + ∈ L^q(\Omega)$,$\frac 1 p + \frac 1 q = 1$ ist + \[ + \norm{f g }_{L^1(\Omega)} \le \norm{f}_{L^p(\Omega)} \norm g _{L^q(\Omega)}. + \] + \item + Für $\Omega$ messbar und beschränkt gilt + \[ + L^∞(\Omega) ⊂ L^p(\Omega) ⊂ L^q(\Omega), \quad 0 < q < p \le ∞. + \] + \item + $C_0^∞(\Omega)$ ist nicht abgeschlossen in $(L^p(\Omega),\norm-_p)$. + Für $1 ≤ p < ∞$ gilt sogar + \[ + \cl{C_0^∞(\Omega)}^{\norm-_{L^p(\Omega)}} = L^p(\Omega), + \] + das heißt, $C_0^∞(\Omega)$ liegt dicht in $L^p(\Omega)$. + \item + Satz von Riesz-Fischer: $(L^p(\Omega),\norm-_{L^p(\Omega)})$ ist für $1 + ≤ p ≤ ∞$ ein Banach"=Raum. + \end{enumerate} +\end{bemerkung-nn} + +\begin{lemma} + $L^p(0,1)$ ist für $0 < p < 1$ nicht lokalkonvex. Tatsächlich sind + $\emptyset$ und $L^p(0,1)$ die einzigen offenen konvexen Mengen. +\end{lemma} + +\begin{beispiel}[Sobolev-Räume] + Wir kennen aus der Analysis bereits die partielle Integration: Für alle $f, h ∈ C^1(\cl \Omega)$, wobei $\Omega$ beschränkt und $∂M$ hinreichend glatt, gilt + \[ + ∫_\Omega f(t) \cdot \left[ \frac ∂ {∂t_i} h(t) \right] \dd t = ∫_{∂\Omega} f(t) h(t) \nu_i \dd S(t) - ∫_\Omega \left[ \frac ∂ {∂t_i} f(t) \right] h(t) \dd t, + \] + wobei $\nu = (\nu_1, …, \nu_n)^T$ die äußere Einheitsnormale ist. + + \begin{bemerkung-nn} + Ist $f$ oder $h ∈ \C_0^∞(\Omega)$, so verschwinden die Randterme. + \end{bemerkung-nn} + + \begin{definition}\label{03-definiton-schwache-ableitung} + Sei $f ∈ L^p(\Omega)$. Dann heißt $g ∈ L^p(\Omega)$ \emph{verallgemeinerte Ableitung} oder \emph{schwache Ableitung} von $f$ nach $t_i$ für ein $i ∈ \{1,…,n\}$, falls für alle $\phi ∈ C_0^∞(\Omega)$ gilt: + \[ + ∫_\Omega f(t) \frac {∂\phi}{∂t_i}(t) \dd t = - ∫_\Omega g(t) \phi(t) \dd t. + \] + \end{definition} + + \begin{lemma} + Verallgemeinerte Ableitungen sind eindeutig bestimmt. + \end{lemma} + + \begin{bemerkung-nn} + \begin{enumerate} + \item Wir schreiben dafür $d_{t_i} f \coloneq g$. + \item + Für beschränktes $\Omega$ und $f ∈ C^1(\cl \Omega)$ ist \cref{03-definiton-schwache-ableitung} mit der klassischen Ableitung $g := D_{t_i} f ∈ C^0(\cl \Omega) ⊂ L^p(\cl \Omega)$ erfüllt. + Also haben wir in dieser Situation + \[ + d_{t_i} f = D_{t_i} f. + \] + Also stimmen die klassische und die schwache Ableitung überein. + \item + Induktiv kann man höhere (schwache) Ableitungen definieren. + \end{enumerate} + \end{bemerkung-nn} + + \begin{definition} + Sei $k ∈ N_0, 1 ≤ p < ∞$. Dann ist der \emph{Sobolev"=Raum} $W^{k,p}(Ω) := \{ f ∈ L^p(Ω): f$ besitzt verallgemeinerte Ableitungen $d^αf ∈ L^p(Ω)$ für alle $α ∈ ℕ_0^n$ mit $0 ≤ |α| ≤ k \}$. + \end{definition} + + \begin{lemma}[Leibniz'sche Regel] + Sei $1 ≤ p < ∞$ und $Ω ⊂ ℝ^n$ offen. Dann gilt für alle $f ∈ W^{k,p}(Ω)$ und für alle $α ∈ ℕ_0^n$ mit $0 ≤ |α| ≤ k$: + \[ + ∀\phi ∈ C_0^∞(Ω): ∫_Ω d^α f(t) \phi(t) \dd t = (-1)^{|α|} ∫_Ω f(t) D^α \phi(t) \dd t + \] + \end{lemma} + + \begin{bemerkung-nn} + Ist umgekehrt $(f^α)_{0 ≤ |α| <= k} ⊂ L^p(Ω)$ eine Familie von Funktionen, für die + \[ + ∀\phi ∈ C_0^∞(Ω): ∫_Ω d^α f(t) \phi(t) \dd t = (-1)^{|α|} ∫_Ω f(t) D^α \phi(t) \dd t + \] + gilt, so ist $f^0 ∈ W^{k,p}(Ω)$ und $d^αf^0 = f^α$. + \end{bemerkung-nn} + + \begin{satz} + $W^{k,p}(Ω)$ ist mit der Norm + \[ + \norm{f}_{W^{k,p}(Ω)} := \left( \sum_{0 ≤ |α| ≤ k} \norm{d^α f}^p_{L^p(\Omega)}\right)^{1/p} + \] + ein Banachraum. + \end{satz} +\end{beispiel} + +Seien $f_n → f ∈ L^p(\Omega), h ∈ C_0^\infty (\Omega)$. +Dann +\[ + \lim_{n → \infty } ∫_\Omega f_n(t) h(t) \dd t = ∫_\Omega f(t) h(t) \dd t, +\] +denn +\begin{align*} + ∫_\Omega (f_n(t) - f(t)) h(t) \dd t &\le ∫_{\supp h} M |f_n(t) - f(t)| \dd t +\\ & \stackrel{\mathclap{\text{Hölder}}}{\le} \; M [ \supp(h)]^{1/q} +\norm{f_n-f}_{L^p(\Omega)} → 0. +\end{align*} + +\section{Beschränkte und kompakte Mengen in metrischen linearen Räumen} + +Wir wissen bereits nach dem Satz von Heine-Borel, dass eine Teilmenge $K ⊂ ℝ^n$ genau dann kompakt ist, wenn sie abgeschlossen und beschränkt ist. +Beschränktheit bedeutet hier Beschränktheit in einer (beliebigen, da alle äquivalent) Norm. + +Nun wollen wir so ein Konzept für Beschränktheit auch in allgemeinen metrischen (topologischen) linearen Räumen finden. + +\begin{problem-nn} + Die natürliche Übertragung $d(x,0) \le M$, $x ∈ B$ definiert \emph{keine} Beschränktheit. + Gründe dafür sind: + \begin{enumerate} + \item + In einigen metrischen Räumen gilt ohnehin $d(x,0) \le 1$ für alle $x ∈ X$. + \item + Ist $d$ eine Metrik auf $X$. Dann ist $\tilde d \coloneq \frac d {1+d} \le 1$ eine zu $d$ äquivalente Metrik auf $X$, wie wir in Topologie gesehen haben. + \end{enumerate} +\end{problem-nn} + +\begin{definition} + Sei $(X,\T)$ ein topologischer linearer Raum, $B ⊂ X$ heißt \emph{beschränkt}, falls zu jeder offenen Umgebung $U$ von $0 ∈ X$ ein $\alpha > 0$ existiert, so dass $B ⊂ \alpha U = \{\alpha u: u ∈ U\}$, das heißt jede Nullumgebung lässt sich so weit „aufblasen“, dass sie $B$ überdeckt. +\end{definition} + +\begin{bemerkung-nn} + Der Begriff „Beschränktheit“ hängt also von der Topologie ab. +\end{bemerkung-nn} + + +\begin{satz} + Sei $(X,d)$ ein metrischer linearer Raum, dessen Metrik gemäß \cref{satz-abzaehlbares-prod-seminormierter-raeume} von abzählbar vielen Seminormen $(p_n)_{n ∈ ℕ}$ induziert ist. + Dann ist eine Teilmenge $B ⊂ X$ genau dann beschränkt, wenn für jedes $k ∈ ℕ$ ein $M_k > 0$ existiert mit $p_k(x) \le M_k$ für alle $x ∈ B$. +\end{satz} +\begin{proof} + „⇒“: Sei $k ∈ ℕ$ gegeben. + Setze $r_k \coloneq \frac 1 {2^{k+1}}$ und $U \coloneq B_{r_k}(0)$. + Da $B$ beschränkt ist, gibt es $\alpha = \alpha _k > 0$, dass + \begin{align*} + & B ⊂ \alpha U = \alpha B_{r_k}(0) \\ + \gdw & \alpha ^{-1} B ⊂ B_{r_k} (0) \\ + \gdw &d(\alpha ^{-1} x, 0) < r_k ∀ x ∈ B + \end{align*} + Dann gilt schon $p_k(x) \le M_k \coloneq \alpha _k$ für alle $x ∈ B$, denn + \[ + \frac 1 {2^{k+1}} = r_k > d(\alpha _k^{-1} x, 0 + \ge 2^k \frac {p_k(\alpha _k^{-1}x)}{1+p_k(\alpha _k^{-1} x)} + = 2^{-k} \frac{\alpha _k^{-1} p_k(x)}{1+\alpha _k^{-1} p_k(x)}. + \] + Also mit $\eta \coloneq \alpha _k^{-1} p_k(x)$ gilt $\frac 1 2 > \frac \eta {1+\eta}$, also $\eta < 1$ oder $p_k(x) \le M_k$ für alle $x ∈ B$. + + „⇐“: + Sei also $p_k(x) \le M_k$ für alle $x ∈ B$ und $k ∈ ℕ$. + Wir müssen nun zeigen, dass es für jedes $r > 0$ ein $\alpha > 0$ gibt mit $B ⊂ \alpha B_r(0)$, also $\alpha ^{-1} B ⊂ B_r(0)$. + Sei also $r > 0$ gegeben. + Wähle nun $m_0 ∈ ℕ$ mit $\sum_{n=m_0+1}^\infty 2^{-n} < r/2$. + Wähle $\alpha > 0$ mit $\sum_{n=1}^{m_0} 2^{-n} \frac{\alpha ^{-1} M_k}{1+\alpha ^{-1} M_k} < r/2$. + Dann gilt für alle $x ∈ B$ + \begin{align*} + d(\alpha ^{-1} x, 0) &= + \sum_{n ∈ ℕ} 2^{-n} \frac{\alpha ^{-1} p_n(x)}{1+\alpha ^{-1} p_n(x)} \\ + &\le \sum_{n=1}^{m_0} 2^{-n} \frac{\alpha ^{-1} p_n(x)}{1+\alpha ^{-1} p_n(x)} + \sum_{n=m_0+1}^\infty 2^{-n} < r/2 + r/2 = r. + \end{align*} +\end{proof} + +\begin{korollar} + Sei $(X,\norm\cdot)$ ein normierter linearer Raum, + Dann ist eine Teilmenge $B ⊂ X$ genau dann beschränkt, wenn $M > 0$ existiert mit $\norm{x} \le M$ für alle $x ∈ B$. +\end{korollar} +\begin{proof} + Wähle $p_1(x) = \norm x$ und $p_k \equiv 0$ für $k \ge 2$ und verwende den vorherigen Satz. +\end{proof} + + +\begin{bemerkung} + Kugeln $B_r(0)$ in $(X,d)$, wobei $d$ wie in \cref{satz-abzaehlbares-prod-seminormierter-raeume}, sinid also immer unbeschränkt, + weil nichttriviale Unterräume $M_{n_0} ⊂ B_r(x)$ existieren. + Insbesondere ist dies gültig in den Räumen $\K^\infty , C(\Omega), C^\ell(\Omega)$ und $C^\infty (\Omega)$. +\end{bemerkung} + +\begin{definition} + Ein topologischer linearer Raum $(X,\T)$ heißt \emph{lokalbeschränkt}, falls $0 ∈ X$ eine beschränkte Umgebung besitzt. +\end{definition} + +\begin{bemerkung} + Normierte Räume sind lokalbeschränkt und lokalkonvex. Es gilt aber auch die Umkehrung: +\end{bemerkung} + +\begin{satz}[Kolmogorov] + Ein topologischer linearer Raum $(X, \T)$ ist genau dann normierbar, das heißt, die Topologie wird von einer Norm induziert, + wenn $(X,\T)$ lokalkonvex und lokalbeschränkt ist. +\end{satz} + +\begin{beispiel-nn} + Die Räume $\K^\infty , C(\Omega), C^\ell(\Omega)$ und $C^\infty (\Omega)$ sind nicht lokalbeschränkt, aber lokalkonvex. Somit sind sie auch nicht normierbar. + Auch $L^p(0,1)$ mit $0 < p < 1$ ist nicht lokalkonvex, aber lokalbeschränkt, also nicht normierbar. +\end{beispiel-nn} + +\begin{definition} + Sei $(X,\T)$ ein topologischer linearer Raum. + Eine Umgebung $U$ der Null heißt \emph{kreisförmig} oder \emph{balanced}, falls + \[ + t U ⊂ U, \quad |t| < 1 + \] +\end{definition} + +\begin{lemma} + Sei $(X,\T)$ ein topologischer linearer Raum. + Dann besitzt die Null eine Umgebungsbasis aus kreisförmigen Mengen. +\end{lemma} +\begin{proof} + Übung. +\end{proof} + +\begin{warnung-nn} + Metrikkugeln müssen im Allgemeinen nicht kreisförmig sein (obwohl die uns bekannten Kugeln dies sind). + Gegenbeispiel: $X = ℝ$, $d(x,y) \coloneq \left| ∫_x^y 1+\ind{ℝ_-}(s)\; ds \right|$. +\end{warnung-nn} + +\begin{lemma} + Sei $(X,\T)$ ein topologischer linearer Raum und $V ∈ \T$ eine Umgebung der 0. + Dann gilt + \[ + X = \bigcup_{n ∈ ℕ} n V. + \] +\end{lemma} +\begin{proof} + „$\supset$“: klar. + + „⊂“: Sei $x ∈ X$. Setze $β_n = 1/n, n ∈ ℕ$. Dann gilt + \[ + β_n x \xrightarrow[n → \infty ]{} 0, + \] + also $β_n ∈ V$ für $n \ge n_0$. Damit haben wir aber $x ∈ n_0 V$. +\end{proof} + +\begin{satz} + Sei $(X,\T)$ ein topologischer linearer $T_2$-Raum und $K ⊂ X$ kompakt. + Dann ist $K$ abgeschlossen und beschränkt. +\end{satz} +\begin{definition-nn} + Falls auch die Umkehrung gilt, dann besitzt $X$ die \emph{Heine"=Borel"=Eigenschaft}. +\end{definition-nn} +\begin{warnung-nn} + Im Allgemeinen ist die Umkehrung falsch. +\end{warnung-nn} +\begin{proof} + Nach einer Übungsaufgabe ist $K$ bereits abgeschlossen. + Also müssen wir nur zeigen, dass $K$ auch beschränkt ist. + Sei $V ∈ \T$ eine Nullumgebung. + Sei $W ⊂ \T$ eine kreisförmige Umgebung der $0$, die ganz in $V$ enthalten ist. + Da + \[ + K ⊂ X = \bigcup_{n ∈ ℕ} n W + \] + eine offene Überdeckung von $K$ ist, besitzt diese wegen $K$ kompakt eine endliche Teilüberdeckung + \[ + K ⊂ \bigcup_{i=1}^s n_i W \stackrel{(*)}= n_s W, \quad n_1 < n_2 < … < n_s, + \] + also folgt die Behauptung mit $\alpha = n_s$. $(*)$ gilt wegen der Kreisförmigkeit und $\left|\frac {n_i}{n_s}\right| \le 1$. +\end{proof} +\begin{bemerkung-nn} + Ohne die Hausdorff-Eigenschaft gilt dies nicht. Gegenbeispiel: $X = ℝ$ mit der Klumpentopologie. + Dann ist jede Teilmenge von $ℝ$ kompakt, aber nur $\emptyset$ und $ℝ$ sind abgeschlossen. +\end{bemerkung-nn} + +\begin{definition} + \begin{enumerate} + \item + In einem topologischen Raum $(X,\T)$ heißt eine Menge $A ⊂ X$ \emph{relativ kompakt}, falls $\cl A$ kompakt ist. + \item + In einem metrischen Raum $(X,d)$ heißt eine Menge $A ⊂ X$ \emph{präkompakt}, falls für jedes $\epsilon > 0$ die Menge $A$ von endlich vielen Bällen mit Radius $\epsilon $ überdeckt werden kann. + \end{enumerate} +\end{definition} + +\begin{satz} + Sei $(X,d)$ ein metrischer Raum, $\emptyset \ne A ⊂ X$. Dann sind äquivalent: + \begin{enumerate} + \item + $A$ ist kompakt. + \item + $A$ ist folgenkompakt. + \item + $(A,d|_{A×A})$ ist vollständig und $A$ präkompakt. + \end{enumerate} +\end{satz} +\begin{proof} + $(a) \iff (b)$ wurde bereits gezeigt. Wir zeigen nur $(b) ⇒ (c)$: + Sei $(x_n)_{n ∈ ℕ} ⊂ A$ eine Cauchy-Folge. Nach (b) besitzt $(x_n)_{n ∈ ℕ}$ einen Häufungspunkt $x^*$. + Da $(x_n)_{n ∈ ℕ}$ Cauchy-Folge ist, konvergiert $(x_n)_{n ∈ ℕ}$ schon gegen $x^*$. Damit ist $A$ vollständig. + + Angenommen, $A$ wäre nicht präkompakt. Dann gibt es $\epsilon > 0$, so dass $A$ keine endliche Überdeckung mit $\epsilon $-Kugeln besitzt. + Dadurch kann man eine Folge $(x_k)_{k ∈ K}$ definieren, mit $d(x_k,x_j) > \epsilon $ für $k \ne j$. + Dann besitzt $(x_k)_{k ∈ K}$ offensichtlich keine Cauchy-Teilfolge, also auch keinen Häufungspunkt. + Also $A$ präkompakt. +\end{proof} + +\begin{korollar} + Ist $(X,d)$ ein vollständiger metrischer Raum und $A ⊂ X$, dann ist $A$ genau dann präkompakt, wenn $A$ relativ kompakt ist. +\end{korollar} + + +\begin{satz}[Ascoli-Arzela] + Sei $S ⊂ ℝ^n$ kompakt und $C(S,ℝ^m)$ mit der Norm + \[ + \norm{f}_∞ \coloneq \max_{x ∈ S} |f(x)|_{ℝ^m} + \] + ausgestattet. Sei $A ⊂ C(S;ℝ^m)$. Dann sind äquivalent: + \begin{enumerate} + \item $A$ ist präkompakt. + \item + $A$ ist beschränkt und gleichgradig stetig, das heißt, + \[ + \sup_{f ∈ A} |f(x)-f(y)|_{ℝ^m} \xrightarrow[|x-y|→ 0]{} 0. + \] + \end{enumerate} +\end{satz} + +\begin{satz}[Fréchet, Kolmogorov] + Sei $1 ≤ p < ∞$. Dann ist $A ⊂ L^p(ℝ^n)$ genau dann präkompakt, wenn + \begin{enumerate}[label=(\roman*)] + \item + $A$ ist beschränkt. + \item + $A$ ist im Mittel gleichgradig stetig, das heißt + \[ + \sup_{f ∈ A} \norm{f(\cdot + h) - f}_{L^p(ℝ^n)} \xrightarrow[|h| → 0]{} 0. + \] + \item + \[ + \sup_{f ∈ A} \norm{f}_{L^p(ℝ^n \setminus B_R(0))} \xrightarrow[R → ∞]{} 0. + \] + \end{enumerate} +\end{satz} + +\begin{bemerkung-nn} + Der Satz gilt auch für Teilmengen $Ω$ von $ℝ^n$ mit den offensichtlichen Anpassungen. Ist $Ω$ beschränkt, so wird (iii) überflüssig. +\end{bemerkung-nn} + + +\section{Stetige lineare Operatoren} + +\begin{satz} + 3.6.4. +\end{satz} +Hier gilt $M = \inf \{ c \ge 0:$ mit $C$ gilt (5) $\}$. +\begin{proof} + $(1) \iff (2)$ schon gezeigt. + + $(3) \iff (4)$ klar durch die Charakterisierung von beschränkten Mengen in + normierten Räumen und Ausnutzung der Linearität. + + $(2) \Rightarrow (4)$. Sei $T$ stetig in $x^*$. Wähle $\epsilon > 0$, so dass $T(\cl B_\epsilon (x^*)) ⊂ B_1(T(x^*))$. + Dann gilt für alle $x ∈ \cl B _1 (0)$ + \[ + x^* + \epsilon x ∈ \cl B_\epsilon (x^*) + \] + und $T(x^*) + \epsilon T(x) = T(x^* + \epsilon x) ∈ B_1(T(x^*))$, das heißt $\epsilon T(x) ∈ B_1(0)$ oder $\norm{T(x)}_Y \le \frac 1 {\epsilon } =: M$ + + $(4) \Rightarrow (5)$. Für $x \ne 0$ gilt + \[ + \norm{T(x)} \le \norm x \norm{T\left( \frac x {\norm x} \right)} \le M \norm x, + \] + also gilt die Aussage mit $C \coloneq M$. + + $(5) \Rightarrow (1)$. Für $x, x_1 ∈ X$ gilt + \[ + \norm{T(x) - T(x_1)} = \norm {T(x-x_1)} \le C \norm x-x_1 \xrightarrow[x → x_1]{} 0. + \] + Damit ist $T$ stetig in $x_1$. +\end{proof} + +\begin{korollar} + Sei die Situation wie in 6.4 Ist $T$ zusätzlich bijektiv, so ist $T$ genau dann ein Homöomorphismus, wenn es Konstanten $m, M > 0$ gibt mit + \[ + m \norm x \le \norm {T(x)} \le M \norm {x} + \] + für alle $x ∈ X$ +\end{korollar} +\begin{beweis} + klar. +\end{beweis} + +\begin{warnung-nn} + $T$ linear, bijektiv und stetig impliziert selbst in normierten Räumen noch nicht, dass auch die Inverse Abbildung $T^{-1}$ auch stetig ist, wie wir in der Übung sehen werden. + Sind $X$ und $Y$ aber Banachräume, so gilt dies aber (Satz von der offenen Abbildung). +\end{warnung-nn} + +Nun zur Charakterisierung von Stetigkeit in metrischen linearen Räumen. + +\begin{satz} + Sei $T: X → Y$ linear, $X$, $Y$ lineare metrische Räume. + Dann ist $T$ genau dann stetig, wenn $T$ beschränkt ist. +\end{satz} + +In topologischen linearen Räumen gilt dies jedoch nciht. + +\begin{satz} + 3.6.7 +\end{satz} +\begin{proof} + Nur „$\Leftarrow$“: Nach 6.6 reicht es, Beschränktheit von $T$ zu zeigen, also dass, wenn $B ⊂ X$ beschränkt ist, auch $T(B) ⊂ Y$ beschränkt ist. + $B ⊂ X$ ist genau dann beschränkt, wenn für alle $k ∈ ℕ$ $C_k > 0$ existieren mit $p_k(x) \le C_k$ für alle $x ∈ B$. + Nach Voraussetzung ist dann aber auch für alle $x ∈ B$ + \[ + q_m(Tx) \le M_m(C_{n_1} + … + C_{n_k}) =: K_m, + \] + was nach 5.2 heißt, dass $T(B)$ beschränkt in $Y$ ist. +\end{proof} + +\begin{definition} + Seien $X, Y$ topologische lineare Räume. Dann bezeichnet $\L(X, Y) \coloneq \{ T: X → Y: T$ linear und stetig $\}$ den \emph{Raum der stetigen (beschränkten) Operatoren}. + Im Spezialfall $Y = \K$ sei $X' \coloneq \L(X, \K)$ der \emph{Raum der stetigen Funktionale} oder auch der \emph{Dualraum von $X$}. +\end{definition} +\begin{bemerkung-nn} + \begin{enumerate} + \item + $\L(X,Y)$ ist wieder ein linearer Raum. + \item + Metrische lineare Räume haben Dualräume, die im Allgemeinen nicht mehr metrisierbar sind. + \item + $X' = \{ 0\}$ ist möglich, wie wir in der Übung sehen werden + \item + Ist $X$ jedoch normierbar, so folgt aus den Hahn-Banach-Sätzen, dass $X'$ nichttrivial ist. + \item + Falls $X$ und $Y$ normierte Räume sind, dann wird $\L(X, Y)$ ebenfalls zu einem normierten Raum mit der Operatornorm + \begin{align*} + \norm T &\coloneq \norm T _{\L(X,Y)} \coloneq \sup \{\norm x _X \le 1\} \norm {Tx}_Y \\ &= \inf \{ C \ge 0: ∀x ∈ X: \norm {Tx} \le C \norm x \}. + \end{align*} + Das heißt, wir haben + \begin{equation} + \label{eq:61} + ∀x ∈ X: \norm {Tx}_Y \le \norm T \norm x _X + \end{equation} + Also haben wir + \[ + \norm{(T_1 + T_2)x} = \norm{T_1x + T_2x} \le \norm{T_1x} + \norm{T_2x} \le \left( \norm{T_1} + \norm{T_2} \right) \norm{x}, + \] + und somit $T_1 + T_2 ∈ \L(X,Y)$ und $\norm{T_1 + T_2} \le \norm{T_1} + \norm{T_2}$ nach \eqref{eq:61}. + \item + Auf $\L(ℝ^n,ℝ^m)$ ergeben sich die bekannten Matrixnormen. + \end{enumerate} +\end{bemerkung-nn} + +\begin{satz} + Seien $X, Y$ normierte Räume, $Y$ vollständig. Dann ist $\L(X,Y)$ ein Banachraum. + Insbesondere ist $X'$ immer ein Banachraum. + + Sei $Z$ ebenfalls ein normierter Raum. + Ist $T ∈ \L(X,Y)$, $S ∈ \L(Y,Z)$, so ist $ST ∈ \L(X,Z)$ und $\norm{ST}_{\L(X,Z)} \le \norm S \norm T$. +\end{satz} + +\begin{proof} + Es ist nur noch die Vollständigkeit zu zeigen. + Sei dazu $(T_n)_{n ∈ ℕ}$ eine Cauchy-Folge in $\L(X,Y)$. + Das heißt, für jedes $\epsilon > 0$ existiert ein $N_0$ mit $\norm {T_n - T_m} < \epsilon $ für $n, m > N_0$. + Also mit \eqref{eq:61} $\norm {T_n x - T_mx} \le \norm {T_n - T_m} \norm x < \epsilon \norm x$ für alle $x ∈ X$ und $n,m > N_0$. + Insbesondere ist $(T_nx)_{n ∈ ℕ}$ eine Cauchy-Folge in $Y$. Da $Y$ vollständig ist, besitzt diese Folge einen Grenzwert $y_x ∈ Y$. + Wir definieren eine Abbildung + \[ + T: X → Y, x ↦ y_x. + \] + Dann ist $T$ linear, weil alle $T_n$ linear sind. Also ist nur die Stetigkeit von $T$ und die Konvergenz von $(T_n)_{n ∈ ℕ}$ gegen $T$ zu zeigen. + Für die Stetigkeit bekommt man unter Verwendung der Dreicksunglechung direkt + \[ + \left| \norm {T_n} - \norm{T_m} \right| \le \norm {T_n - T_m} < \epsilon \quad ∀ n, m \ge N_0, + \] + also eine Cauchyfolge $\left( \norm{T_n} \right)_{n ∈ ℕ}$ in $ℝ$, die wegen der Vollständigkeit von $ℝ$ konvergent, also insbesondere auch beschränkt ist. + Damit gibt es $M > 0$ mit $\norm {T_n} \le M$ für alle $n ∈ ℕ$, also mit~\eqref{eq:61} + \[ + \norm{Tx} \xleftarrow[n → \infty ]{} \norm{T_nx } \le M \norm x, ∀ x ∈ X, + \] + also die stetigkeit von $T$. + Jetzt zur Konvergenz: + Für $\norm x \le$ 1 gilt + \[ + \norm {T_n x - T_m x } < \epsilon , \quad ∀n, m \ge N_0, + \] + also durch Grenzwertbildung $n → \infty $ + \[ + \norm {T_n x - T x } < \epsilon , \quad ∀n \ge N_0, + \] + und mit~\eqref{eq:61} + \[ + \norm {T_n -T} = \sup_{\norm x \le 1} \norm {T_n x - T_x} < \epsilon , \quad ∀ n \ge N_0, + \] + das heißt $T_n → T$ wie gewünscht. + + Für den Zusatz haben wir + \[ + \norm {S(Tx)} \le \norm S \norm {Tx} \le \norm S \norm T \norm x. + \] + Da das für alle $x ∈ X$ gilt, haben wir $\norm {ST} \le \norm S \norm T$. +\end{proof} + + +\begin{korollar} + Ist $X$ ein Banachraum, dann ist $\L(X) \coloneq \L(X,X)$ eine \emph{Banachalgebra}, das heißt ein vollständiger normierter Vektorraum mit einer Multiplikation, so dass für $T, S ∈ \L(X)$ gilt: + \[ + \norm {TS} \le \norm T \norm S. + \] +\end{korollar} + +\begin{bemerkung} + Ist $T ∈ \L(X,Y)$, so ist $\ker T$ als Urbild der abgeschlossenen Menge $\{ 0\}$ stets abgeschlossen in $X$. + Das Bild hingegen $R(T) \coloneq \im T$ ist im Allgemeinen jedoch nicht abgeschlossen. + Wann sind Elemente in $\L(X)$ invertierbar? +\end{bemerkung} + +\begin{satz} + Sei $X$ ein Banachraum und $T ∈ \L(X)$ mit $\limsup\limits_{m → \infty } \norm{T}^{1/m} < 1$. Dann ist $(\id - T)^{-1} ∈ \L(X)$ und es gilt + \[ + (\id-T)^{-1} = \lim_{m → \infty } \sum_{n = 0}^m T^n =: \sum_{n = 0}^\infty T^n ∈ \L(X). + \] + mit Konvergenz in $\L(X)$. +\end{satz} +\begin{proof} + Wähle $m_0$ und $\Theta < 1$ mit $\norm {T^n} < \Theta ^n$ für $n \ge m_0$. + Für $S_k = \sum_{n=0}^k T^n$ gilt dann für $m_0 \le k < l$ + \[ + \norm{ S_l - S_k} = \norm { \sum_{n=k+1}^l T^n} \le \sum_{n=k+1}^l \norm{ T^k} \le \sum_{n=k+1}^l \Theta ^n < \epsilon , \quad k, l \ge N_0. + \] + Damit ist $(S_k)_{k ∈ ℕ}$ eine Cauchy-Folge in $\L(X)$ und somit konvergent. + Sei $S$ der Grenzwert. Dann gilt für jedes $x ∈ X$ auch $S_k x \xrightarrow[k → \infty ]{\norm\cdot_{X}} Sx$, also damit ist für alle $x∈ X$ + \[ + (\id - T) Sx = \lim_{k → \infty } (\id -T) S_k x = \lim_{k → \infty } \sum_{n=0}^k (T^n -T^{n-1})x = \lim_{k→\infty } x - T^{k+1}x = x. + \] + Damit ist $(\id -T)S = \id$. Da sich analog $S(\id-T) = \id$ auch zeigen lässt, folgt die Behauptung. +\end{proof} + + +\begin{lemma} + 3.7.6 +\end{lemma} +\begin{bemerkung-nn} + Mit $\Theta = 1$ geht es nicht immer. Gegenbeispiel: Sei $X = C[0,1] ∩ \{ x(0) = + 0 \}$ und $M = \{ x ∈ X : g∫_0^1 x(t) dt = 0 \}$. + Dann ist $M$ ein abgeschlossener linearer Unterraum, weil $T: X → ℝ, ∫_0^1 \cdot$ stetig ist und somit $M = T^{-1}(\{0\})$ als Urbild einer abgeschlossenen Menge in $ℝ$ abgeschlossen ist. + Angenommen, ($\Theta =1$), es existierte ein $x_\Theta = x ∈ X$ mit $\norm x_1 = $ und $\norm {x-x_1} \ge 1 $ für alle $x ∈ M$. + Dann setze + \[ + c(y) \coloneq \frac{∫_0^1 x_1(t) dt}{∫_0^1 y(t) dt} ∈ ℝ + \] + für alle $y \not\in M$. Man beachte, dass dies wohldefiniert ist. + Dann ist $x_1 - c(y)y ∈ M$, also $1 \le \norm{ x_1 - c(y)y - x_1} = |c(y)|\norm y$. + Dann $∫_0^1 x_1 c(y)y\; dt = 0 $ oder $\frac {1}{|c(y)} \le \norm y $ oder $\left| ∫_0^1 y(t)\;dt \right| \le \left| ∫_0^1x_1(t)\;dt \right| \norm y$ für alle $y ∈ X \setminus M)$. + Wähle $y_n(t) = t^{1/n} ∈ X$, also $\norm {y_n} = 1$. + Es gilt $\left| ∫_0^1 y_n(t) dt \right| \le \left| ∫_0^1 x_1(t) dt \right| \le 1$ für alle $n ∈ ℕ$, also + $∫_0^1 x_1(t) dt = 1$ und $x_1(t) \le 1$, was aber bereits impliziert, dass $x_1$ identisch 1 ist. Damit ist $x_1 \not\in X$. +\end{bemerkung-nn} + +\begin{satz} + 7.7 +\end{satz} +\begin{proof} + „⇐“ war Korollar 7.4. + + „⇒“. Angenommen, $\dim X = \infty.$ Sei $S^1 \coloneq \{ x ∈ X: \norm x = 1\}$. + Da $S^1$ abgeschlossen und beschränkt ist, ist $S^1$ nach Annahme kompakt. + Wähle $x_1 ∈ S^1$ und $M_1 \coloneq \lspan \{ x_1 \} \subsetneq X$. + $M_1$ ist ein abgeschlossener Unterraum nach Korollar 7.5. + Nach Ries existiert ein $x_2 ∈ S_1$ mit $\norm {x_2-x_1} \ge \Theta \coloneq \frac 1 2 $. + Setze nun $M_2 \coloneq \lspan \{x_1,x_2\}$. + Da $M_2$ ein abgeschlossener Unterraum ist, existiert ein $x_3 ∈ S_1$ mit $\norm {x_3 - x} \ge \Theta $ für alle $x ∈ M_2$, also insbesondere $\norm {x_3-x_1} \ge \Theta = \frac 1 2$ und $\norm {x_3-x_2} \ge \Theta = \frac 1 2$. + Iterativ (da $\dim X = \infty $) existiert $x_n ∈ S_1$ mit $\norm {x_m - x_n} \ge \frac 1 2$ für $m \ge n$. + Somit haben wir eine Folge $(x_n)_{n ∈ ℕ}$ ohne Häufungspunkt in $S^1$ gefunden im Widerspruch zu $S^1$ kompakt. +\end{proof} + +Damit sind in unendlich-dimensionalen normierten Räumen weder die Sphären noch die abgeschlossenen Kugeln kompakt. + + +\begin{definition} + Ein topologischer linearer Raum $X$ heißt \emph{lokalkompakt}, wenn $0 ∈ X$ eine Umgebung $U$ besitzt, deren Abschluss kompakt ist. +\end{definition} + +\begin{korollar} + Sei $X$ normiert, $\dim X = \infty $. Dann ist $X$ nicht lokalkompakt. +\end{korollar} +\begin{proof} + Angenommen, dass doch. Dann gibt es $r > 0$, so dass $S_r = \{ x ∈ X : \norm x = r\} ⊂ \cl U$. + Da $\cl U$ nach Annahme kompakt ist und $S_r$ abgeschlossen, ist $S_r$ ebenfalls kompakt. Das ist ein Widerspruch. +\end{proof} + + +%%% Local Variables: +%%% mode: latex +%%% TeX-master: "funkana-ebook" +%%% End: diff --git a/ch04-unitaere-raeume.tex b/ch04-unitaere-raeume.tex new file mode 100644 index 0000000..1cd310f --- /dev/null +++ b/ch04-unitaere-raeume.tex @@ -0,0 +1,447 @@ +\chapter{Unitäre Räume und Hilberträume} +\section{Grundbegriffe} +Sei wieder $\K = \R$ oder $\K = ℂ$. + +\begin{definition} + Sei $X$ ein linearer Raum über $\K$. + Eine Abbildung $\langle \cdot, \cdot \rangle: X × X → \K$ heißt \emph{Skalarprodukt} auf $X$, falls gilt + \begin{enumerate}[label=(U\arabic*)] + \item + $\langle x, x \rangle > 0$ für alle $0 \ne x ∈ X$. + \item + $\langle x, y \rangle = \cl {\langle y, x \rangle}$ für alle $x, y ∈ X$. + \item + $\langle x, \alpha y + β z \rangle = \alpha \langle x, y \rangle + β \langle x,z \rangle$ für alle $\alpha , β ∈ \K$, $x,y,z ∈ X$. + \end{enumerate} + $(X,\langle -,- \rangle)$ heißt \emph{Skalarproduktraum}, \emph{unitärer Raum} oder \emph{Prähilbertraum}. +\end{definition} + +\begin{bemerkung-nn} + Offenbar ist $\langle -,- \rangle$ in der ersten Komponente konjugiert linear. +\end{bemerkung-nn} + +\begin{satz} + Sei $(X, \langle -,- \rangle)$ ein unitärer Raum. Dann gelten die folgenden Aussagen: + \begin{enumerate} + \item + Durch $\norm x \coloneq \sqrt{\langle x, x \rangle}$ wird eine Norm definiert. + Dadurch wird jeder unitäre Raum auf natürliche Art und Weise normiert und trägt dadurch die induzierte natürliche Topologie. + \item + $|\langle x,y \rangle| \le \norm x \norm y$ mit Gleichheit genau dann, wenn $x$ und $y$ linear abhängig (Cauchy-Schwarz-Ungleichung). + \item + $\norm {x+y}^2 + \norm{x-y}^2 = 2(\norm x^2 + \norm y^2)$ (Parallelogrammgleichung), + \item + Für $\K = ℝ$ gilt + \[ + \langle x,y \rangle = \frac 1 4 \left( \norm { x+y}^2 - \norm{x-y}^2 \right), + \] + für $\K = ℂ$ + \[ + \langle x, y \rangle = \frac 1 4 \left( \norm {x+y}^2 - \norm{x-y}^2 - i \norm{x+iy}^2 + i\norm{x-iy}^2 \right). + \] + \end{enumerate} +\end{satz} +\begin{proof} + \begin{enumerate} + \item + Einfaches Nachrechnen unter Verwendung von (b) + \item + Für $y = 0$ ist die Behauptung klar. Sei also $y \ne 0, \alpha ∈ ℂ$. + Dann + \[ + \langle x + \alpha y, x+\alpha y \rangle = \langle x, x \rangle + \cl \alpha \langle y, x \rangle + \alpha \langle x,y \rangle + |\alpha |^2 \langle y,y \rangle. + \] + Speziell für $\cl \alpha \coloneq - \frac{\langle x,y \rangle}{\langle y,y \rangle}$ ergibt sich + \[ + 0 \le \langle x + \alpha y, x+\alpha + \rangle = \langle x,x \rangle - \frac{|\langle x,y \rangle^2|}{\langle y,y \rangle} - \frac{|\langle x,y \rangle^2|}{\langle y,y \rangle} + \frac{|\langle x,y \rangle^2|}{\langle y,y \rangle} = \langle x,x \rangle - \frac{|\langle x,y \rangle^2|}{\langle y,y \rangle}. + \] + Durch Umstellen ergibt sich + \[ + \langle x, x \rangle \ge \frac{|\langle x,y \rangle|^2}{\langle y,y \rangle} \gdw |\langle x,y \rangle|^2 \le \norm x ^2 \norm y^2. + \] + Die CSU erhält man durch Wurzel ziehen. + Gleichheit gilt genau dann, wenn + \[ + \langle x+ \alpha y, x+\alpha y \rangle = 0 \gdw x + \alpha y = 0, + \] + also wenn $x$ und $y$ linear abhängig sind. + \item + Es gilt + \[ + \norm {x \pm y}^2 = \norm x^2 \pm 2\Re(\langle x,y \rangle) + \norm y ^2. + \] + Addieren dieser Gleichungen für $+$ und $-$ ergibt die Behauptung. + \item + Es gilt + \begin{align*} + \norm {x+y}^2 - \norm{x-y}^2 &= (\norm x^2 + 2 \Re \langle x,y\rangle + \norm y^2) - (\norm x^2 - 2 \Re \langle x,y \rangle + \norm y^2) \\ + & = 4 \Re \langle x,y \rangle. + \end{align*} + Analog haben wir + \[ + -i \norm{x+iy}^2 + i \norm{x-iy}^2 = … = 4i \Im \langle x,y \rangle, + \] + was die Behauptung impliziert. + \end{enumerate} +\end{proof} + +\begin{satz} + Sei $(X,\norm\cdot)$ ein normierter Raum, der die Parallelogrammgleichung erfüllt. + Dann definieren + \[ + \langle x,y \rangle = \frac 1 4 \left( \norm { x+y}^2 - \norm{x-y}^2 \right), + \] + und + \[ + \langle x, y \rangle = \frac 1 4 \left( \norm {x+y}^2 - \norm{x-y}^2 - i \norm{x+iy}^2 + i\norm{x-iy}^2 \right). + \] + Skalarprodukte auf $X$ (für $\K = ℝ$ bzw $ℂ$). +\end{satz} +\begin{proof} + Stupides nachrechnen (oder so ähnlich). +\end{proof} + +\begin{bemerkung} + \begin{enumerate} + \item + Die Paralellogrammgleichung ist somit charakteristisch für unitäre Räume. + \item + $(C(S),\norm\cdot_\infty )$ mit $S ⊂ ℝ^n$ kompakt erfüllt dies nicht. + \item + Die Abbildung $\langle \cdot,\cdot \rangle$ in unitären Räumen ist stetig in beiden Komponenten als unmittelbare Konsequenz aus der Stetigkeit der Norm. + \end{enumerate} +\end{bemerkung} + +\begin{definition} + Ein bezüglich der Norm $\norm \cdot \coloneq \sqrt{ \langle \cdot,\cdot \rangle}$ vollständiger unitärer Raum $(X,\langle \cdot,\cdot \rangle)$ heißt \emph{Hilbertraum}. +\end{definition} + + +Hier fehlt eine VL. + +\begin{korollar} + $\hat y$ erfüllt die Gleichung aus dem vorherigen Satz genau dann, wenn $(x- \hat y) \perp Y$ gilt. +\end{korollar} +\begin{proof} + „⇐“: + Sei also $\hat y ∈ Y$ mit $x-\hat y \perp Y$, also $x-\hat y \perp (\hat y - y)$ für $y ∈ Y$ beliebig. + Dann gilt mit Pythagoras + \[ + \norm{x-y}^2 = \norm{x-\hat y + \hat y - y}^2 = \norm{x-\hat y}^2 + \norm{\hat y - y}^2 \ge \norm{x-\hat y}^2, + \] + was die Behauptung impliziert. +\end{proof} +\begin{bemerkung-nn} + Damit gilt im Hilbertraum das Riesz'sche Lemma (3.7.6) mit $\Theta = 1$. + Setze dazu + $ x_{\Theta =1} \coloneq \frac{x-\hat y }{\norm{x-\hat y}} $ + für ein $x \notin Y$. Dann ist $\norm{x_\Theta } = 1$ und für alle $z ∈ Y$ +gilt $\norm {z-x_\Theta }^2 + 2 \Re \langle z,x_\Theta \rangle + \norm{x_\Theta +}^2 \ge 1 = \Theta $. +\end{bemerkung-nn} +\begin{satz} + Es sei $Y$ ein vollständiger Unterraum eines unitären Raums $X$. + Dann existiert zu jedem $x ∈ X$ eine eindeutige Zerlegung der Form + \[ + x= y + v + \] + mit $y ∈ Y$ und $v ∈ Y^\perp$, das heißt $X = Y \oplus Y^\perp$. +\end{satz} +\begin{proof} + Jedes $x ∈ X$ lässt sich als $x = \hat y + (x- \hat y)$ schreiben, wobei $\hat y$ wie im Vorherigen Satz ist. + Dann ist $\hat y ∈ Y$ und $(x-\hat y) ∈ Y^\perp$. + Für die Eindeutigkeit seien $x = y_1 + v_1 = y_2 + v_2$ zwei Darstellungen von $x$ mit $y_i ∈ Y, v_i ∈ Y^\perp, i=1,2$. + Dann $y_1 - y_2 = v_2 - v_1$, wobei die linke Seite in $Y$ ist und die rechte in $Y^\perp$, aber $Y ∩ Y^\perp = \{ 0\}$ nach einem vorherigen Resultat, also $y_1 = y_2$ und $v_1 = v_2$. +\end{proof} +\begin{bemerkung-nn} +Weil für jedes $x ∈ X$ das Element $y = \hat y(x) ∈ Y$ in dieser Darstellung eindeutig ist, lässt sich dadurch eine Abbildung $P: X → X, x ↦ y$ definieren. +Diese Abbildung ist eine Projektion, das heißt $P \circ P = P$. +Wir schreiben für $P$ auch $\Proj_Y : X → X$ mit Wertebereich $\im P = Y$ und $P|_Y = \id|_Y$. +\end{bemerkung-nn} +\begin{korollar} + Falls $M ⊂ X$ ein Unterraum des Hilbertraums $X$ ist, dann gilt + \[ + \cl M = (M^\perp)^\perp. + \] +\end{korollar} +\begin{proof} + „⊂“ wurde bereits in Definition 2.1 gezeigt. + + „$\supset$“: Falls $(M^\perp)\perp \ne \cl M$, dann existiert $x_0 ∈ (M^\perp)^\perp \setminus \cl M$. + Da $X$ ein Hilbertraum ist, ist $\cl M$ vollständig. + Nach dem Satz vom orthogonalen Komplement gibt es eine eindeutige orthogonale Zerlegung von $x_0 = \hat x_0 + h_0^\perp$ mit $\hat x_0 = \Proj_M(x_0) ∈ \cl M$ und $x_0^\perp ∈ (\cl M)^\perp$. + Da $x_0^\perp ∈ (\cl M)^\perp$, ist auch $x_0^\perp ∈ (M)^\perp$ und $x_0 ∈ (M^\perp)^\perp$, also insbesondere $\langle x_0, x_0^\perp \rangle = 0$. + Das bedeute mit Hilfe der Zerlegung + \[ + 0 = \langle x_0, x_0^\perp \rangle + = \langle \hat x_0 + x_0^\perp, x_0^\perp \rangle + = \langle \hat x_0, x_0 ^\perp \rangle + \langle x_0^\perp, x_0^\perp \rangle + = \langle x_0^\perp, x_0^\perp \rangle + = \norm{x_0^\perp}^2. + \] + somit ist bereits $x_0^\perp = 0$, also $x_0 = \hat x_0 ∈ \cl M$. + Damit ist $\cl M = (M^\perp)^\perp$. +\end{proof} +\begin{bemerkung-nn} +Die Abbildung $P$ ist beschränkt mit Operatornorm $\norm P = \sup\limits_{x \ne 0} \frac{\norm{P(x)}}{\norm x} \le 1$, +denn für jedes $x = y + v$ mit $y ∈ Y, v ∈ Y^\perp$ gilt +\[ + \norm{P(x)}^2 = \norm{y^2} \le \norm y^2 + 2 \Re \langle y, v \rangle + \norm{v}^2 = \norm{y +v}^2 = \norm{x}^2. +\] +Desweiteren ist $P$ symmetrisch, das heißt für alle $x_1, x_2 ∈ X $ ist +\[ + \langle P(x_1), x_2 \rangle = \langle x_1, P(x_2) \rangle. +\] +Ist $x_1 = y_1 + v_2$, $x_2 = y_2 + v_2$ mit $y_i ∈ Y, v_i ∈ Y^\perp, i=1,2$, dann ist +\[ + \langle P(x_1), x_2 \rangle = \langle y_1,x_2 \rangle = \langle y_1,y_2 + v_2 \rangle + = \langle y_1,y_2 \rangle = \langle y_1+v_1, y_2 \rangle = \langle x_1, P(x_2) \rangle. +\] +\end{bemerkung-nn} +\begin{korollar} + Es Sei $Y \ne \{0\}$ ein vollständiger Unterraum des unitären Raums $X$ mit der Projektion $P = \Proj_Y: X → Y ⊂ X$. Dann gilt + \begin{enumerate} + \item $x-P(x) \perp Y $ für alle $x ∈ X$. + \item + $P$ ist symmetrisch. + \item + $P$ ist beschränkt mit Operatornorm $\norm P = 1$. + \end{enumerate} +\end{korollar} +\begin{proof} + (1) und (2) wurden bereits gezeigt. Bei (3) fehlt nur noch „$\ge$“. + Da $P_Y = \id|_Y$ und $Y \ne \{0\}$ ist das aber ebenfalls klar. +\end{proof} +Zentral in der Hilbertraumtheorie ist der Begriff der Hilbertraumbasis. +\begin{definition} + Ein Orthonormalsystem $(\hat e_k)_{k ∈ ℕ}$ eines unitären Raums $X$ heißt eine Orthonormalbasis oder eine \emph{Hilbertraumbasis}, falls eine der folgenden äquivalenten Bedingungen erfüllt ist: + \begin{enumerate} + \item Für alle $x ∈ X$ gilt die Vollständigkeitsrelation + \[ + \lim_{n → \infty } \norm{x - \sum_{k=1}^n \langle \hat e_k, x \rangle \hat e_k} = 0 + \] + \item + Für alle $x, y ∈ X$ ist + \[ + \langle x,y \rangle = \sum_{k=1}^\infty \cl{\langle \hat e_k. x \rangle} \langle \hat e_k, y \rangle. + \] + \item + Für alle $x ∈ X$ gilt die Parseval-Gleichung + \[ + \norm{x}^2 = \sum_{k=1}^\infty \left| \langle \hat e_k, x \rangle \right|^2. + \] + \end{enumerate} +\end{definition} +\begin{proof} + Übung. +\end{proof} +\begin{bemerkung-nn} + \begin{enumerate} + \item Statt (a) kann man auch + \[ + x = \lim_{n → \infty } \sum_{k=1}^n \langle \hat e_k, x \rangle \hat e_k + = \sum_{k=1}^\infty \langle \hat e_k,x \rangle \hat e_k + \] + schreiben. Dies nennt man die Fourier-Reihe von $x$. + \item + Die approximierenden Elemente + \[ + \sum_{k=1}^n \langle \hat e_k, x \rangle \hat e_k + \] + liegen offenbar in $\lspan S$ wenn $S = \{ \hat e_k : k ∈ ℕ \}$ ist, + was sich nicht notwendigerweise auf den Grenzwert überträgt. + Falls $X$ aber vollständig ist (also ein Hilbertraum), so sind diese Aussagen äquivalent zu $\cl{\lspan S}^{\norm\cdot} = X$. + \end{enumerate} +\end{bemerkung-nn} +\begin{satz} + \begin{enumerate} + \item + Für einen unitärer Raum $X$ gilt: Jede Hilbertraumbasis ist auch ein vollständiges Orthonormalensystem. + + \item + Ist zusätzlich $X$ ein Hilbertraum und $(\hat e_k)_{k ∈ ℕ}$ ein vollständiges Orthonormalensystem, dann ist $(\hat e_k)_{k ∈ ℕ}$ auch eine Hilbertraumbasis. + \end{enumerate} +\end{satz} +\begin{proof} + \begin{enumerate} + \item + Sei $S$ wie oben. Sei $x ∈ X$ mit $x \perp S$. Nach (c) gilt dann + \[ + \sum_{k=1}^\infty \big| \underbrace{\langle \hat e_k^\infty , x \rangle}_{=0} \big|^2 = \norm x ^2, + \] + also $\norm x = 0$ und $x = 0$. + \item + Sei nun $S$ ein abzählbares vollständiges Orthonormalensystem und $X$ ein Hilbertraum. + Führe den Beweis indirekt. + Angenommen, $S$ wäre keine Hilbertraumbasis. + Dann gelten die Eigenschaften (a)-(c) aus der Definition nicht und wegen der obigen Bemerkung ist dann $Y \coloneq \cl{\lspan S} \subsetneq X$. + $Y$ ist also ein abgeschlossener Unterraum von $X$, und da $X$ Hilbertraum ist, damit vollständig. + Nach Satz 2.9 ist $X = Y \oplus Y^\perp$. + Insbesondere ist also $Y^\perp \ne \{ 0\}$. + Damit gibt es ein $x ∈ X \setminus \{ 0\}$ mit $x ∈ Y^\perp$, also + \[ + \langle \hat e_k, x \rangle = 0 + \] + für alle $k ∈  ℕ$ im Widerspruch zur Vollständigkeit von $S$. + \end{enumerate} +\end{proof} +\begin{frage-nn} + Hat jeder Hilbertraum $H$ mit $\dim H = \infty $ ein abzählbares vollständiges ONS (also eine Hilbertbasis)? +\end{frage-nn} +Die Antwort darauf ist nein, aber falls $H$ zusätzlich separabel ist, dann ist sie ja. +Dagegen ist die Existenz eines vollständigen Orthonormalensystems (also eventuell überabzählbar, also keine ONB) kein Problem: +\begin{satz} + In jedem Hilbertraum $X \ne \{ 0\}$ gibt es ein vollständiges Orthonormalensystem. + Es lässt sich sogar jedes ONS $S_0$ zu einem vollständigen Orthonormalensystem $\tilde S_0$ mit $S_0 ⊂ \tilde S_0$ ergänzen. +\end{satz} +\begin{proof} + Simple Anwendung von Zorns Lemma. +\end{proof} +\begin{beispiel} + \begin{enumerate} + \item + Sei $X = L^2(0,2\pi), \K = ℝ$. + Dann ist ein VONS in $X$ gegeben durch + \[ + S = \left\{ \frac 1 {\sqrt{2\pi }}\right\} + ∪ \left\{ \frac 1 {\sqrt{\pi }} \cos(nx) : n ∈ ℕ\right\} + ∪ \left\{ \frac 1 {\sqrt{\pi }} \sin(nx) : n ∈ ℕ\right\}. + \] + In der klassischen Fourieranalysis werden Entwicklungen nach diesem VONS $S$ untersucht. + Man zeigt dort, dass $\lspan S$ bezüglich $\norm\cdot_\infty $ dicht liegt in $C_{\text{per}}([0,2\pi]) = \{ f: \R → \R: f$ ist stetig und $2\pi $-periodisch $\}$. + Die Aussage von 2.13(2) und (2.10) liefert nur die Begründung +für die Dichtheit von $\lspan S$ in $\norm-_{L^2}$. + \item + Durch $(f,g)_\mu \coloneq ∫_a^b \mu (t) f(t) g(t)\; dt $, wobei $\mu > 0$ und stetig auf $(a,b)$, ist auf $L^2(a,b)$ ein reelles Skalarprodukt definiert. + Für verschiedene Gewichtsfunktionen $\mu $ und verschiedene Wahlen von $(a,b)$ erhält man $\mu $-orthogonale Polynomsysteme durch Anwendung des Gram-Schmidt-Verfahrens auf die Monome $\{t^i: i ∈ ℕ_0\}$. + \begin{enumerate}[label=(\roman*)] + \item + $a=-1, b=1$, $\mu (t) = 1$ liefert die Legendre-Polynome. + \item + $a=-1, b=1$, $\mu (t) = \frac 1 {\sqrt{1-t^2}}$ liefert die Tschebyscheff-Polynome. + \item + $a=0, b=\infty $, $\mu (t) = \exp(-t)$ liefert die Laguerre-Polynome. + \item + $a=-\infty , b=\infty $, $\mu (t) = \exp(-t^2)$ liefert die Hermite-Polynome. + \end{enumerate} + \item + Ist $X$ ein unitärer Raum mit ONB, kann er formal vervollständigt werden: + Sei also $(\hat e_k)_{k ∈ ℕ} ⊂ X$ diese ONB, dann ist + \[ + H \coloneq \left\{ \sum_{k=1}^\infty c_k \hat e_k: (c_k)_{k ∈ ℕ} ∈ \ell^2 \right\} + \] + ist ein Hilbertraum, den man die Vervollständigung von $X$ nennt. + Das Skalarprodukt zwischen $x = \sum_{k ∈ ℕ} c_k \hat e_k$ und $y = \sum_{k ∈ ℕ} d_k \hat e_k$ + wird definiert als + \[ + \langle x,y \rangle \coloneq \sum_{k=1}^\infty \cl{c_k} d_k. + \] + Tatsächlich kann $H$ mit dem Koordinatenraum $\ell^2 = \ell^2(ℕ)$ identifiert werden. + Die Abbildung + \[ + \Phi: \ell^2(ℕ) → H, (c_k)_{k ∈ℕ} ↦ \sum_{k=1}^\infty c_k \hat e_k + \] + ist linear, bijektiv und normerhaltend wegen der Parsevalgleichung + \[ + \norm{x}^2 = \sum_{k=1}^\infty \left| \langle \hat e_k, x \rangle \right|^2. + \] + Also $\ell^2(ℕ)$ und $H$ isometrisch und insbesondere $H$ vollständig. + \end{enumerate} +\end{beispiel} + + +% VL NÄCHSTE WOCHE + +Der Satz 4.1 liefert also, dass die Abbildung $J_x: X → X', y ↦ y'$ definiert +durch $y': X → \K, x ↦ \langle y,x \rangle$ bijektiv ist. +Wir schreiben nun +\[ +\lAngle J_x(y),x \rAngle = \lAngle J_x(y),x \rAngle_{X'×X} \coloneq J(x)(y)[x] += \langle y,x \rangle. +\] +Diese Abbildung ist sesquiliniear, das heißt +\[ + J_x (y_1 + y_2) = J_x (y_1) + J_x(y_2), \quad y_1, y_2 ∈ X, +\] +\[ + J_x(\alpha y) = \cl{\alpha} J_x(y), \quad \alpha ∈ \K, +\] +denn +\[ + \lAngle J_x(\alpha y),x \rAngle = \langle \alpha y, x \rangle = \cl \alpha \langle y, x \rangle = \cl \alpha J_x(y) [x] = \cl \alpha \lAngle J_x(y), x \rAngle + \lAngle \cl \alpha J_x(y), x \rAngle, +\] +also $X \cong X'$ sesquilinear isomorph. + +Gilt da sauch topologisch? +Die Topologie von $X'$ sei hierbei die von $\L(X, \K)$, also die von der Norm $\norm{y'}_{X',N} = \sup_{\norm{x} \le 1}|y'[x]|$ erzeugte. +\begin{satz} + $X$ und $X'$ sind Hilberträume und $J_x: X → X'$ ist kanonischer sesquilinearer Isomorphismus, der die Norm erhält, also eine Isometrie. + + Genauer gilt: + \begin{enumerate} + \item + $\langle y_1', y_2' \rangle_{X'} \coloneq \cl{ \langle y_1, y_2 \rangle_X}$, wobei $J_x(y_1) = y_1', J_x(y_2) = y_2'$, macht $X'$ zum Skalarproduktraum. + \item + Die durch $\langle -,- \rangle_{X'}$ induzierte Norm + \[ + \norm{y'}_{X',S} = \sqrt{\langle y', y' \rangle_{X'}} + \] + ist gerade die von $X' = \L(X, \K)$ bekannte, das heißt, $\norm{y'}_{X',S} = \norm{y'}_{X',N}$. + \item + Da $(X',\norm-_{X',N})$ schon bekanntlich vollständndig ist, ist $(X', \langle -,- \rangle)$ damit ein Hilbertraum. + \item + $J_x: X → X'$ ist eine Isometrie. + \end{enumerate} +\end{satz} +\begin{proof} + \begin{enumerate} + \item + Beispielsweise ist + \[ + \langle \alpha y_1' , y_2' \rangle_{X'} \stackrel{\text{def}}{=} \cl{\langle \cl \alpha y_1, y_2 \rangle_X} = \cl{ \alpha \langle y_1, y_2 \rangle_X} = \cl{\alpha} \langle y_1',y_2' \rangle_{X'}, + \] + die anderen Eigenschaften folgen analog. + \item + Wegen $y'[x] = \langle y,x \rangle$ und $\norm{y'}_{X',S} = \sqrt{\langle y', y' \rangle_{X'}} = \sqrt{\langle y, y \rangle_X} = \norm{y}$, das heißt, es genügt, zu zeigen, dass + \[ + \norm{y'}_{X',N} = \sup_{\norm x \le 1} |y'[x]| = \norm{y}_{X} \quad \text{für alle $y ∈ X$}. + \] + hierbei ist aber „$\le$“ gerade die Cauchy"=Schwarzsche Ungleichung, für „$\ge$“ wähle $x = \frac y {\norm y _{X}}$ für $y \ne 0$ ($y=0$ ist sowieso klar). + \item + nichts zu zeigen. + \item + $J_x: X → X'$ ist eine Isometrie, denn $y ↦ J_x(y) = y'$ und $\norm{J_X(y)}_X = \norm{y'}_{X'} = \norm{y}_X$ für alle $y ∈ X$. + \end{enumerate} +\end{proof} + +\section{Separable Hilberträume} +\begin{definition} + Ein metrischer Raum $(X,d)$ heißt \emph{separabel}, wenn es $U ⊂ X$ dicht + und abzählbar gibt. +\end{definition} + +\begin{beispiele} + $ℝ^n, ℂ^n, \ell^2, L^2(\Omega)$ für $\Omega ⊂ ℝ^n$ offen sind separable Hilberträume. +\end{beispiele} + +\begin{satz} + In einem separablen unendlich"=dimensionalen Hilbertraum $(X,\langle -,- \rangle)$ gilt + \begin{enumerate} + \item Jedes ONS in $X$ ist höchstens abzählbar. + \item + Sei $S = (\hat e_i)_{i ∈ ℕ}$ ein VONS in $X$. Dann existiert zu jeder + Folge $\alpha = (\alpha _k)_{k ∈ ℕ} ∈ \ell^2$ genau ein $x ∈ X$ mit $\langle \hat + e_k, x \rangle = \alpha _k, k ∈ ℕ$ (Satz von \emph{Riesz-Fischer}). + \item + $X$ ist isometrisch isomorph zum $\ell^2$. Insbesondere sind + $L^2(\Omega)$ und $\ell^2$ isometrisch isomorph. + \end{enumerate} +\end{satz} + + +\section{Riesz'scher Darstellungssatz und Lax-Milgram} +Für einen topologischen linearen Raum $X$ ist der Dualraum $X' = \{x': X → \K, x' $ linear und stetig $\}$ definiert. +Im Allgemeinen kann auch $X' = \{0\}$ gelten. +Ist $X$ jedoch ein Hilbertraum, so ist stets $X' \ne \{0\}$, denn zu $y ∈ X$ ist durch $y'[x] \coloneq \langle y,x \rangle, x ∈ X$ jeweils ein $y' ∈ X'$ erklärt. +Tatsächlich bekommt man dadurch sogar schon alle Elemente des Dualraums: \ No newline at end of file diff --git a/ch05-hahn-banach.tex b/ch05-hahn-banach.tex new file mode 100644 index 0000000..ce92a84 --- /dev/null +++ b/ch05-hahn-banach.tex @@ -0,0 +1,370 @@ +\chapter{Der Satz von Hahn-Banach und seine Konsequenzen} +\section{Fortsetzbarkeit linearer Funktionale} + +Wir fragen uns, ob sich Abbildungen so erweitern, dass gewisse Eigenschaften (wie z.B. Linearität oder Stetigkeit) erhalten bleiben. + +\begin{definition} + Eine Abbildung $A: M → Y$ heißt eine Fortsetzung einer Abbildung $A_0: M_0 → X$, falls + \begin{enumerate} + \item $ M_0 ⊂ M$, + \item $∀x ∈ M_0: A_0 x = Ax $. + \end{enumerate} + Wir schreiben dann $A = A|_{M_0}$. +\end{definition} + +\begin{satz} + Seien $(X,\norm-)$ und $(X_0,\norm-)$ normietre Räume, $X_0 ⊂ X$ dicht in $X$. + Weiter sei $(Y, \norm-_{Y})$ ein Banachraum und $A_0 : X_0 → Y$ stetig und linear. + Dann gibt es genau eine stetige lineare Fortsetzung $A : X → Y$ von $A_0$ auf $X$. + Für diese gilt: + \[ + \norm{A_0}_{\L(X_,Y)} = \norm{A}_{\L(X,Y)}. + \] +\end{satz} +\begin{proof} + Zeigen wir zunächst die Existenz der Fortsetzung. + Da $X_0$ dicht in $X$ ist, existiert zu jedem $x ∈ X$ eine Folge $(x_n)_{n \ge1}$, die ganz in $X_0$ liegt und gegen $x$ konvergiert. + Wir behaupten, dass $(A_0x_n)_{n ∈ ℕ}$ eine Cauchy-Folge in $Y$ ist. + Dazu beachte, dass + \[ + \norm{A_0 x_n - A_0 x_m}_{Y} \le \norm{A_0}_{\L(X_0,Y)} \norm{x_n-x_m} \xrightarrow[n,m → \infty ]{} 0. + \] + Da $Y$ ein Banachraum ist, ist $(A_0x_n)_{n\ge1}$ konvergiert, etwa gegen $y$. + Wir setzen $Ax \coloneq y$. + Zunächst ist $A$ wohldefiniert, denn wenn $(z_n)_{n \ge 1}$ eine weitere Folge mit $\lim_{n → \infty } z_n = x$ ist, dann gilt + $z_n - x_n \xrightarrow[n→\infty ]{} 0$ und + \begin{align*} + \norm{A_0 z_n - y} &\le \norm{A_0 z_n - A_0 x_n} + \norm{A_0 x_n - y} \\ + & \le + \norm{A_0} \norm{z_n - x_n} + \norm{A_0 x_n - y} \xrightarrow[n→\infty ]{} 0. + \end{align*} + Offensichtlich ist $A$ eine Fortsetzung von $A_0$. + Dass $A$ linear ist, ist ebenfalls klar. + Zur Stetigkeit ist + \begin{align*} + \norm{Ax}_Y &= \norm{\lim_{n → \infty } A_0 x_n}_Y = \lim_{n → \infty } \norm{A_0 x_n}_{Y} \\ + &\le + \lim_{n → \infty } \norm{A_0}_{\L(X_0,Y)} \norm{x_n}_X = \norm{A_0} \norm{x}. + \end{align*} + Damit ist $A$ beschränkt, also auch stetig. + + Es gilt $\norm{A_0}_{\L(X_0,Y)} = \norm{A}_{\L(X,Y)}$: + „$\ge$“ ist aus dem Vorherigen klar. Für die andere Ungleichung ist + \[ + \norm{A}_{L(X,Y)} = + \sup_{\norm{x \le 1}, x ∈ X} \norm{Ax}_{Y} + \ge + \sup_{\norm{x \le 1}, x ∈ X_0} \norm{Ax}_{Y} = \norm{A_0}_{\L(X_0,Y)}. + \] + + Für die Eindeutigkeit sei $B: X → Y$ eine weitere stetige, lineare Fortsetzung von $A_0$. + Wie oben existiert zu jedem $x ∈ X$ eine Folge $(x_n)_{n ∈ ℕ} ⊂ X$ mit $\lim_{n → \infty } x_n = x$. + Dann ist + \[ + Ax_n = A_0 x_n = Bx_n \quad ∀ n ∈ ℕ + \] + und für $x ∈ X$ + \[ + \norm{B_x - A_x} \le \norm{B_x - Bx_n} + \norm{Bx_n - Ax_n} + \norm{Ax_n - Ax} \xrightarrow[n→\infty ]{} 0, + \] + da $A$ und $B$ stetig sind. Also $Bx = Ax$ für alle $x ∈ X$ und damit $B = A$. +\end{proof} + +\begin{korollar} + Ist $A ∈ \L(X,Y)$, $X, Y$ normiert sowie $Y$ vollständig und $M ⊂ X$ dicht, dann gilt: + Falls $Ax = 0$ für alle $x ∈ M$, dann ist $A$ schon die Nullabbildung auf $X$. +\end{korollar} +\begin{proof} + ~ +\end{proof} + +Ist $X_0$ nicht dicht in $X$, wird die Fortsetzung schwieriger. + + +\begin{satz} + Auf dem linearen Raum $X$ über $ℝ$ gebe es eine Abbildung $p: X → ℝ$ mit: + \begin{enumerate}[label=(\roman*)] + \item + $p(\alpha x) = \alpha p(x)$ für alle $\alpha \ge 0, x ∈ X$ (positiv homogen) + \item + $p(x+y) \le p(x) + p(y)$ für alle $x, y ∈ X$ (subadditiv) + \end{enumerate} + + Weiter seine $X_0$ ein linearer Teilraum von $X$ und $f_0 : X_0 → ℝ$ eine lineare Abbildung mit + \[ + ∀x ∈ X_0 : f_0(x) \le p(x). + \] + Dann gibt es eine lineare Fortsetzung $f: X → ℝ$ von $f_0$, welche die Ungleichung respektiert, das heißt + \[ + f|_{X_0} = f_0 \quad \text{und} \quad ∀x ∈ X: f(x) \le p(x). + \] +\end{satz} +\begin{bemerkung-nn} + Halbnormen oder Normen $p$ Erfüllen die Voraussetzungen dieses Satzes. +\end{bemerkung-nn} +\begin{proof} + Schritt 1. + Wir setzen $f_0$ auf $X_1 \coloneq X_0 \oplus \lspan{x_1}$ für ein $x_1 \not\in X$ (existiert immer solange $X_0 \subsetneqq X$). + Offenbar hat jedes $x ∈X_1$ eine eindeutig Darstellung als + $ y = y + \alpha x_1 $, mit $y ∈ X_0$, $\alpha ∈ ℝ$. + Dann ist mit $c ∈ ℝ$ beliebig + \[ + f(x) = f(y + \alpha (x_1)) \coloneq f_0(y) + \alpha c + \] + eine lineare Abbildung $X_1 → ℝ$, die $f_0$ fortsetzt. + Wir müssen $c$ so wählen, dass $f(x) \le p(x)$ für alle $x ∈ X_1$, also $f_0(y) + \alpha c \le p(y+\alpha x_1)$ für alle $y ∈ X_0, \alpha ∈ ℝ$. + Mit (i) ist diese Bedingung äquivalent zu zwei anderen Bedingungen: + \begin{enumerate} + \item + Für $a > 0$: $f_0(y/\alpha ) + c \le p(y/\alpha + x_1)$. + \item + Für $\alpha < 0$: $f_0(-y/\alpha ) - c \le p(-y/\alpha - x_1)$ + \end{enumerate} + für alle $y ∈ X_0$. Der Fall $\alpha = 0$ ist nach Annahme ohnehin klar. + Um diese Bedingungen erfüllen zu können, muss $c ∈ ℝ$ so gewählt werden, dass + \[ + ∀y_1, y_2 ∈ X_0: f_0(y_1) - p(y_1 - x_1) \le c \le p(y_2 + x_2) - f_0(y_2). + \] + Das ist möglich, da + \[ + f_0(y_1) + f_0(y_2) = f_0(y_1+y_2) \le p(y_1 + y_2) = p(y_1 - x_1 + y_2 + x_1) \le p(y_1 - x_1)+p(y_2+x_1). + \] + Folglich gilt + \[ + \sup_{y_1 ∈ X_0} f_0(y_1-p(y_1-x_1)) \le \inf{y_2 ∈ X_0} p(y_2+x_1)-f_0(y_2). + \] + + + Schritt 2. + Finde eine maximale Fortsetzung mit dem Lemma von Zorn. + Betrachte dazu + \[ + \{: X \supset D_g \supset X_0 → ℝ\}: g|_{X_0} = f_0 ∧ ∀x ∈ D_g: g(x) \le p(x) \}. + \] + Diese Menge ordnen wir mit $\succeq$ definiert durch + \[ + h \succeq g \gdw h \text{ ist Fortsetzung von $g$}. + \] + Nach dem Lemma von Zorn existiert eine maximale Fortsetzung $g^*$ von $f_0$ mit $g^*(x) \le p(x)$ für alle $x ∈ X$. + Wäre $D_{g^*}$ nicht $X$, so verfahre wie in Schritt 1 im Widerspruch zur Maximalität. + Damit hat $g^*$ die gewünschten Eigenschaften. +\end{proof} + +\begin{bemerkung-nn} + \begin{enumerate} + \item + Ohne die Zusatzforderung $f(x) \le p(x)$ für alle $x ∈X$ ist die lineare Fortsetzbarkeit trivial. + \item + Eine Fortsetzung für lineare Funktionale $f_0: X_0 → \K = ℂ$ ist analog möglich. % yos IV 4 + \end{enumerate} +\end{bemerkung-nn} + +%% HIER FEHLT EINE VORLESUNG + +\begin{satz}[5.3.1] + Sei $(X,\norm\cdot)$ ein normierter Raum über $ℝ$, $M ⊂ X$ abgeschlossen und konvex und $0 ∈ M$. + + Dann existiert zu jedem $x_0 \not\in M$ ein $f ∈ X'$ mit + \[ + f(x_0) > 1 ∧ ∀ x ∈ M: f(x) \le 1. + \] +\end{satz} +Die Hyperebene $H = \{ x ∈ X: f(x) = 1 + \epsilon \}$ für $0 < \epsilon < f(x_0) < 1$ trennt also $x_0$ und $M$. + +\begin{proof} + Setze $2r \coloneq \inf_{y ∈ M} \norm{y - x_0}$ (positiv, da $M$ abgeschlossen). + Sei $N \coloneq \cl{M + \cl{B_r(0)}} = \cl{\{ z = y + u: y ∈ M, u ∈ \cl{B_r(0)}\}} ⊂ X$. + Dann ist (i) $N$ abgeschlossen und (ii) $\cl{B_r(0)} ⊂ N$, da $0 ∈ M$, insbesondere ist $0 ∈ N^\circ$. + (iii) ist $N$ konvex: Es genügt, zu zeigen, dass $A = M + B_r(0)$ konvex ist, denn dann ist auch $\cl A$ konvex. + Sei $x _i = y_i + v_i, y_i ∈ M, v_i ∈ \cl{B_r(0)}, i=1,2$ und $\alpha ∈ (0,1)$. Dann ist + \[ + \alpha x_1 + (1-\alpha )x_2 = \underbrace{[\alpha y_1 + (1-\alpha )y_2]}_{∈ M} + \underbrace{[\alpha u_1+(1-\alpha )v_2]}_{∈ \cl{B_r(0)}}. + \] + (iv) ist $x_0 \not\in N$. + Angenommen, $x_0 ∈ N$. Dann existiert eine Folge $z_n = y_n + u_n$ in $A$ mit $z_n → x_0 (n→\infty )$. + Dann ist für $n_0$ hinreichend groß + \[ + \frac r 2 > \norm{z_{n_0} - x_0} = \norm{y_{n_0 - x_0} + u_{n_0}} \ge |\underbrace{\norm{y_{n_0-x_0}}}_{\ge 2r} - \underbrace{\norm{u_{n_0}}}_{\le r}| \ge r. + \] + + Verwende nun das Minkowski-Funktional + \[ + p_N(x) \coloneq \inf \{ρ > 0: ρ^{-1} x ∈ N\}, \quad x ∈ X. + \] + Dieses hat die Eigenschaften + \begin{enumerate} + \item + $p_N(\alpha x) = \alpha p_n(x),\quad \alpha \ge 0, x ∈ X$ (positiv homogen) + \item + $p_N(x+y) \le p_N(x) + p_N(y), \quad x, y ∈ X$ (subadditiv) + \item + $p_N(x) \le 1 \iff x ∈ N$ + \item + Ist zusätzlich $\cl{B_r(0)} ⊂ N$, so gilt $p_nNx) \le r^{-1}\norm x$ für alle $x ∈ X$. + \end{enumerate} + Sei nun $X_0 \coloneq \lspan\{x_0\}$ und $f_0 : X_0 → ℝ$ linear definiert durch $f_0(x_0) \coloneq p_N(x_0)$. + Wir behauptung, dass $f_0 (x) \le p_N(x)$ für alle $x = \lambda x_0 ∈ X_0$. + Falls $\lambda \ge 0$, so ist $f_0(x) = f_0(\lambda x_0) = \lambda p_N(x_0) = p_N(\lambda x_0) = p_N(x)$. + Falls $\lambda < 0$, so ist wegen $p_n \ge 0$ ohnehin $f_0(\lambda x_0) = \lambda p_N(x_0) \le 0 \le p_N(\lambda x_0)$. + Da $p_N$ die Bedingungen (i) und (ii) aus Hahn-Banach erfüllt, + gibt es eine lineare Fortsetzung $f$ von $f_0$ mit $f(x) \le p_N(x)$ für alle $x ∈ X$. + + Nun ist $f$ stetig, also $f ∈ X'$, denn für alle $x ∈ X$ gilt + \begin{multline*} + |f(x) = \max\{f(x), -f(x)\} = \max\{f(x),f(-x)\} \le \max\{p_N(x),p_N(-x)\} \\ + \le \max\left\{\frac{\norm{x}}{r},\frac{\norm{-x}}{r}\right\} = \frac{\norm x}{r}. + \end{multline*} + + Außerdem erfüllt $f$ die Gleichung 3.1 (?), denn + \[ + f(x_0) = f_0(x_0) = p_n(x_0) > 1 + \] + und für $x ∈ M ⊂ N$ gilt + \[ + f(x) \le p_N(x) \le 1. + \] +\end{proof} + +\section{Einbettung von $X$ in seinen Bidualraum} +Zunächst zur Motivation: Sei $X$ ein normierter linearer Raum. +Dann existiert $X'$ und ist ein Banachraum. +Aber dann existiert auch $X'' \coloneq (X')'$ und ist ebenfalls ein Banachraum. +Unser Ziel wird es nun sein, $X$ in $X''$ einzubetten. + +\begin{definition} + Die kanonische Abbildung $J_0: X → X''$ ist definiert durch + \[ + J_0(x) [x'] = \lAngle J_0(x), x' \rAngle_{X''×X'} \coloneq \lAngle x', x \rAngle_{X'×X} = x'[x] ∈ \K + \] + für $x ∈ X, x' ∈ X'$. + + Offensichtlich gilt für $x ∈ X$ fest $J_0(x): X' → \K$ linear, aber $J_0(x)$ ist auch stetig bzw beschränkt: + Dazu ist + \[ + |J_0(x)[x']| = | \langle \langle x',x \rAngle \le \norm{x'}_{X'} \underbrace{\norm{x}_X}_{=: M}. + \] + Also ist $J_0(x) ∈ X''$, also insbesondere $J_0$ wohldefiniert. + Wegen der linearität von $J_0$ in $x$ schreiben wir statt $J_0(x)$ auch $J_0 x$. +\end{definition} + +\begin{satz} + Die kanonische Abbildung $J_0: X → X''$ ist eine normerhaltende lineare Einbettung von $X$ in seinen Bidualraum $X''$. +\end{satz} + +\begin{warnung-nn} + $J_0$ ist in der Regel nicht surjektiv. +\end{warnung-nn} + +\begin{proof} + Zur Injektivität: Seien $x_1, x_2 ∈ X$ mit $J_0x_1 = J_0x_2$. + Dann ist für jedes $x' ∈ X'$ + \[ + \lAngle x',x_1 \rAngle = J_0 x_1[x'] = J_0x_2[x'] = \lAngle x', x_2 \rAngle, + \] + also wegen Linearität von $x'$ + \[ + \lAngle x', x_1-x_2 \rAngle = 0. + \] + Mit Folgerung 2.3(1) folgt $x_1-x_2 = 0$. + + Zur Isometrieeigenschaft bleibt zu zeigen: $\norm{J_0x} = \norm{x}$ für alle $x ∈ X''$. + „$\le$“: Aus (4.1) folgt bereits + \[ + \norm{J_0(x)}_{X''} = \sup_{\norm{x'} \le 1} |J_0(x)[x'] \le \norm{x}_X. + \] + „$\ge$“: Zu $x_0 ∈ X$ existiert nach Korollar 2.1 ein $x_0' ∈ X'$ mit + $\norm{x_0'}_{X'} = 1$ und $x_0'[x_0]= \norm{x_0}$. + Also folgt + \[ + \underbrace{|J_0x_0[x_0']|}_{\le \norm{J_0x_0}_{X''}} = \lAngle x_0', x_0 \rAngle = \norm{x_0}. + \] + Da $x_0$ beliebig war, gilt $\norm{J_0x}_{X''} \ge \norm{x}$. +\end{proof} + +\begin{definition} + Ein Banachraum $X$ heißt \emph{reflexiv}, wenn $J_0$ surjektiv ist, also $X$ und $X''$ isomorph sind vermöge $J_0$. +\end{definition} + +\begin{bemerkung-nn} + Ein unvollständiger normierter Raum hätte offensichtlich keine Chance, reflexiv zu sein. +\end{bemerkung-nn} + +\begin{warnung-nn} + „vermöge $J_0$“ in der Definition ist wesentlich, denn es gibt Beispiele mit $X \cong X''$, aber $J_0$ ist nicht surjektiv. %% werner, I 4.7 +\end{warnung-nn} + +\begin{satz} + Jeder Hilbertraum $H$ ist reflexiv +\end{satz} +\begin{proof} + Übung. +\end{proof} + +\begin{bemerkung-nn} + Offensichtlich sind $H$ und $H''$ isometrisch isomorph: + Denn $H$ und $H'$ sind bereits konjugiert linear isomorph via $J_H, X → X'$ (Kapitel IV, \S 5, aus Ries'schem Darstellungssatz). + Mit dem gleichen Argument sind $H'$ und $H''$ konjugiert linear isomorph via $J_{H'}$, also $H$ und $H''$ linear isometrisch durch $J_{H'} \circ J_H$. + Dies genügt aber nicht für den Nachweis der Reflexivität. + Dafür müssen wir zu $x'' ∈ H''$ ein $x ∈ H$ finden mit $J_0x = x''$. +\end{bemerkung-nn} + +\begin{bemerkung-nn} + Wozu Reflexivität gut ist, werden wir später im Kapitel über schwache Topologien genauer sehen. + Beispielsweise ist $\cl{B_1(0)}$ im reflexiven Banachraum $X$ schwach folgenkompakt, das heißt jede Folge in $\cl{B_1(0)}$ hat eine schwach konvergente Teilfolge mit Grenzwert in $\cl{B_1(0)}$. + Dies ist zum Beispiel in der Variationsrechnung sehr wichtig. +\end{bemerkung-nn} + +\begin{definition} + Eine Folge $(x_n)_{n ∈ ℕ}$ in einem normierten Raum $X$ heißt \emph{schwach konvergent} gegen $x ∈ X$ (in Zeichen: $x_n \xrightharpoonup[n → \infty ]{} x$), wenn + \[ + \lim_{n → \infty } x'[x_n] = x'[x] + \] + für alle $x' ∈ X'$ gilt. +\end{definition} + +\begin{bemerkung-nn} + Der Grenzwert (so er denn existiert) ist eindeutig. Denn ist $x'[x] = x'[\tilde x]$ für alle $x' ∈ X'$, so folgt $x = \tilde x$ mit Folgerung 2.3 (2). +\end{bemerkung-nn} + +\begin{beispiel-nn} + Für $(\hat e_i)_{i ∈ ℕ}$ Hilbertraumbasis in einem separablem Hilbertraum $X$ gilt + \[ + \hat e_i \rightharpoonup 0 ∈ X (i → \infty ) + \] +\end{beispiel-nn} + +\begin{bemerkung-nn} + $(\hat e_i)_{i ∈ ℕ}$ ist nicht konvergent in der Normtopologie, die Folge ist noch nicht mal Cauchy, insbesondere ist $\norm{\hat e_i - 0} \not\rightarrow 0 (i → \infty )$. +\end{bemerkung-nn} + +\begin{proof} + Der kanonische Isomorphismus $J_X: X → X', y ↦ y'$ mit $y'[x] = \langle y,x \rangle$ für alle $x ∈ X$ liefert + \[ + X' = \{ x' : x' ∈ X'\} = \{ J_X(y) : y ∈ X\}. + \] + Zu zeigen ist $\lim\limits_{i → \infty }x'[\hat e_i] = x'[0]$ für alle $x' ∈ X'$, also äquivalent + $\lim\limits_{i → \infty } J_x(y)[\hat e_i] = J_x(y)[0]$ für alle $y ∈ X$ bzw. $\lim\limits_{i → \infty } \langle y, \hat e_i \rangle = \langle y, 0 \rangle$ für alle $y ∈ X$. + + Sei also $y ∈ X$ fest gewählt. Dann ist $y = \sum_{i=1}^\infty \alpha _i \hat e_i$ mit $\alpha _i = \langle \hat e_i, y \rangle$. + Es gilt $\sum_{i=1}^\infty |\alpha _i|^2 < \infty $ (vgl Def 4.2.12). + Damit folgt $\alpha _i = \langle \hat e_i, y \rangle → 0 (i → \infty )$, weil $\alpha ∈ \ell^2$. + Damit folgt die Schwache Konvergenz von $(\hat e_i)_{i ∈ ℕ}$. +\end{proof} + + +\begin{satz} + Sei $M$ ein abgeschlossener Unterraum eines Banachraums $(X, \norm -)$. + \begin{enumerate} + \item + Ist $X$ reflexiv, so ist auch $(M, \norm -)$ reflexiv. + \item + Ist $X$ ist reflexiv, so auch $X'$. + \end{enumerate} + +\end{satz} + + +%%% Local Variables: +%%% mode: latex +%%% TeX-master: "funkana-ebook" +%%% End: \ No newline at end of file diff --git a/funkana-ebook.tex b/funkana-ebook.tex new file mode 100644 index 0000000..4bf0282 --- /dev/null +++ b/funkana-ebook.tex @@ -0,0 +1,68 @@ +\documentclass[12pt,footinclude=yes,headinclude=no,a5paper,DIV=50,twoside=false,chapterprefix=true,headings=big]{skript} +\title{Funktionalanalysis} +\subtitle{Mitschrift zur Vorlesung} +\author{\gitAuthorIsoDate \\ {\small\gitReferences} \\ {\small \gitAbbrevHash}} +% \author{Prof. Dr. Maier-Paape} +\date{WS 17/18} + +\AtBeginDocument{ +\newcommand\norm[1]{\left\|#1\right\|} +\def\R{\mathbb{R}} +\def\C{\mathbb{C}} +\def\K{\mathbb{K}} +\def\N{\mathbb{N}} +\def\L{\mathcal{L}} +\def\T{\mathcal{T}} +\def\U{\mathcal{U}} +\def\D{\mathcal{D}} +\def\dd{\;\mathrm{d}} +\def\eps{\varepsilon} +\def\iff{\Leftrightarrow} \def\gdw{\;\Longleftrightarrow\;} +\newcommand\cl[1]{\overline{#1}} +\newcommand\ind[1]{\mathbb{1}_{#1}} +\newcommand\Pot[1]{\mathcal{P}(#1)} +\DeclareMathOperator{\End}{End} +\DeclareMathOperator{\grad}{grad} +\DeclareMathOperator{\lspan}{span} +\DeclareMathOperator{\supp}{supp} +\DeclareMathOperator*{\supess}{sup\,ess} +\DeclareMathOperator{\conv}{conv} +\DeclareMathOperator{\Proj}{proj} +\DeclareMathOperator{\im}{im} +\DeclareMathOperator{\id}{id} +\renewcommand{\Re}{\operatorname{Re}} +\renewcommand{\Im}{\operatorname{Im}} +\def\phi{\varphi} +\def\epsilon{\varepsilon} +\def\Tnat{\ensuremath{\T_{\mathrm{nat}}}} +\def\Tcof{\ensuremath{\T_{\mathrm{cof}}}} +} + +\addbibresource{ref.bib} + +\begin{document} +\sloppy % ohne Ränder sind overfull hboxes eher schlecht, erlaube Unmengen an Kleber. +\maketitle +Dies ist eine Vorlesungsmitschrift, die nichts mit den Dozenten oder dem Lehrstuhl, der die Veranstaltung hält, zu tun hat. + +Alle Fehler sind vermutlich einzig und allein meine Schuld. +Über Verbesserungen und Vervollständigungen freue ich mich sehr (bevorzugt Patches an \verb|ulli.kehrle@rwth-aachen.de|). + +Der Quelltext dieser Mitschrift ist unter \url{https://git.server-speed.net/users/hrnz/funkana.git} online verfügbar. + +Es werden regelmäßig PDFs unter \url{https://hrnz.li/files/funkana/funkana.pdf} (DIN A4, für große Bildschirme und zum Ausdrucken) und \url{https://hrnz.li/files/funkana/funkana-ebook.pdf} (DIN A5, keine Ränder, bietet sich wohl für Ebook-Reader, Smartphones und Tablets an) erscheinen. + +\tableofcontents +\cleardoublepage + +\include{motivation} +\include{ch01-lineare-struktur} +\include{ch02-topologie} +\include{ch03-topologisch-lineare-raeume} +\include{ch04-unitaere-raeume} +\include{ch05-hahn-banach} + +\nocite{*} +\printbibliography + +\end{document} diff --git a/funkana.tex b/funkana.tex index 92764ab..7d586d5 100644 --- a/funkana.tex +++ b/funkana.tex @@ -1,9 +1,11 @@ -\documentclass[12pt,twoside=false,chapterprefix=true,headings=big]{skript} +\documentclass[12pt,DIV=12,twoside=false,chapterprefix=true,headings=big]{skript} \title{Funktionalanalysis} \subtitle{Mitschrift zur Vorlesung} +\author{\gitAuthorIsoDate \\ {\small\gitReferences} \\ {\small \gitAbbrevHash}} % \author{Prof. Dr. Maier-Paape} \date{WS 17/18} +\AtBeginDocument{ \newcommand\norm[1]{\left\|#1\right\|} \def\R{\mathbb{R}} \def\C{\mathbb{C}} @@ -13,6 +15,7 @@ \def\T{\mathcal{T}} \def\U{\mathcal{U}} \def\D{\mathcal{D}} +\def\dd{\;\mathrm{d}} \def\eps{\varepsilon} \def\iff{\Leftrightarrow} \def\gdw{\;\Longleftrightarrow\;} @@ -28,26 +31,38 @@ \DeclareMathOperator{\Proj}{proj} \DeclareMathOperator{\im}{im} \DeclareMathOperator{\id}{id} -\let\Re\relax -\DeclareMathOperator{\Re}{Re} -\let\Im\relax -\DeclareMathOperator{\Im}{Im} +\renewcommand{\Re}{\operatorname{Re}} +\renewcommand{\Im}{\operatorname{Im}} \def\phi{\varphi} +\def\epsilon{\varepsilon} \def\Tnat{\ensuremath{\T_{\mathrm{nat}}}} \def\Tcof{\ensuremath{\T_{\mathrm{cof}}}} +} + +\addbibresource{ref.bib} \begin{document} \maketitle - Dies ist eine Vorlesungsmitschrift, die nichts mit den Dozenten oder dem Lehrstuhl, der die Veranstaltung hält, zu tun hat. -Alle Fehler sind vermutlich meine Schuld. -Über Verbesserungen und Vervollständigungen freue ich mich sehr. -Diese Mitschrift ist unter \url{https://git.server-speed.net/users/hrnz/funkana.git} verfügbar. +Alle Fehler sind vermutlich einzig und allein meine Schuld. +Über Verbesserungen und Vervollständigungen freue ich mich sehr (bevorzugt Patches an \verb|ulli.kehrle@rwth-aachen.de|). + +Der Quelltext dieser Mitschrift ist unter \url{https://git.server-speed.net/users/hrnz/funkana.git} online verfügbar. + +Es werden regelmäßig PDFs unter \url{https://hrnz.li/files/funkana/funkana.pdf} (DIN A4, für große Bildschirme und zum Ausdrucken) und \url{https://hrnz.li/files/funkana/funkana-ebook.pdf} (DIN A5, keine Ränder, bietet sich wohl für Ebook-Reader, Smartphones und Tablets an) erscheinen. \tableofcontents \cleardoublepage -\input{inhalt} +\include{motivation} +\include{ch01-lineare-struktur} +\include{ch02-topologie} +\include{ch03-topologisch-lineare-raeume} +\include{ch04-unitaere-raeume} +\include{ch05-hahn-banach} + +\nocite{*} +\printbibliography \end{document} diff --git a/inhalt.tex b/inhalt.tex index a0fb88a..2957d64 100644 --- a/inhalt.tex +++ b/inhalt.tex @@ -1,9 +1,7 @@ -\section*{Motivation} -\markboth{}{Motivation} - +\section*{Motivation} \markboth{}{Motivation} In der klassischen Analyis haben wir Funktionen im $\K^n$, wobei $\K$ entweder $ℝ$ oder $ℂ$ ist, untersucht. Dabei war das Betrachten von Eigenschaften wie Konvergenz, Stetigkeit und Differenzierbarkeit sehr nützlich. -Die Funktionalanalysis beschäftigt sich nun mit vergleichbaren Problemen in üblicherweise unendlich-dimensionalen Funktionenräumen. +Die Funktionalanalysis beschäftigt sich nun mit vergleichbaren Problemen in üblicherweise unendlich"=dimensionalen Funktionenräumen. Hierfür werden wir versuchen, die aus der klassischen Analysis bekannten Untersuchungsmethoden zu verallgemeinern. Doch zunächst ein paar Probleme, für deren Lösung man die Funktionalanalysis benötigt. @@ -11,41 +9,41 @@ Doch zunächst ein paar Probleme, für deren Lösung man die Funktionalanalysis Ein klassisches Beispiel aus der Variationsrechnung: Wir wollen die Funktion \[ - f(u) = \int_0^π |u'(x)|^2 dx + f(u) = \int_0^\pi |u'(x)|^2 dx \] - unter den Nebenbedingungungen $u(0) = u(π) = 0$ und $\int_0^π |u(x)|^2 dx = 1$ minimieren. + unter den Nebenbedingungungen $u(0) = u(\pi ) = 0$ und $\int_0^\pi |u(x)|^2 dx = 1$ minimieren. In der klassischen Analysis haben wir für Minimierungsprobleme mit Nebenbedingungungen Lagrange-Multiplikatoren genutzt. - Im unendlich-dimensionalen Fall ist das jedoch nicht so einfach. - Wir betrachten $f : Y → ℝ$ wie oben, wobei $Y$ eine Teilmenge des unendlich-dimensionalen Funktionenraums + Im unendlich"=dimensionalen Fall ist das jedoch nicht so einfach. + Wir betrachten $f : Y → ℝ$ wie oben, wobei $Y$ eine Teilmenge des unendlich"=dimensionalen Funktionenraums \[ - X = \left\{ u ∈ C^1[0,π]: u(0) = u(π) = 0 \right\} + X = \left\{ u ∈ C^1[0,\pi ]: u(0) = u(\pi ) = 0 \right\} \] ist, die durch \[ - Y = \left\{ u ∈ X: \int_0^π |u(x)|^2 dx = 1 \right\} + Y = \left\{ u ∈ X: \int_0^\pi |u(x)|^2 dx = 1 \right\} \] gegeben ist. - Zwar ist $Y$ (in der $\L^2([0,π])$-Metrik) beschränkt und abgeschlossen, jedoch nicht kompakt. + Zwar ist $Y$ (in der $\L^2([0,\pi ])$-Metrik) beschränkt und abgeschlossen, jedoch nicht kompakt. \end{problem-nn} -\begin{problem-nn}[Fourierreihenentwicklung] +\begin{problem-nn} Sei $\mathcal T = \{ 1, \cos t, \sin t, \cos (2t), \sin (2t), … \} = \{\phi_i\}_{i ∈ ℕ}$. Dann ist bekanntlich \[ - \langle \phi_i, \phi_j \rangle = ∫_0^{2π} φ_i(t) φ_j(t) dt = 2π δ_{i,j}, + \langle \phi_i, \phi_j \rangle = ∫_0^{2\pi } φ_i(t) φ_j(t) dt = 2\pi \delta _{i,j}, \] - wobei $δ_{i,j}$ das Kronecker-Delta bezeichne. + wobei $\delta _{i,j}$ das Kronecker-Delta bezeichne. Also lässt sich durch Normierung ein Orthonormalsystem aus $\mathcal T$ gewinnen. - Jetzt fragen wir uns, ob sich jede $2π$-periodische Funktion $u$ bezüglich eines geeigneten Konvergenzbegriffs in eine Reihe $u = \sum_{i ∈ ℕ} α_i φ_i$ mit $α_i ∈ ℝ$ entwickeln können. + Jetzt fragen wir uns, ob sich jede $2\pi $-periodische Funktion $u$ bezüglich eines geeigneten Konvergenzbegriffs in eine Reihe $u = \sum_{i ∈ ℕ} \alpha _i φ_i$ mit $\alpha _i ∈ ℝ$ entwickeln können. Bereits bekannt ist, dass das für das entsprechende endlich-dimensionale Problem geht: Sei $T = \{ e_1,…,e_n\}$ die kanonische Standardbasis des $ℝ^n$ Dann gilt bekanntlich \[ - \langle e_i, e_j \rangle_{ℝ^n} = δ_{i,j} + \langle e_i, e_j \rangle_{ℝ^n} = \delta _{i,j} \] und für jedes $x ∈ ℝ^n$ ist \[ - x = \sum_{i=1}^n α_i e_i, \quad α_i = \langle x, e_i \rangle_{ℝ^n}. + x = \sum_{i=1}^n \alpha _i e_i, \quad \alpha _i = \langle x, e_i \rangle_{ℝ^n}. \] - Wir fragen uns nach den Zusammenhängen zwischen den Problemen im endlich- und unendlich-dimensionalen. + Wir fragen uns nach den Zusammenhängen zwischen den Problemen im endlich- und unendlich"=dimensionalen. \end{problem-nn} \begin{problem-nn} Das Biegemoment eines Trägers kann man als Randwertaufgabe (gesucht ist $u: [0,1] → ℝ$, gegeben sind $p,r: [0,1] → ℝ$) @@ -54,7 +52,7 @@ Doch zunächst ein paar Probleme, für deren Lösung man die Funktionalanalysis \] bestimmen. Mit Hilfte der sogenannten Green'schen Funktion lässt sich diese Randwertaufgabe in eine Integralgleichung \[ - (T_u)(t) := ∫_0^1 G(t,s) \big(r(s)-p(s)u(s)\big) ds = u + (T_u)(t) \coloneq ∫_0^1 G(t,s) \big(r(s)-p(s)u(s)\big) ds = u \] umwandeln. Das heißt, man sucht einen Fixpunkt eines Integraloperators $T$ in einer geeigneten Menge von Funktionen. \end{problem-nn} @@ -77,11 +75,11 @@ Sei im folgenden stets $\K = ℝ$ oder $\K = ℂ$. Zunächst die \[ \cdot : \K × X → X \] - heißt $\K$-Vektorraum, falls für alle $α, β ∈ \K$ und $x, y ∈ X$ gilt: + heißt $\K$-Vektorraum, falls für alle $\alpha , β ∈ \K$ und $x, y ∈ X$ gilt: \begin{enumerate}[label=(V\arabic*)] - \item $α x+y) = αx + βy$ - \item $(α+β)x = αx + βx$ - \item $(αβ)x = α(βx)$ + \item $\alpha x+y) = \alpha x + βy$ + \item $(\alpha +β)x = \alpha x + βx$ + \item $(\alpha β)x = \alpha (βx)$ \item $1 \cdot x = x$ \end{enumerate} \end{definition} @@ -91,7 +89,7 @@ Sei im folgenden stets $\K = ℝ$ oder $\K = ℂ$. Zunächst die \end{bemerkung-nn} \begin{bemerkung-nn} - Eine nichtleere Teilmenge $Y ⊂ X$ ist bereits dann ein linearer Raum, falls aus $α, β ∈ \K$, $x, y ∈ Y$ bereits $αx + βy ∈ Y$ folgt, also $Y$ abgeschlossen unter den Vektorraumoperationen ist. + Eine nichtleere Teilmenge $Y ⊂ X$ ist bereits dann ein linearer Raum, falls aus $\alpha , β ∈ \K$, $x, y ∈ Y$ bereits $\alpha x + βy ∈ Y$ folgt, also $Y$ abgeschlossen unter den Vektorraumoperationen ist. $Y$ heißt dann \emph{linearer Teilraum} oder auch \emph{linearer Unterraum}. \end{bemerkung-nn} @@ -99,24 +97,24 @@ Sei im folgenden stets $\K = ℝ$ oder $\K = ℂ$. Zunächst die Zu jeder Teilmenge $M ⊂ X$ bildet die Menge aller Linearkombinationen von je endlich vieler Elemente einen linearen Teilraum von $X$. Dieser heißt die \emph{lineare Hülle} von $M$ oder der \emph{Aufspann} von $M$ \[ - \lspan M = \left\{ x ∈ X: ∃ l ∈ ℕ, α_1,…,α_l ∈ \K, m_1,…,m_l ∈ M \text{ mit } \sum_{i=1}^l α_i m_i = x \right\}. + \lspan M = \left\{ x ∈ X: ∃ l ∈ ℕ, \alpha _1,…,\alpha _l ∈ \K, m_1,…,m_l ∈ M \text{ mit } \sum_{i=1}^l \alpha _i m_i = x \right\}. \] \end{bemerkung-nn} \begin{bemerkung-nn} - $M = \{x_λ\}_{λ ∈ Λ} ⊂ X$ heißt \emph{Basis} oder \emph{Hamel-Basis} von $X$, falls $M$ \emph{linear unabhängig}, das heißt, - $0 ∈ X$ lässt sich nur auf triviale Art und Weise als Linearkombination endlich vieler der $x_λ$ schreiben, und $\lspan M = X$ ist. + $M = \{x_\lambda \}_{\lambda ∈ \Lambda } ⊂ X$ heißt \emph{Basis} oder \emph{Hamel-Basis} von $X$, falls $M$ \emph{linear unabhängig}, das heißt, + $0 ∈ X$ lässt sich nur auf triviale Art und Weise als Linearkombination endlich vieler der $x_\lambda $ schreiben, und $\lspan M = X$ ist. \end{bemerkung-nn} \begin{bemerkung-nn} - Besitzt $X$ eine Basis von $n < ∞$ Elementen, dann heißt $n$ die \emph{Dimension} von $X$ und wir schreiben $\dim X = n$. - Andernfalls heißt $X$ \emph{unendlich-dimensional} ($\dim X = ∞$). + Besitzt $X$ eine Basis von $n < \infty $ Elementen, dann heißt $n$ die \emph{Dimension} von $X$ und wir schreiben $\dim X = n$. + Andernfalls heißt $X$ \emph{unendlich-dimensional} ($\dim X = \infty $). \end{bemerkung-nn} \begin{bemerkung-nn} Seien $X_1, X_2 ⊂ X$ lineare Teilräume. Dann ist \[ - X_1 + X_2 := \left\{ αx_1 + βx_2: α, β ∈ \K, x_1 ∈ X_1, x_2 ∈ X_2 \right\} + X_1 + X_2 \coloneq \left\{ \alpha x_1 + βx_2: \alpha , β ∈ \K, x_1 ∈ X_1, x_2 ∈ X_2 \right\} \] ebenfalls ein linearer Teilraum. Falls $X_1 ∩ X_2 = \{ 0\}$, schreiben wir $X_1 \oplus X_2$ und nennen die Summe \emph{direkt}. @@ -139,16 +137,16 @@ Sei im folgenden stets $\K = ℝ$ oder $\K = ℂ$. Zunächst die \[ C[a,b] = \{x: [a,b] → \K, x \text { ist stetig}\} \] - ein $\K$-Vektorraum mit $\dim C[a,b] = ∞$. + ein $\K$-Vektorraum mit $\dim C[a,b] = \infty $. Zum Beispiel sind die Monome $(t^k)_{k ∈ ℕ}$ ein unendliches linear unabhängiges System, jedoch keine Basis. Tatsächlich ist jede Basis dieses Raumes überabzählbar. \end{beispiel} \section{Lineare Abbildungen} \begin{definition} - Seien $X, Y$ lineare Räume über $\K$. $A: X → Y$ heißt \emph{linear}, falls für alle $x_1, x_2 ∈ X$ und $α, β ∈ \K$ gilt: + Seien $X, Y$ lineare Räume über $\K$. $A: X → Y$ heißt \emph{linear}, falls für alle $x_1, x_2 ∈ X$ und $\alpha , β ∈ \K$ gilt: \[ - A(αx_1 + βx_2) = αA(x_1) + βA(x_2). + A(\alpha x_1 + βx_2) = \alpha A(x_1) + βA(x_2). \] $A: X → \K$ heißt \emph{lineares Funktional}. Für $A$ linear heißt $R(A) = \im A = \{A(x): x ∈ X\}$ der \emph{Bildraum} von $A$ und $N(A) = \ker A = \{ x ∈ X: A(x) = 0\}$ der \emph{Kern} von $A$. @@ -157,7 +155,7 @@ Sei im folgenden stets $\K = ℝ$ oder $\K = ℂ$. Zunächst die \begin{bemerkung} Sei $A: X → Y$ linear. \begin{enumerate} - \item Sei $M ⊂ X $ ein linearer Unterraum. Dann ist $A(M) ⊂ Y$ wieder ein linearer Unterraum und es gilt $\dim A(M) \le \dim M$ mit Gleichheit bei injektivität. + \item Sei $M ⊂ X $ ein linearer Unterraum. Dann ist $A(M) ⊂ Y$ wieder ein linearer Unterraum und es gilt $\dim A(M) \le \dim M$ mit Gleichheit bei Injektivität. \item Es gilt \[ A \text{ injektiv} \Longleftrightarrow N(A) = \{ 0\}. @@ -167,14 +165,14 @@ Sei im folgenden stets $\K = ℝ$ oder $\K = ℂ$. Zunächst die X/(N(A)) \cong \im A. \] \item - Falls $\dim X = \dim Y = n < ∞$, dann ist $A$ genau dann injektiv, wenn $A$ surjektiv ist. + Falls $\dim X = \dim Y = n < \infty $, dann ist $A$ genau dann injektiv, wenn $A$ surjektiv ist. \item - $A: X → Y$ ist linear und bijektiv genau dann, wenn es eine lineare Umkehrabbildung $A^{-1}: Y → X$. + $A: X → Y$ ist linear und bijektiv genau dann, wenn es eine lineare Umkehrabbildung $A^{-1}: Y → X$ gibt. \item Falls so ein $A: X → Y$ linear und bijektiv existiert, nennen wir $X$ und $Y$ \emph{linear isomorph.} $A$ heißt dann ein \emph{linearer Isomorphismus}. - Nur falls $\dim X = \dim Y < ∞$ sind $X$ und $Y$ auch „topologisch“ isomorph. + Nur falls $\dim X = \dim Y < \infty $ sind $X$ und $Y$ auch „topologisch“ isomorph. In diesem Fall erhält man die Prototypen $ℝ^n$ und $ℂ^n$ für endlich-dimensionale Vektorräume und andere gitbt es nicht (die sie auch als Topologische Räume isomorph sind). \end{enumerate} \end{bemerkung} @@ -183,7 +181,7 @@ Sei im folgenden stets $\K = ℝ$ oder $\K = ℂ$. Zunächst die $X = \{ x: [a,b] → ℝ, x, \dot x, \ddot x \text{ stetig},\; x(a) = \dot x(a) = 0\}$ ist ein linearer Raum. Sei $Y = C[a,b]$ und $A: X → Y$ gegeben durch \[ - (Ax)(t) := \ddot x(t) + c_1 (t) \dot x (t) + c_2 (t) x(t), \quad t ∈ [a,b], c_1,c_2 ∈ C[a,b]. + (Ax)(t) \coloneq \ddot x(t) + c_1 (t) \dot x (t) + c_2 (t) x(t), \quad t ∈ [a,b], c_1,c_2 ∈ C[a,b]. \] Dann ist $A$ linear, weil differenzieren linear ist und $A$ ist injektiv: Zunächst ist $x = 0$ eine Lösung der linearen Differentialgleichung zweiter Ordnung $Ax = 0$. @@ -201,16 +199,16 @@ Sei im folgenden stets $\K = ℝ$ oder $\K = ℂ$. Zunächst die \begin{beispiel-nn} Sei $X = Y = C[a,b]$, $A: X → X$ gegeben durch \[ - (Ax)(t) := ∫_a^b k(s,t) x(s) ds, \quad t ∈ [a,b], + (Ax)(t) \coloneq ∫_a^b k(s,t) x(s) ds, \quad t ∈ [a,b], \] wobei $k : [a,b] × [a,b] → ℝ$ stetig und gegeben ist. Dann ist $A$ linear, da das Integral linear ist. - Auch ist, wenn $λ ∈ ℝ$ ein Parameter ist, die Abbildung + Auch ist, wenn $\lambda ∈ ℝ$ ein Parameter ist, die Abbildung \[ - (A_λx)(t) := λx(t) - (Ax)t), \quad t ∈ [a,b] + (A_\lambda x)(t) \coloneq \lambda x(t) - (Ax)t), \quad t ∈ [a,b] \] linear. - Die Probleme $Ax = y$ (bei gegebenem $y ∈ Y$ und gesuchtem $x ∈ X$) oder $A_λ x = 0$ (gesucht ist $λ ∈ ℝ$ und eine nichttriviale Lösung $x ∈ X \setminus \{ 0\}$) + Die Probleme $Ax = y$ (bei gegebenem $y ∈ Y$ und gesuchtem $x ∈ X$) oder $A_\lambda x = 0$ (gesucht ist $\lambda ∈ ℝ$ und eine nichttriviale Lösung $x ∈ X \setminus \{ 0\}$) heißen Integralgleichungen erster und zweiter Ordnung. \end{beispiel-nn} @@ -233,7 +231,7 @@ Sei im folgenden stets $\K = ℝ$ oder $\K = ℂ$. Zunächst die Ax = (0,ξ_1, ξ_2, \dots) ∈ \ell^2. \] $A$ heißt (Rechts-)Shiftoperator und ist linear und injektiv, jedoch nicht surjektiv. - Solche Abbildungen gibt es für $\dim X = \dim Y < ∞$ nicht. + Solche Abbildungen gibt es für $\dim X = \dim Y < \infty $ nicht. \end{beispiel-nn} \section{Duale Räume} @@ -247,14 +245,14 @@ Wir schreiben nun \] Wir setzen \[ - X^f := \left\{ x': x' \text{ ist lineares Funktional auf } X \right\}. + X^f \coloneq \left\{ x': x' \text{ ist lineares Funktional auf } X \right\}. \] Hierbei sollte man nicht $x'$ nicht mit der Ableitung von $x$ verwechseln. Auch ist $\langle -, - \rangle_{X × X^f}$ kein Skalarprodukt. Der Raum $X^f$ wird auf natürlicher Weise zum linearen Raum mit \[ - (αx_1' + βx_2')(x) := αx_1'(x) + βx_2'(x), \quad x ∈ X, x_1', x_2' ∈ X^f, α, β ∈ \K. + (\alpha x_1' + βx_2')(x) \coloneq \alpha x_1'(x) + βx_2'(x), \quad x ∈ X, x_1', x_2' ∈ X^f, \alpha , β ∈ \K. \] So ist \[ @@ -263,7 +261,7 @@ So ist bilinear. \begin{definition} $X^f$ heißt der \emph{algebraische Dualraum} zu $X$. - $X^{ff} := (X^f)^f$ heißt der \emph{biduale Raum} zu $X$. + $X^{ff} \coloneq (X^f)^f$ heißt der \emph{biduale Raum} zu $X$. \end{definition} \begin{beispiel-nn} @@ -273,7 +271,7 @@ bilinear. \] mit \[ - \langle x', x'' \rangle := \langle x. x' \rangle \quad ∀x' ∈ X^f. + \langle x', x'' \rangle \coloneq \langle x. x' \rangle \quad ∀x' ∈ X^f. \] Damit ist $x'': X^f → \K$ linear wohldefiniert. \end{beispiel-nn} @@ -283,23 +281,23 @@ bilinear. \end{definition} \begin{bemerkung} - $X$ ist genau dann algebraisch reflexiv, wenn $\dim X < ∞$ ist. + $X$ ist genau dann algebraisch reflexiv, wenn $\dim X < \infty $ ist. - Im Fall $\dim X < ∞$ lässt sich leicht eine duale Basis angeben: - Sei dazu $M := \{x_1,…,x_n\}$ eine Basis von $X$. Dann wird durch + Im Fall $\dim X < \infty $ lässt sich leicht eine duale Basis angeben: + Sei dazu $M \coloneq \{x_1,…,x_n\}$ eine Basis von $X$. Dann wird durch \[ - \langle x_i, x_k' \rangle := δ_{i,k} + \langle x_i, x_k' \rangle \coloneq \delta _{i,k} \] - und linearer Fortsetzung die Menge $ M := \{x_1',…,x_n'\} ⊂ X^f$ erklärt. + und linearer Fortsetzung die Menge $ M \coloneq \{x_1',…,x_n'\} ⊂ X^f$ erklärt. Dann ist $M'$ eine Basis von $X'$, die die \emph{duale Basis} von $M$ genannt wird. - Tatsächlich ist $X^f$ im Falle $\dim X = ∞$ wesentlich größer. + Tatsächlich ist $X^f$ im Falle $\dim X = \infty $ wesentlich größer. Man wählt deshalb eine (neue) Defintion des Dualraums: \end{bemerkung} \begin{definition}[Dualraum] Zu einem linearen Raum $X$ ist \[ - X' := \left\{ x' : X → \K, x' \text{ linear und stetig} \right\} ⊂ X^f + X' \coloneq \left\{ x' : X → \K, x' \text{ linear und stetig} \right\} ⊂ X^f \] der Dualraum von $X$. \end{definition} @@ -320,7 +318,7 @@ Um Allerdings von Stetigkeit reden zu können, müssen wir zunächst \emph{Topol \item Für alle Mengen $X$ ist $\T = \Pot X$ eine Topologie, die sogenannte \emph{diskrete Topologie} oder \emph{feinste Topologie} auf $X$. \item - In Analysis I wird eine Menge $U ⊂ ℝ$ für offen erklärt, wenn es zu jedem $x ∈ U$ ein $ε > 0$ gibt, so dass für alle $ y ∈ ℝ$ mit $|x - y| < ε$ auch $y ∈ U$ gilt. + In Analysis I wird eine Menge $U ⊂ ℝ$ für offen erklärt, wenn es zu jedem $x ∈ U$ ein $\epsilon > 0$ gibt, so dass für alle $ y ∈ ℝ$ mit $|x - y| < \epsilon $ auch $y ∈ U$ gilt. Aus der Analysis ist bekannt, dass die so definierten offenen Mengen den Axiomen genügen. Diese Topologie $\Tnat$ wird \emph{natürliche Topologie} genannt. \item @@ -342,22 +340,22 @@ Um Allerdings von Stetigkeit reden zu können, müssen wir zunächst \emph{Topol \item $U ⊂ X$ heißt \emph{Umgebung von $A$}, wenn es eine offene Menge $V$ gibt mit $A ⊂ V ⊂ U$. Wir setzen \[ - \U_A := \U_A (\T) := \{ U ⊂ X : U\; \text{Umgebung von $A$}\}. + \U_A \coloneq \U_A (\T) \coloneq \{ U ⊂ X : U\; \text{Umgebung von $A$}\}. \] $\U_A$ heißt \emph{Umgebungssystem} oder \emph{Umgebungsfilter} von $A ⊂ X$. - Für $x ∈ X$ setzen wir $\U_x := \U_{\{x\}}$. $x$ heißt dann \emph{innerer Punkt} von $U$ für alle $U ∈ \U_x$. + Für $x ∈ X$ setzen wir $\U_x \coloneq \U_{\{x\}}$. $x$ heißt dann \emph{innerer Punkt} von $U$ für alle $U ∈ \U_x$. \item $x ∈ X$ heißt \emph{Häufungspunkt} von $M$, falls jede Umgebung von $x_0$ ein $y ∈ M$ enthält mit $y \ne x$.k \item Das \emph{Innere von M} ist \[ - M^\circ := \bigcup \left\{ U ∈ \T: U ⊂ M \right\} + M^\circ \coloneq \bigcup \left\{ U ∈ \T: U ⊂ M \right\} \] die größte offene Menge, die in $M$ enthalten ist. \item Der \emph{Abschluss von} M ist \[ - \cl M := \bigcap \left\{ U ⊂ M: U \text{ abgeschlossen} \right\} + \cl M \coloneq \bigcap \left\{ U ⊂ M: U \text{ abgeschlossen} \right\} \] die kleinste abgeschlossene Menge, die $M$ enthält. \item @@ -455,7 +453,7 @@ Um Allerdings von Stetigkeit reden zu können, müssen wir zunächst \emph{Topol \begin{definition}[Relativtopologie oder Spurtopologie] $M \subset \T$ eines topologischen Raumes $(X,\T)$ lässt sich in natürlicher Weise - zu einem topologischen Raum machen, nämlich mit $\T' := {M \cap V : V \in \T}$. + zu einem topologischen Raum machen, nämlich mit $\T' \coloneq {M \cap V : V \in \T}$. \end{definition} \begin{bemerkung} $M = M \cap X \in \T'$ da $X \in \T$, d.h. $M$ ist offen in der Spurtopologie. @@ -507,21 +505,23 @@ Um Allerdings von Stetigkeit reden zu können, müssen wir zunächst \emph{Topol \item Es gilt \[ - \lim_{n \to ∞} x_n = x \; \Longleftrightarrow \; \lim_{n→∞} d(x,x_n) = 0. + \lim_{n \to \infty } x_n = x \; \Longleftrightarrow \; \lim_{n→\infty } d(x,x_n) = 0. \] \item - Es ist $x_0 ∈ M$ genau dann ein innerer Punkt von $M ⊂ X$, wenn ein $ε > 0$ existiert mit $B_ε(x_0) ⊂ M$. + Es ist $x_0 ∈ M$ genau dann ein innerer Punkt von $M ⊂ X$, wenn ein $\epsilon > 0$ existiert mit $B_\epsilon (x_0) ⊂ M$. \item - $M$ ist nirgends dicht in $X$ genau dann, wenn es zu jeder Kugel $B_ε(x_0)$ mit $x_0 ∈ X, ε > 0$ eine Kugel $B_δ(x_1) ⊂ B_ε(x_0)$ mit $B_θ(x_1) ∩ M = \emptyset$ gibt. + $M$ ist nirgends dicht in $X$ genau dann, wenn es zu jeder Kugel +$B_\epsilon (x_0)$ mit $x_0 ∈ X, \epsilon > 0$ eine Kugel $B_\delta (x_1) ⊂ +B_\epsilon (x_0)$ mit $B_\delta(x_1) ∩ M = \emptyset$ gibt. \item Seien $(X,d_X)$ und $(Y,d_Y)$ metrische Räume. Dann ist auch $(X×Y,d_{X×Y})$ ein metrischer Raum vermöge der Metrik \[ - d_{X×Y}((x_1,y_1),(x_2,y_2)) := \max\{d_x(x_1,x_2),d_y(y_1,y_2)\} + d_{X×Y}((x_1,y_1),(x_2,y_2)) \coloneq \max\{d_x(x_1,x_2),d_y(y_1,y_2)\} \] oder auch mit \[ - d_{X×Y}((x_1,y_1),(x_2,y_2)) := \sqrt{d_x^2(x_1,x_2)+d_y^2(y_1,y_2)}. + d_{X×Y}((x_1,y_1),(x_2,y_2)) \coloneq \sqrt{d_x^2(x_1,x_2)+d_y^2(y_1,y_2)}. \] Tatsächlich induzieren diese beiden Metriken die gleiche Topologie (nämlich die Produkttopologie) \item @@ -532,7 +532,7 @@ Um Allerdings von Stetigkeit reden zu können, müssen wir zunächst \emph{Topol heißen \emph{Isometrien}. \item Ein metrischer Raum muss im allgemeinen keine lineare Struktur haben. - Man betrachte hierzu die Menge $X := \{1,2,3,4,5,6\}$ mit der diskreten Metrik. + Man betrachte hierzu die Menge $X \coloneq \{1,2,3,4,5,6\}$ mit der diskreten Metrik. Diese kann keine Vektorraumstruktur haben, da $|X| = 6$ keine Primzahlpotenz ist. \end{enumerate} \end{lemma} @@ -561,7 +561,7 @@ Nun ein paar Charakterisierungen von kompakten Mengen in metrischen Räumen. \section{Vollständigkeit in metrischen Räumen und der Satz von Baire} \begin{definition} - Eine Folge $(x_n)_{n ∈ ℕ} ⊂ X$ in $(X,d)$ heißt \emph{Cauchy-Folge}, falls zu jedem $ε > 0$ ein $N = N(ε)$ existiert mit $d(x_m,x_n) < ε$ für alle $n,m \ge N$. + Eine Folge $(x_n)_{n ∈ ℕ} ⊂ X$ in $(X,d)$ heißt \emph{Cauchy-Folge}, falls zu jedem $\epsilon > 0$ ein $N = N(\epsilon )$ existiert mit $d(x_m,x_n) < \epsilon $ für alle $n,m \ge N$. \end{definition} \begin{lemma} @@ -580,14 +580,14 @@ jedoch lässt sich jeder metrische Raum zu einem vollständigen Erweitern. Dieser Raum $(\tilde X, \tilde d)$ heißt die Vervollständigung von $(X,d)$. \end{satz} \begin{proof} - Zwei Cauchyfolgen $(x_n)_{n ∈ ℕ}$ und $(y_n)_{n ∈ ℕ}$ seien äquivalent, falls $d(x_n,y_n) \xrightarrow[n → ∞]{} 0$. + Zwei Cauchyfolgen $(x_n)_{n ∈ ℕ}$ und $(y_n)_{n ∈ ℕ}$ seien äquivalent, falls $d(x_n,y_n) \xrightarrow[n → \infty ]{} 0$. Hierdurch ist eine Äquivalenzrelation definiiert. Sei $[(x_n)_{n ∈ ℕ}]$ die vom Repräsententaten $(x_n)_{n ∈ ℕ}$ erzeugte Klasse. Man setzt \[ - \tilde X := \{ [ (x_n)_{n ∈ ℕ}] : (x_n)_{n ∈ ℕ} \text{ ist Cauchy-Folge in }(X,d)\} + \tilde X \coloneq \{ [ (x_n)_{n ∈ ℕ}] : (x_n)_{n ∈ ℕ} \text{ ist Cauchy-Folge in }(X,d)\} \] und \[ - \tilde d([(x_n)_{n ∈ ℕ}],[(y_n)_{n ∈ ℕ}]) := \lim_{n → ∞} d(x_n,y_n). + \tilde d([(x_n)_{n ∈ ℕ}],[(y_n)_{n ∈ ℕ}]) \coloneq \lim_{n → \infty } d(x_n,y_n). \] Dann ist $(d(x_n,y_n))_{n ∈ ℕ}$ eine Cauchy-Folge in $ℝ$, da \[ @@ -605,17 +605,18 @@ jedoch lässt sich jeder metrische Raum zu einem vollständigen Erweitern. \begin{bemerkung-nn} Wendet man diese Technik auf $ℚ$ mit der natürlichen Metrik an, dann erhält man $(ℝ,d)$ als vollständige Hülle. + Man beachte jedoch, dass dies nicht für die Konstruktion von $ℝ$ ausreicht, da hier schon die Existenz von $ℝ$ verwenden wird -- Aber das funktioniert größtenteils analog. \end{bemerkung-nn} \begin{satz}[Schachtelsatz]\label{schachtelsatz} Sei $(X,d)$ ein vollständiger metrischer Raum und seien - $(x_n)_{n * ℕ} ⊂ X$ und $(r_n)_{n ∈ ℕ} ⊂ (0,∞) $ Folgen mit der Eigenschaft + $(x_n)_{n * ℕ} ⊂ X$ und $(r_n)_{n ∈ ℕ} ⊂ (0,\infty ) $ Folgen mit der Eigenschaft \begin{enumerate} \item $\cl B_{r_{n+1}}(x_{n+1}) ⊂ B_{r_n} (x_n)$ - \item $\lim_{n \to ∞} r_n = 0$. + \item $\lim_{n \to \infty } r_n = 0$. \end{enumerate} - Dann gibt es genau ein $x_0 ∈ X$ mit $x_0 ∈ \bigcap_{n ∈ ℕ \cl B_{r_n} (x_n)}$. + Dann gibt es genau ein $x_0 ∈ X$ mit $x_0 ∈ \bigcap_{n ∈ ℕ} \cl B_{r_n} (x_n)$. \end{satz} \begin{proof} @@ -625,14 +626,14 @@ jedoch lässt sich jeder metrische Raum zu einem vollständigen Erweitern. \] Also \[ - d(x_{n+p},x_n) \le r_n \xrightarrow[n → ∞]{} 0. + d(x_{n+p},x_n) \le r_n \xrightarrow[n → \infty ]{} 0. \] Damit ist $(x_n){n ∈ ℕ}$ eine Cauchyfolge und damit konvergiert gegen ein $x_0 ∈ X$. Außerdem gilt \[ - d(xp,x_n) \le \underbrace{d(x_0, x_{n+p})}_{→ 0 (p → ∞)} + \underbrace{d(x_{n+p},x_n)}_{ \le r_n}. + d(x_p,x_n) \le \underbrace{d(x_0, x_{n+p})}_{→ 0 (p → \infty )} + \underbrace{d(x_{n+p},x_n)}_{ \le r_n}. \] - Damit folgt für $p → ∞$ + Damit folgt für $p → \infty $ \[ d(x_0, x_n) \le r_n \quad ∀ n ∈ ℕ \] @@ -640,7 +641,7 @@ jedoch lässt sich jeder metrische Raum zu einem vollständigen Erweitern. Für die Eindeutigkeit sei $\tilde x_0$ ebenfalls in $\bigcap_{n ∈ ℕ} \cl B_{r_n}(x_n)$. Dann folgt \[ - d(x_0,\tilde x_0) \le \underbrace{d(x_0,x_n)}_{\le r_n} + \underbrace{d(x_n, \tilde x_0)}_{\le r_n} \le 2r_n \xrightarrow[n → ∞]{} 0. + d(x_0,\tilde x_0) \le \underbrace{d(x_0,x_n)}_{\le r_n} + \underbrace{d(x_n, \tilde x_0)}_{\le r_n} \le 2r_n \xrightarrow[n → \infty ]{} 0. \] Doch damit war bereits $x_0 = \tilde x_0$. \end{proof} @@ -672,7 +673,7 @@ Der folgende Satz wird beim Beweis mehrerer fundamentaler Sätze benötigt, z.B B_{r_2}(x_2) ⊂ B_{r_1/2} (x_1) \] und $B_{r_2}(x_2) ∩ M_2 = \emptyset$. - Durch Fortsetzen dieses Schemas finden wir eine Folge $(x_n)_{n ∈ ℕ} ⊂ X$ und Radien $(r_n)_{n ∈ ℕ} ⊂ (0,∞)$ mit $r_n \le r/2^n \xrightarrow[n → ∞]{} 0$. + Durch Fortsetzen dieses Schemas finden wir eine Folge $(x_n)_{n ∈ ℕ} ⊂ X$ und Radien $(r_n)_{n ∈ ℕ} ⊂ (0,\infty )$ mit $r_n \le r/2^n \xrightarrow[n → \infty ]{} 0$. Damit sind alle Voraussetzungen von \cref{schachtelsatz} erfüllt. Folglich existiert genau ein \[ \tilde x ∈ \bigcap_{n ∈ ℕ} B_{r_n} (x_n) ⊂ B_r(x_0) ⊂ M. @@ -711,14 +712,14 @@ Der folgende Satz wird beim Beweis mehrerer fundamentaler Sätze benötigt, z.B % $\cl{B_{r_n}(x_n)} ⊂ U_n ∩ B_{r_{n-1}} (x_{n-1})$ % \end{enumerate} % Dazu beachte man, dass $U_n ∩ B_{r_{n-1}}(x_{n-1})$ nichtleer und offen -% ist, also existiert $x_n$ und $\frac 1 n > ε > 0$ mit $B_ε(x_n) ⊂ U_n ∩ -% B_{r_{n-1}} (x_{n-1})$ und $r_n = \frac ε 2$ ist wie gewünscht. Für $m +% ist, also existiert $x_n$ und $\frac 1 n > \epsilon > 0$ mit $B_\epsilon (x_n) ⊂ U_n ∩ +% B_{r_{n-1}} (x_{n-1})$ und $r_n = \frac \epsilon 2$ ist wie gewünscht. Für $m % \ge n$ impliziert (ii), dass $x_m ∈ B_{r_n}(x_n)$ und aus (i) folgt, % dass die Folge $(x_n)_{n ∈ ℕ}$ damit eine Cauchyfolge ist. Damit % konvergiert $(x_n)_{n ∈ ℕ}$ gegen ein $x ∈X$. Sei nun $N ∈ ℕ$ und $m > % N$. Dann folgt aus $x_m ∈ B_{r_N}(x_N)$, dass % \begin{align*} -% x &= \lim_{m → ∞} x_m ∈ \cl{B_{r_N}(x_n)} ⊂ U_N ∩ B_{r_{N-1}}(x_{N-1}) \\ +% x &= \lim_{m → \infty } x_m ∈ \cl{B_{r_N}(x_n)} ⊂ U_N ∩ B_{r_{N-1}}(x_{N-1}) \\ % & ⊂ U_N ∩ B_{r_1}(x_1) ⊂ U_N ∩ W, % \end{align*} % also $x ∈ \bigcap_{n ∈ ℕ} U_N ∩ W$. @@ -776,12 +777,12 @@ Erklärtes Ziel dieses Kapitels wird sein, die beiden Strukturen aus den vorheri \section{Normierte Räume} \begin{definition} - Sei $X$ ein linearer Raum über $\K$. Die Abbildung $\norm\cdot: X → [0,∞)$ - heißt \emph{Norm} auf $X$, falls für alle $x, y ∈ X, α ∈ K$ gilt: + Sei $X$ ein linearer Raum über $\K$. Die Abbildung $\norm\cdot: X → [0,\infty )$ + heißt \emph{Norm} auf $X$, falls für alle $x, y ∈ X, \alpha ∈ K$ gilt: \begin{enumerate} \item $\norm x = 0 \Longleftrightarrow x = 0$ \item - $\norm{αx} = |α| \norm x$ + $\norm{\alpha x} = |\alpha | \norm x$ \item $\norm{x+y} \le \norm x + \norm y$ \end{enumerate} @@ -789,7 +790,7 @@ Erklärtes Ziel dieses Kapitels wird sein, die beiden Strukturen aus den vorheri \end{definition} \begin{bemerkung} - Durch $d(x,y) := \norm{x-y}$ wird ein normierter Raum auch ein metrischer, also insbesondere auch ein topologischer Raum. + Durch $d(x,y) \coloneq \norm{x-y}$ wird ein normierter Raum auch ein metrischer, also insbesondere auch ein topologischer Raum. Diese induzierte Topologie auf $(X, \norm\cdot)$ heißt \emph{Normtopologie}. Ohne die lineare Struktur macht der normierte Raum gar keinen Sinn, da für die Definition einiger der Normaxiome die Vektorraumoperationen verwendet werden. @@ -798,35 +799,35 @@ Erklärtes Ziel dieses Kapitels wird sein, die beiden Strukturen aus den vorheri \begin{beispiele} \begin{enumerate} \item - Betrachte den $ℝ^n$ mit $\norm x _{p} := \left( \sum_{i=1}^n |x_i|^p \right)^{1/p}$ mit $1 \le p < ∞$ ist ein normierter Raum, - genauso wie mit $\norm{x}_{∞} := \max_{1 \le i \le n} |x_i|$. + Betrachte den $ℝ^n$ mit $\norm x _{p} \coloneq \left( \sum_{i=1}^n |x_i|^p \right)^{1/p}$ mit $1 \le p < \infty $ ist ein normierter Raum, + genauso wie mit $\norm{x}_{\infty } \coloneq \max_{1 \le i \le n} |x_i|$. Insbesondere gibt es im $ℝ^n$ überabzählbar viele verschiedene Normen. Wir werden jedoch später sehen, dass diese Normen alle die gleiche Topologie erzeugen. \item - Der Raum aller stetigen Funktionen auf einem kompaktem Intervall $C[a,b]$ mit $\norm{x}_{∞} := \max_{t ∈ [a,b]} |x(t)|$ ist ein normierter Raum. + Der Raum aller stetigen Funktionen auf einem kompaktem Intervall $C[a,b]$ mit $\norm{x}_{\infty } \coloneq \max_{t ∈ [a,b]} |x(t)|$ ist ein normierter Raum. Außerdem wird durch \[ - \norm x := ∫_a^b |x(t)| dt + \norm x \coloneq ∫_a^b |x(t)| dt \] ebenfalls eine Norm definiert. \item Sei $\Omega ⊂ ℝ^n$ offen und beschränkt. Dann wird $C(\cl{\Omega})$ mit \[ - \norm{x}_{∞} := \max_{t ∈ \cl \Omega} |x(t)| + \norm{x}_{\infty } \coloneq \max_{t ∈ \cl \Omega} |x(t)| \] auch zu einem normierten Raum. \item $L^p(\Omega) = \L^p(\Omega)/\mathcal N$, wobei $\mathcal N = \{ f: \Omega → \R, f(t) = 0 \text{ fast überall}\}$ ist mit \[ - \norm x := \left(∫_{\Omega} |x(t)|^p dt \right)^{1/p} + \norm x \coloneq \left(∫_{\Omega} |x(t)|^p dt \right)^{1/p} \] - ein normierter Raum, wobei $1 \le p < ∞$. + ein normierter Raum, wobei $1 \le p < \infty $. \item $\ell^p$ mit \[ - \norm x _{p} := \left( \sum_{i=1}^n |x_i|^p \right)^{1/p} + \norm x _{p} \coloneq \left( \sum_{i=1}^n |x_i|^p \right)^{1/p} \] - ist ebenfalls ein normierter Raum, wobei $1 \le p < ∞$. + ist ebenfalls ein normierter Raum, wobei $1 \le p < \infty $. \end{enumerate} \end{beispiele} @@ -834,13 +835,13 @@ Erklärtes Ziel dieses Kapitels wird sein, die beiden Strukturen aus den vorheri Sei $(X, \norm\cdot)$ ein normierter Raum. Dann sind die Abbildungen $+$, $\cdot$ und $\norm\cdot$ stetig. \end{lemma} \begin{proof} - Für beliebige Folgen $(x_n)_{n ∈ ℕ},(y_n)_{n ∈ ℕ} ⊂ X, (α_n)_{n ∈ ℕ}$ mit $\lim x_n = x$, $\lim y_n = y$, $\lim α_n = α$ gelten + Für beliebige Folgen $(x_n)_{n ∈ ℕ},(y_n)_{n ∈ ℕ} ⊂ X, (\alpha _n)_{n ∈ ℕ}$ mit $\lim x_n = x$, $\lim y_n = y$, $\lim \alpha _n = \alpha $ gelten \[ \norm{(x_n + y_n) - (x+y)} \le \norm{x-x_n} + \norm{y -y_n} \] sowie \[ - \norm{α_nx_n - αx} \le |α_n| \norm{x_n-x} + \norm{x} |α_n - α| + \norm{\alpha _nx_n - \alpha x} \le |\alpha _n| \norm{x_n-x} + \norm{x} |\alpha _n - \alpha | \] und \[ @@ -858,12 +859,12 @@ Erklärtes Ziel dieses Kapitels wird sein, die beiden Strukturen aus den vorheri \section{Topologische lineare Räume} \begin{bemerkung-nn} Hierbei sei stetis die Topologie von $X×X$ die Produktopologie, bei den Körpern $\K = \begin{cases} ℝ \\ ℂ \end{cases}$ die übliche Topologie. - Wir schreiben im Folgenden für Mengen $M, M_1, M_2 ⊂ X$ und $α ⊂ \K$ nun + Wir schreiben im Folgenden für Mengen $M, M_1, M_2 ⊂ X$ und $\alpha ⊂ \K$ nun \[ - M_1 + M_2 := s(M_1,M_2) := \{x+y : x ∈ M_1, y ∈ M_2\}, + M_1 + M_2 \coloneq s(M_1,M_2) \coloneq \{x+y : x ∈ M_1, y ∈ M_2\}, \] \[ - A \cdot M := m(A,M) := \{ αx: α ∈ A, x ∈ M\}. + A \cdot M \coloneq m(A,M) \coloneq \{ \alpha x: \alpha ∈ A, x ∈ M\}. \] \end{bemerkung-nn} @@ -890,11 +891,11 @@ Erklärtes Ziel dieses Kapitels wird sein, die beiden Strukturen aus den vorheri \begin{enumerate} \item Die Addition $m$ ist stetig. \item - Für beliebiges $α ∈ \K, x ∈ X$ gilt: Zu jeder Umgebung $O_{αx} ∈ \T$ existieren Umgebungen $O_x ∈ \T$ von $x$ und $O_α ∈ \T$ von $y$ mit $O_α × O_x ⊂ O_{αx}$. + Für beliebiges $\alpha ∈ \K, x ∈ X$ gilt: Zu jeder Umgebung $O_{\alpha x} ∈ \T$ existieren Umgebungen $O_x ∈ \T$ von $x$ und $O_\alpha ∈ \T$ von $y$ mit $O_\alpha × O_x ⊂ O_{\alpha x}$. \end{enumerate} \end{lemma} -Betrachtet man insbesondere die Stetigkeit am Punke $α=0$ und $x ∈ X$ beliebig, dann gilt also: +Betrachtet man insbesondere die Stetigkeit am Punke $\alpha =0$ und $x ∈ X$ beliebig, dann gilt also: Für jede Umgebung $O_0 ∈ \U_0 ⊂ X$ existiert eine Umgebung $O_x ∈ \U_x$ und ein $r > 0$, so dass \[ ∀β: |β| 0 ∃ δ > 0 ∃ r> 0 ∀β ∈ \K ∀y ∈ X: + ∀\epsilon > 0 ∃ \delta > 0 ∃ r> 0 ∀β ∈ \K ∀y ∈ X: \begin{rcases} - |β - α| < r \\ - d(x,y) < δ + |β - \alpha | < r \\ + d(x,y) < \delta \end{rcases} - \implies d(βy,αx) < ε + \implies d(βy,\alpha x) < \epsilon \] \begin{lemma} \label{lemma-metrischer-linearer-raum-charak} Sei $(X,d)$ ein metrischer Raum mit linearer Struktur und mit einer translationinvarianten Metrik. - Dann ist $X$ mit der von $d$ erzeugten Topologie ein \emph{metrischer linearer Raum} genau dann, wenn für alle $α ∈ \K, x ∈ X$ und beliebige Nullfolgen $(x_n)_{n ∈ ℕ} ⊂ X, (α_n)_{n ∈ ℕ) ⊂ \K}$ gilt + Dann ist $X$ mit der von $d$ erzeugten Topologie ein \emph{metrischer linearer Raum} genau dann, wenn für alle $\alpha ∈ \K, x ∈ X$ und beliebige Nullfolgen $(x_n)_{n ∈ ℕ} ⊂ X, (\alpha _n)_{n ∈ ℕ)} ⊂ \K$ gilt \begin{gather*} - αx_n \xrightarrow[n → ∞]{} 0 \\ - αx_n \xrightarrow[n → ∞]{} 0 \\ - α_nx_n \xrightarrow[n → ∞]{} 0 + \alpha x_n \xrightarrow[n → \infty ]{} 0 \\ + \alpha x_n \xrightarrow[n → \infty ]{} 0 \\ + \alpha _nx_n \xrightarrow[n → \infty ]{} 0 \end{gather*} \end{lemma} \begin{proof} @@ -1002,28 +1003,29 @@ Für die Stetigkeit der skalaren Multiplikation im Punkt $(α,x) ∈ \K × X$ ha „$⇐$“: Wegen der Äquivalenz von Stetigkeit und Folgenstetigkeit ist zu zeigen \[ \begin{rcases} - α_n \xrightarrow[n → ∞]{} α ∈ \K \\ - x_n \xrightarrow[n → ∞]{} x ∈ X + \alpha _n \xrightarrow[n → \infty ]{} \alpha ∈ \K \\ + x_n \xrightarrow[n → \infty ]{} x ∈ X \end{rcases} - \implies α_n x_n \xrightarrow[n → ∞]{} αx. + \implies \alpha _n x_n \xrightarrow[n → \infty ]{} \alpha x. \] - Sei dazu $z_n := x_n - x ∈ X$, $γ_n := α_n - α ∈ \K$. Dann ist + Sei dazu $z_n \coloneq x_n - x ∈ X$, $γ_n \coloneq \alpha _n - \alpha ∈ \K$. Dann ist \[ - γ_n z_n + γ_n x + α z_n = (α_n - α)(x_n-x) + (α_n-α) x + α(x_n-x) - = α_n x_n - α×. + γ_n z_n + γ_n x + \alpha z_n = (\alpha _n - \alpha )(x_n-x) + (\alpha _n-\alpha ) x + \alpha (x_n-x) + = \alpha _n x_n - \alpha ×. \] Somit ist \begin{align*} - d(α_nx_n,αx) &= d(αnx_n - αx,0) = d(γ_nz_n + γnx + αz_n, 0) \\ - &\le \underbrace{d(γ_nz_n,0)}_{→ 0} + \underbrace{d(γ_nx, 0)}_{→ 0} + \underbrace{d(αz_n, 0)}_{→ 0} \xrightarrow{n → 0} 0. + d(\alpha _nx_n,\alpha x) &= d(\alpha nx_n - \alpha x,0) = d(γ_nz_n + γnx + \alpha z_n, 0) \\ + &\le \underbrace{d(γ_nz_n,0)}_{→ 0} + \underbrace{d(γ_nx, 0)}_{→ 0} + +\underbrace{d(\alpha z_n, 0)}_{→ 0} \xrightarrow{n → \infty } 0. \end{align*} Da die Addition ohnehin immer stetig ist, sind wir fertig. \end{proof} \begin{definition} - Eine Abbildung $|\cdot|: X → [0,∞)$ heißt \emph{Quasi-Norm} auf dem Linearen + Eine Abbildung $|\cdot|: X → [0,\infty )$ heißt \emph{Quasi-Norm} auf dem Linearen Raum $X$, falls gilt: \begin{enumerate}[label=(Q\arabic*)] \item @@ -1033,11 +1035,11 @@ Für die Stetigkeit der skalaren Multiplikation im Punkt $(α,x) ∈ \K × X$ ha \item $|x+y| \le |x| + |y|$ für alle $x,y ∈ X$ \item - $|αx_n| \xrightarrow[n → ∞]{} 0$ für $α ∈ \K$, falls $|x_n| → 0$ + $|\alpha x_n| \xrightarrow[n → \infty ]{} 0$ für $\alpha ∈ \K$, falls $|x_n| → 0$ \item - $|α_nx| \xrightarrow[n → ∞]{} 0$ für $x ∈ X$, falls $|α_n| → 0$ + $|\alpha _nx| \xrightarrow[n → \infty ]{} 0$ für $x ∈ X$, falls $|\alpha _n| → 0$ \item - $|α_nx_n| \xrightarrow[n → ∞]{} 0$ falls $|x_n| → 0$ und $|α_nx_n| → 0$ + $|\alpha _nx_n| \xrightarrow[n → \infty ]{} 0$ falls $|x_n| → 0$ und $|\alpha _nx_n| → 0$ \end{enumerate} $(X,|\cdot|)$ heißt dann \emph{quasi-normierter} Raum. \end{definition} @@ -1049,10 +1051,10 @@ Für die Stetigkeit der skalaren Multiplikation im Punkt $(α,x) ∈ \K × X$ ha \begin{satz} \begin{enumerate} \item - Ist $|\cdot|$ eine Quasi-Norm auf $X$, so wird durch $d(x,y) := |x-y|$ eine translationsinvariante Metrik definiert, welche $X$ zu einem metrischen linearen Raum macht. + Ist $|\cdot|$ eine Quasi-Norm auf $X$, so wird durch $d(x,y) \coloneq |x-y|$ eine translationsinvariante Metrik definiert, welche $X$ zu einem metrischen linearen Raum macht. \item Ist $(X,d)$ ein metrischer linearer Raum mit translationsinvarianter Metrik $d$, so ist - $(X,|\cdot|)$ mit $|x| := d(x,0)$ ein quasi-normierter Raum. + $(X,|\cdot|)$ mit $|x| \coloneq d(x,0)$ ein quasi-normierter Raum. \end{enumerate} \end{satz} \begin{proof} @@ -1067,11 +1069,11 @@ Speziell für die Anwendung sehr wichtige metrische lineare Räume werden von Se Eine Abbildung $p: X → ℝ$ heißt \emph{Semi-Norm} oder \emph{Halbnorm}, falls folgendes gilt: \begin{enumerate}[label=(S\arabic*)] \item - $∀x ∈ X: p(x) ≥ 0$ + $∀x ∈ X: p(x) \ge 0$ \item - $∀ x ∈ X, α ∈ \K: p(αx) = |α| p(x)$ + $∀ x ∈ X, \alpha ∈ \K: p(\alpha x) = |\alpha | p(x)$ \item - $∀ x, y ∈ X: p(x+y) ≤ p(x) + p(y)$ + $∀ x, y ∈ X: p(x+y) \le p(x) + p(y)$ \end{enumerate} $(X,p)$ heißt dann \emph{semi-normierter} Raum. \end{definition} @@ -1092,7 +1094,7 @@ Speziell für die Anwendung sehr wichtige metrische lineare Räume werden von Se \end{equation} Dann ist \[ - d(x,yr) := \sum_{n = 1}^∞ 2^{-n} \frac{p_n(x-y)}{1+p_n(x-y)} + d(x,y) \coloneq \sum_{n = 1}^\infty 2^{-n} \frac{p_n(x-y)}{1+p_n(x-y)} \] eine translationsinvariante Metrik auf $X$, welche $X$ zum metrischen linearen Raum macht. \end{satz} @@ -1108,24 +1110,24 @@ Speziell für die Anwendung sehr wichtige metrische lineare Räume werden von Se \begin{satz} \label{satz-umgebungsbasis-produkt-von-seminorm} Sei $(X,d)$ der in \cref{satz-abzaehlbares-prod-seminormierter-raeume} gegebene metrische lineare Raum (mit der von der Metrik erzeugten Topologie). - Dann bilden die Mengen ($ε_n > 0$) + Dann bilden die Mengen ($\epsilon _n > 0$) \[ - U (p_n,ε_n) := \bigcup B^{p_n}_{ε_n}(0) - = \{ x ∈ X: p_n(x) < ε_n\} + U (p_n,\epsilon _n) \coloneq \bigcup B^{p_n}_{\epsilon _n}(0) + = \{ x ∈ X: p_n(x) < \epsilon _n\} \] und deren endliche Durchschnitte eine Umgebungsbasis von $0 ∈ X$ \end{satz} \begin{bemerkung-nn} - Nach dem Invarianzprinzip ist damit durch $\bigcup B^{p_n}_{ε_n}$ die ganze Topologie bestimmt. + Nach dem Invarianzprinzip ist damit durch $\bigcup B^{p_n}_{\epsilon _n}$ die ganze Topologie bestimmt. Mit anderen Worten: Die Topologie welche über die Metrik bestimmt ist, ist dieselbe wie die, welche von den - $U(p_n,ε_n)$ und endlichen Schnitten davon erzeugt wird. + $U(p_n,\epsilon _n)$ und endlichen Schnitten davon erzeugt wird. \end{bemerkung-nn} \begin{proof}[\cref{satz-umgebungsbasis-produkt-von-seminorm}] - Zunächst ist $U (p_n,ε_n) ∈ \T$: - Sei $n ∈ ℕ$ und $ε_n > 0$ fest und $y ∈ U(p_n,ε_n)$ beliebig gegeben. - Dann ist $p_n(y) < ε_n$. Dann wähle $ρ = ρ(y) > 0$, so dass $p_n(y) + ρ < ε_n$. - Dann gilt für $r := 2^{-n} \frac{ρ}{1+ρ} > 0$: + Zunächst ist $U (p_n,\epsilon _n) ∈ \T$: + Sei $n ∈ ℕ$ und $\epsilon _n > 0$ fest und $y ∈ U(p_n,\epsilon _n)$ beliebig gegeben. + Dann ist $p_n(y) < \epsilon _n$. Dann wähle $ρ = ρ(y) > 0$, so dass $p_n(y) + ρ < \epsilon _n$. + Dann gilt für $r \coloneq 2^{-n} \frac{ρ}{1+ρ} > 0$: \[ x ∈ B_r(y) \implies p_n(x+r) < ρ. \] @@ -1133,10 +1135,10 @@ Speziell für die Anwendung sehr wichtige metrische lineare Räume werden von Se \[ \frac{p_n(x-y)}{1+p_n(x-y)} \le 2^n \underbrace{d(x,y)}_{< r} < 2^n r = \frac{ρ}{1+ρ}, \] - also $p_n(x-y) < ρ$. Mit diesem $r$ gilt $B_r(y) ⊂ U(p_n,ε_n)$: + also $p_n(x-y) < ρ$. Mit diesem $r$ gilt $B_r(y) ⊂ U(p_n,\epsilon _n)$: Sei $x ∈ B_r(y)$. Dann gilt \[ - p_n(x) \le \underbrace{p_n(x-y)}_{< ρ} + p_n(y) < p_n(y) + ρ = ε_n + p_n(x) \le \underbrace{p_n(x-y)}_{< ρ} + p_n(y) < p_n(y) + ρ = \epsilon _n \] wie gewünscht. @@ -1144,31 +1146,31 @@ Speziell für die Anwendung sehr wichtige metrische lineare Räume werden von Se Sei $ B_r(0), r > 0$ gegeben. Wähle $n_0 ∈ ℕ$ mit \[ - \sum_{n=n_0}^∞ 2^{-n} < \frac r 2. + \sum_{n=n_0}^\infty 2^{-n} < \frac r 2. \] - mit $ε := \frac r 2 $ gilt dann + mit $\epsilon \coloneq \frac r 2 $ gilt dann \[ - \bigcap_{n=1}^{n_0} U(p_(,ε) ⊂ B_r(0). + \bigcap_{n=1}^{n_0} U(p_(,\epsilon ) ⊂ B_r(0). \] - Sei dazu $x ∈ \bigcap_{n=1}^{n_0} U(p_n,ε)$ beliebig. + Sei dazu $x ∈ \bigcap_{n=1}^{n_0} U(p_n,\epsilon )$ beliebig. Dann ist \[ - d(x,0) \le \sum_{n=1}^{n_0} 2^{-n} \frac{p_n(x)}{1+p_n(x)} + \sum_{n=n_0}^∞ 2^{-n} < ε \sum_{n=1}^{n_0} 2^{-n} + \frac r 2 < ε + \frac r 2 = r, + d(x,0) \le \sum_{n=1}^{n_0} 2^{-n} \frac{p_n(x)}{1+p_n(x)} + \sum_{n=n_0}^\infty 2^{-n} < \epsilon \sum_{n=1}^{n_0} 2^{-n} + \frac r 2 < \epsilon + \frac r 2 = r, \] somit also $x ∈ B_r(0)$. \end{proof} \begin{bemerkung} - Die Mengen $U(p_n,ε_n)$ und deren endlichen Schnitte sind konvexe Mengen, das heißt + Die Mengen $U(p_n,\epsilon _n)$ und deren endlichen Schnitte sind konvexe Mengen, das heißt \[ - x, y ∈ U(p_n,ε_n),α ∈ [0,1] \implies αx+(1-α)y ∈ U(p_n,ε_n) + x, y ∈ U(p_n,\epsilon _n),\alpha ∈ [0,1] \implies \alpha x+(1-\alpha )y ∈ U(p_n,\epsilon _n) \] \end{bemerkung} \begin{proof} Es ist \[ - p_n(αx + (1-α)y) \le |α| \underbrace{p_n(x)}_{< ε_n} + |1-α|\underbrace{p_n(y)}_{< ε_n} = ε_n. + p_n(\alpha x + (1-\alpha )y) \le |\alpha | \underbrace{p_n(x)}_{< \epsilon _n} + |1-\alpha |\underbrace{p_n(y)}_{< \epsilon _n} = \epsilon _n. \] \end{proof} @@ -1185,10 +1187,10 @@ Also besitzt der in \cref{satz-abzaehlbares-prod-seminormierter-raeume} gewonne \] Dann sind die Mengen \[ - U(p_i,ε_i) = \{ x ∈ X: p_{(x) < ε_i}\}, \quad ε_i > 0, i ∈ I + U(p_i,\epsilon _i) = \{ x ∈ X: p_{(x) < \epsilon _i}\}, \quad \epsilon _i > 0, i ∈ I \] und deren endliche Schnitte eine konvexe Umgebungsbasis von $0 ∈ X$. - Die dadurch gewonne Topologie $\T$ macht $X$ zu einem \emph{lokalkonvexen Hausdorff-Raum}. + Die dadurch gewonne Topologie $\T$ macht $X$ zu einem \emph{lokalkonvexen Hausdorff"=Raum}. \end{satz} \section{Beispiele} @@ -1197,9 +1199,9 @@ Wir werden die unten angegebenen Beispiele auch gleich auf Vollständigkeit unte \begin{definition} \begin{enumerate} \item - Ein metrischer linearer Raum $(X,d)$ der vollständig ist, heißt \emph{Fréchet-Raum}. + Ein metrischer linearer Raum $(X,d)$ der vollständig ist, heißt \emph{Fréchet"=Raum}. \item - Ein normierter Raum $(X,\norm\cdot)$, der vollständig ist, heißt \emph{Banach-Raum}. + Ein normierter Raum $(X,\norm\cdot)$, der vollständig ist, heißt \emph{Banach"=Raum}. \end{enumerate} \end{definition} @@ -1207,119 +1209,122 @@ Wir werden die unten angegebenen Beispiele auch gleich auf Vollständigkeit unte \begin{beispiel-nn}[$\ell^p$-Räume] \begin{enumerate} \item - $(\ell^p,\norm\cdot_p)$, $1 \le p < ∞$ ist normierter Raum mit + $(\ell^p,\norm\cdot_p)$, $1 \le p < \infty $ ist normierter Raum mit \[ - \norm x _p = \left( \sum_{i=1}^∞ |x_i|^p \right)^{1/p}. + \norm x _p = \left( \sum_{i=1}^\infty |x_i|^p \right)^{1/p}. \] \item - $(\ell^∞,\norm\cdot_∝)$, ist normierter Raum mit $\norm x _∞ = \sup_{i ∈ ℕ} |x_i|$. + $(\ell^\infty ,\norm\cdot_\infty)$, ist normierter Raum mit $\norm x _\infty = \sup_{i ∈ ℕ} |x_i|$. \item $(\ell^p,|\cdot|_p = \norm\cdot_p^p)$, $0 \le p < 1$ ist quasi-normierter Raum. \end{enumerate} \end{beispiel-nn} \begin{bemerkung} - Für $0 < p < q \le ∞$ gilt $\ell^p ⊂ \ell^q ⊂ \ell^∞$. + Für $0 < p < q \le \infty $ gilt $\ell^p ⊂ \ell^q ⊂ \ell^\infty $. \end{bemerkung} \begin{beweis} Sei $x ∈ \ell^p$ mit $|x| = 1 = \sum_{i ∈ ℕ} |x_i|^p$. Dann ist für alle $i ∈ ℕ$ $|x_i|^p \le 1$, also auch $|x_i| < 1$. - Dann folgt auch $\sum_{i ∈ ℕ} |x_i|^q < 1$, also $x ∈ \ell^q$ und $\sup_{i ∈ ℕ} |x_i| ≤ 1$, also $x ∈ \ell^∞$. + Dann folgt auch $\sum_{i ∈ ℕ} |x_i|^q < 1$, also $x ∈ \ell^q$ und $\sup_{i ∈ ℕ} |x_i| \le 1$, also $x ∈ \ell^\infty $. \end{beweis} \begin{satz} - Für $1 \le p \le ∞$ ist $(\ell^p,\norm\cdot_p)$ ein Banachraum. - Für $0 < p < ∞$ ist $(\ell^p,|\cdot|_p)$ ein Fréchet-Raum. + Für $1 \le p \le \infty $ ist $(\ell^p,\norm\cdot_p)$ ein Banachraum. + Für $0 < p < \infty $ ist $(\ell^p,|\cdot|_p)$ ein Fréchet-Raum. \end{satz} \begin{proof} - Nur für $1 \le p < ∞$. + Nur für $1 \le p < \infty $. Sei dazu $(x_n)_{n ∈ ℕ} ⊂ \ell^p$ eine Cauchy-Folge, also - $x_n = (ξ_k^n)_{k ∈ ℕ}$ und für jedes $ε > 0$ gibt es ein $n_0$ mit + $x_n = (ξ_k^n)_{k ∈ ℕ}$ und für jedes $\epsilon > 0$ gibt es ein $n_0$ mit \[ - ∀n,m > n_0: \norm{x_n-xm}_p = \left( \sum_{k=1}^∞ |ξ_k^n-ξ_k^m|^p \right)^{1/p} < ε. + ∀n,m > n_0: \norm{x_n-x_m}_p = \left( \sum_{k=1}^\infty |ξ_k^n-ξ_k^m|^p \right)^{1/p} < \epsilon . \] Sei $k_0 ∈ ℕ$ beliebig. Dann ist $(ξ_k^n)_{n ∈ ℕ}$ eine Cauchy-Folge in $\K$, besitzt also einen Grenzwert $ξ_{k_0}$. - Setze nun $x := (ξ_k)_{k ∈ ℕ} ∈ \K^∞ = s$. Wir vermuten $x$ als Grenzwert unserer Cauchy-Folge. + Setze nun $x \coloneq (ξ_k)_{k ∈ ℕ} ∈ \K^\infty = s$. Wir vermuten $x$ als Grenzwert unserer Cauchy-Folge. Also müssen wir zeigen, dass $x ∈ \ell^p$, und dass unsere Folge tatsächlich gegen $x$ konvergiert. Es gilt \[ - \norm{x_n}_! \le \underbrace{\norm{x_n-x_{n_0}}}_{< ε} + \norm{x_{n_0}} \quad \forall n \ge n_0 + \norm{x_n}_! \le \underbrace{\norm{x_n-x_{n_0}}}_{< \epsilon } + \norm{x_{n_0}} \quad \forall n \ge n_0 \] Deshalb existiert ein $M > 0$ mit $\norm{x_n}_p < M$ für alle $n ∈ ℕ$, also \[ - \sum_{k=1}^N |ξ_k^n|p < \sum_{k =1}^∞ |ξ_k^n|^p \le M^p < ∞. + \sum_{k=1}^N |ξ_k^n|p < \sum_{k =1}^\infty |ξ_k^n|^p \le M^p < \infty . \] Also haben wir \[ \sum_{k=1}^N |ξ_k^p| \le M^p \quad ∀ n ∈ ℕ, \] - also durch Grenzwertbildung $N → ∞$ auch $\norm{x}_p^p ≤ M^p$ bzw. $x ∈ \ell^p$. + also durch Grenzwertbildung $N → \infty $ auch $\norm{x}_p^p \le M^p$ bzw. $x ∈ \ell^p$. Ferner haben wir \[ - \sum_{k=1}^N |ξ_k^n-ξ_k^m|^p < ε^p \quad ∀ N ∈ ℕ, n, m ≥ n_0(ε). + \sum_{k=1}^N |ξ_k^n-ξ_k^m|^p < \epsilon ^p \quad ∀ N ∈ ℕ, n, m \ge n_0(\epsilon ). \] - Für $n → ∞$ folgt + Für $n → \infty $ folgt \[ - \sum_{k=1}^N |ξ_k-ξ_k^m|^p < ε^p \quad ∀N ∈ ℕ, m ≥ n_0, + \sum_{k=1}^N |ξ_k-ξ_k^m|^p < \epsilon ^p \quad ∀N ∈ ℕ, m \ge n_0, \] - und mit $N → ∞$ + und mit $N → \infty $ \[ - \sum_{k=1}^∞ |ξ_k-ξ_k^m|^p < ε^p \quad ∀m ≥ n_0, + \sum_{k=1}^\infty |ξ_k-ξ_k^m|^p < \epsilon ^p \quad ∀m \ge n_0, \] also die Konvergenz. \end{proof} \begin{beispiel-nn} - Betrachte den Folgenraum $S = \K^∞ = \{x = (ξ_n)_{n ∈ ℕ}, ξ_n ∈ \K\}$. + Betrachte den Folgenraum $S = \K^\infty = \{x = (ξ_n)_{n ∈ ℕ}, ξ_n ∈ \K\}$. Dann ist \[ - p_n(x) := |ξ_n|, \quad p_n: \K^∞ → ℝ + p_n(x) \coloneq |ξ_n|, \quad p_n: \K^\infty → ℝ \] eine abzählbare Familie von Halbnormen mit \[ - p_n(x) = 0 ∀n ∈ ℕ \implies x = 0 ∈ \K^∞ + p_n(x) = 0 ∀n ∈ ℕ \implies x = 0 ∈ \K^\infty \] - Nach \cref{satz-abzaehlbares-prod-seminormierter-raeume} folgt, dass $(\K^∞, d)$ mit + Nach \cref{satz-abzaehlbares-prod-seminormierter-raeume} folgt, dass $(\K^\infty , d)$ mit \[ - d(x,y) := \sum_{n ∈ ℕ} 2^{-n} \frac{p_n(x-y)}{1+p_n(x-y)} + d(x,y) \coloneq \sum_{n ∈ ℕ} 2^{-n} \frac{p_n(x-y)}{1+p_n(x-y)} \] ein metrischer linearer Raum ist. Der Konvergenzbegriff entspricht gerade der komponentenweisen Konvergenz, das heißt, für eine Folge $(x_k)_{k ∈ ℕ}$ mit $x_k = (ξ^k_n)_{n ∈ ℕ}$ gilt - \[ - x_k \xrightarrow[k→∞]{} 0 \; \Longleftrightarrow \; d(x_n,0) \xrightarrow[k→∞]{} 0 \; \Longleftrightarrow \; p_n(x_k) \xrightarrow[k→∞]{} ∀ n ∈ ℕ \; \Longleftrightarrow \; |ξ_n^k| \xrightarrow[k→∞] 0 ∀ n ∈ ℕ. - \] + \begin{align*} + x_k \xrightarrow[k→\infty ]{} 0 + &\gdw d(x_n,0) \xrightarrow[k→\infty ]{} 0 \\ + &\gdw p_n(x_k) \xrightarrow[k→\infty ]{} ∀ n ∈ ℕ \\ + &\gdw |ξ_n^k| \xrightarrow[k→\infty ]{} 0 ∀ n ∈ ℕ. + \end{align*} - Wir fragen uns nun, ob auf dem $\K^∞$ auch eine Topologie existiert, so dass der induzierte Konvergenzbegriff der der gleichmäßigen Konvergenz in allen Komponenten entspricht? + Wir fragen uns nun, ob auf dem $\K^\infty $ auch eine Topologie existiert, so dass der induzierte Konvergenzbegriff der der gleichmäßigen Konvergenz in allen Komponenten entspricht. Also \[ - x_k \xrightarrow[k → ∞]{\text{glm}} 0 ∈ \K^∞ \gdw ∀ε > 0 ∃ k_0 ∈ ℕ: |ξ_n^k| < ε ∀ k \ge k_0 ∀n ∈ ℕ. + x_k \xrightarrow[k → \infty ]{\text{glm}} 0 ∈ \K^\infty \gdw ∀\epsilon > 0 ∃ k_0 ∈ ℕ: |ξ_n^k| < \epsilon ∀ k \ge k_0 ∀n ∈ ℕ. \] - Wenn $\K^∞$ ein topologischer linearer Raum sein soll, ist das nicht möglich. Notwendig wäre, dass für eine Folge $x  ∈ \K^∞$ + Wenn $\K^\infty $ ein topologischer linearer Raum sein soll, ist das nicht möglich. Notwendig wäre, dass für eine Folge $x  ∈ \K^\infty $ \[ - α_k \xrightarrow[k → ∞]{} 0 \text{ in } \K \implies α_k x \xrightarrow[k→∞]{} \text{ in } X = \K^∞. + \alpha _k \xrightarrow[k → \infty ]{} 0 \text{ in } \K \implies \alpha _k x \xrightarrow[k→\infty ]{} \text{ in } X = \K^\infty . \] - Wähle dazu die Nullfolge $(α_k)_{k ∈ ℕ} = (1/k)_{k ∈ ℕ} ⊂ ℝ$, $x= (n)_{n ∈ ℕ} ⊂ X$. Dann ist + Wähle dazu die Nullfolge $(\alpha _k)_{k ∈ ℕ} = (1/k)_{k ∈ ℕ} ⊂ ℝ$, $x= (n)_{n ∈ ℕ} ⊂ X$. Dann ist \[ - α_k x = (n/k)_{n ∈ ℕ} ∈ \K^∞ + \alpha _k x = (n/k)_{n ∈ ℕ} ∈ \K^\infty \] - zwar eine Nullfolge in $\K^∝$ ist, diese Konvergenz ist jedoch nicht gleichmäßig in $n$. - Man kann zeigen, dass $\K^∞$ mit $d$ vollständig, also ein Fréchet-Raum, ist. - Ist $\K^∞$ auch normierbar? - Also gibt es auf $\K^∞$ eine Norm, welche die gleiche Topologie erzeugt wie die $d$? + zwar eine Nullfolge in $\K^\infty$ ist, diese Konvergenz ist jedoch nicht gleichmäßig in $n$. + Man kann zeigen, dass $\K^\infty $ mit $d$ vollständig, also ein Fréchet-Raum, ist. + Ist $\K^\infty $ auch normierbar? + Also gibt es auf $\K^\infty $ eine Norm, welche die gleiche Topologie erzeugt wie die $d$? Auch das ist nicht möglich: \end{beispiel-nn} \begin{lemma} \label{lemma-s-metrikkugeln-enthalten-unterraeme} - In $(\K^∞,d)$ gilt: + In $(\K^\infty ,d)$ gilt: \begin{enumerate} \item - $B_1(0) = \K^∞$ + $B_1(0) = \K^\infty $ \item - Betrachte den linearen Unterraum $M_{n_0} := \{ x = (ξ_n)_{n ∈ ℕ}$ mit $ξ_n = 0$ für $n = 1,…,n_0 \}$. + Betrachte den linearen Unterraum $M_{n_0} \coloneq \{ x = (ξ_n)_{n ∈ ℕ}$ mit $ξ_n = 0$ für $n = 1,…,n_0 \}$. Dann gibt es für jeden Radius $r > 0$ ein $n_0 ∈ ℕ$, so dass $M_{n_0} ⊂ B_{r}(0)$. Das heißt, jede noch so kleine Metrikkugel enthält einen nichttrivialen Unterraum. \end{enumerate} @@ -1330,20 +1335,20 @@ Wir werden die unten angegebenen Beispiele auch gleich auf Vollständigkeit unte Das ist trivial. \item Sei $r > 0$ gegeben. - Wähle nun $n_0$, so dass $\sum_{n=n_0+1}^∞ 2^{-n} < r$. + Wähle nun $n_0$, so dass $\sum_{n=n_0+1}^\infty 2^{-n} < r$. Dann gilt \[ ∀ x ∈ M_{n_0}: d(x,0) = \sum_{n ∈ ℕ} 2^{-n} \frac{p_n(x)}{1+p_n(x)} = - \sum_{n=n_0}^∞ 2^{-n} \frac{p_n(x)}{1+p_n(x)} \le - \sum_{n=n_0}^∞ 2^{-n} < r. + \sum_{n=n_0}^\infty 2^{-n} \frac{p_n(x)}{1+p_n(x)} \le + \sum_{n=n_0}^\infty 2^{-n} < r. \] \end{enumerate} \end{proof} -Wäre nun die Topologie auf $(\K^∞,d)$ nun auch von einer Norm erzeugt, dann wären die Normkugeln +Wäre nun die Topologie auf $(\K^\infty ,d)$ nun auch von einer Norm erzeugt, dann wären die Normkugeln \[ - B_r^{\norm\cdot}(0) = \{ x ∈ \K^∞: \norm x < \tilde r \} + B_r^{\norm\cdot}(0) = \{ x ∈ \K^\infty : \norm x < \tilde r \} \] auch eine Umgebungsbasis der Null. Das heißt insbesondere würden wir zu jedem $\tilde r$ ein $r$ finden, so dass $0 ∈ B_r^d(0) ⊂ B_r^{\norm\cdot} (0)$. @@ -1352,19 +1357,19 @@ Mit \cref{lemma-s-metrikkugeln-enthalten-unterraeme} folgt also M_{n_0} ⊂ B_r(0) ⊂ B_r^{\norm\cdot}(0) \] für ein geeignetes $n_0$. -Sei nun ein $0 \ne x ∈ M_{n_0}$. Dann ist, da $M_{n_0}$ ein Unterraum ist, auch $αx ∈ M_{n_0}$ für alle $α ∈ \K$. +Sei nun ein $0 \ne x ∈ M_{n_0}$. Dann ist, da $M_{n_0}$ ein Unterraum ist, auch $\alpha x ∈ M_{n_0}$ für alle $\alpha ∈ \K$. Das heißt, \[ - |α| \cdot \norm x = \norm{αx} < \tilde r \text{ für alle } α ∈ \K, + |\alpha | \cdot \norm x = \norm{\alpha x} < \tilde r \text{ für alle } \alpha ∈ \K, \] -was bereits $α = 0$ impliziert. Das ist ein Widerspruch. +was bereits $\alpha = 0$ impliziert. Das ist ein Widerspruch. \begin{beispiel-nn}[Räume beschränkter Funktionen] - Sei $S$ eine beliebige Menge und $B(S) := \{ f: S → \K, f(s)$ ist beschränkt $\}$. + Sei $S$ eine beliebige Menge und $B(S) \coloneq \{ f: S → \K, f(s)$ ist beschränkt $\}$. Dann wird $B(S)$ mit \[ - \norm f _{B(S)} := \sup_{x ∈ S} |f(x)| < ∞, + \norm f _{B(S)} \coloneq \sup_{x ∈ S} |f(x)| < \infty , \] der $\sup$-Norm, zu einem Banachraum. Dabei ist offensichtlich, dass $\norm\cdot_{B(S)}$ tatsächlich eine Norm ist, und wir werden in einer Übung zeigen, dass die induzierte Metrik tatsächlich vollständig ist. @@ -1393,14 +1398,14 @@ was bereits $α = 0$ impliziert. Das ist ein Widerspruch. \] ein normierter Raum mit \[ - \norm{f}_{C(K)} = \norm{f}_{∞} = \max_{t ∈ K} |f(t)|, + \norm{f}_{C(K)} = \norm{f}_{\infty } = \max_{t ∈ K} |f(t)|, \] der Maximumsnorm. Dieses Maximum wird tatsächlich immer angenommen, da $K$ kompakt ist (Satz von Minimum und Maximum). Insbesondere sind alle stetigen Funktionen auf $K$ beschränkt. Damit gilt offensichtlich $C(K) ⊂ B(K)$ und $\norm{f}_{C(K)} = \norm{f}_{B(K)}$ für alle $f ∈ C(K)$. Da jede stetige Funktion auf kompakten Teilmengen von metrischen Räumen auch gleichmäßig stetig ist, das heißt \[ - ∀ ε > 0 ∃ δ > 0: \left( |t_1-t_2| < δ \implies |f(t_1)-f(t_2)| < ε \right) ∀ t_1,t_2 ∈ K + ∀ \epsilon > 0 ∃ \delta > 0: \left( |t_1-t_2| < \delta \implies |f(t_1)-f(t_2)| < \epsilon \right) ∀ t_1,t_2 ∈ K \] \end{beispiel-nn} @@ -1409,19 +1414,19 @@ was bereits $α = 0$ impliziert. Das ist ein Widerspruch. \end{lemma} \begin{proof} Sei $(f_i)_{i ∈ ℕ}$ eine konvergente (in $(B(K),\norm\cdot_{B(K)})$) Folge in $C(K)$. - Dann existiert ein $f ∈ B(K)$ mit $f_i \xrightarrow[i → ∞]{\norm{\cdot}_{B(K)}} f$. + Dann existiert ein $f ∈ B(K)$ mit $f_i \xrightarrow[i → \infty ]{\norm{\cdot}_{B(K)}} f$. Wir müssen zeigen, dass $f$ bereits stetig ist. Für beliebige $t₁, t_2 ∈ K$ gilt \[ - |f(t_1)-f(t_2) | \le \underbrace{|f_i(t_1)-f_i(t_2)|}_{< ε/3 \text{ für } |t_1-t_2| < δ^{(i)}(ε)} + 2 \underbrace{\norm{f_i - f}_{B(K)}}_{< ε/3 \text{ für } i > i_0} < ε. + |f(t_1)-f(t_2) | \le \underbrace{|f_i(t_1)-f_i(t_2)|}_{< \epsilon /3 \text{ für } |t_1-t_2| < \delta ^{(i)}(\epsilon )} + 2 \underbrace{\norm{f_i - f}_{B(K)}}_{< \epsilon /3 \text{ für } i > i_0} < \epsilon . \] Damit ist $f$ auch gleichmäßig stetig, also insbesondere auch stetig und in $C(K)$. \end{proof} -Das heißt, die Stetigkeit der Folgenglieder $(f_i)_{i ∈ ℕ} ⊂ C(K)$ überträgt sich auf die Grenzfunktion und Konvergenz in $(C(K),\norm\cdot_{∞})$ ist „gleichmäßig auf $K$“. -Wegen dieser Eigenschaft ist die Maximumsnorm $\norm\cdot_∞$ auch die natürliche Norm auf $C(K)$. +Das heißt, die Stetigkeit der Folgenglieder $(f_i)_{i ∈ ℕ} ⊂ C(K)$ überträgt sich auf die Grenzfunktion und Konvergenz in $(C(K),\norm\cdot_{\infty })$ ist „gleichmäßig auf $K$“. +Wegen dieser Eigenschaft ist die Maximumsnorm $\norm\cdot_\infty $ auch die natürliche Norm auf $C(K)$. Andere mögliche Normen (und damit andere Topologien) auf $C(K)$ wären z.B. \[ - \norm{f}_p = \left( \int_K |f(t)|^p dt \right)^{1/p}, \quad 1 \le p < ∞. + \norm{f}_p = \left( \int_K |f(t)|^p dt \right)^{1/p}, \quad 1 \le p < \infty . \] Allerdings ist die Konvergenz in dieser Topologie impliziert keine Stetigkeit für die Grenzfunktion. @@ -1429,7 +1434,7 @@ Allerdings ist die Konvergenz in dieser Topologie impliziert keine Stetigkeit f \begin{beispiel-nn} Sei $\Omega ⊂ ℝ^n$ offen und analog \[ - C(\Omega) := \{ f: \Omega → \K, f \text { stetig }\}. + C(\Omega) \coloneq \{ f: \Omega → \K, f \text { stetig }\}. \] Hier können Funktionen aber auch unbeschränkt sein. Also braucht $\sup |f|$ nicht mehr zu existieren. \end{beispiel-nn} @@ -1447,7 +1452,7 @@ Man nehme z.B. \[ K_m = \{ x ∈ \Omega ⊂ ℝ^n: \norm{x} \le m, \operatorname{dist}(x,∂\Omega) \ge 1/m\}, \] -wobei $\operatorname{dist}(x,∂\Omega) := \inf\{ \norm{x-y}: y ∈ ∂\Omega\}$ und $∂M = \cl \Omega \setminus \Omega$. +wobei $\operatorname{dist}(x,∂\Omega) \coloneq \inf\{ \norm{x-y}: y ∈ ∂\Omega\}$ und $∂M = \cl \Omega \setminus \Omega$. Dann ist $C(\Omega)$ mit der Metrik \[ @@ -1460,8 +1465,8 @@ ein Fréchetraum, also ein metrisierbarer linearer Raum nach \cref{satz-abzaehlb Es gilt in diesem Raum \[ - d(f_i,f) \xrightarrow[i → ∞]{} 0 \gdw - \norm{f_i-f}_{C(K_m)} \xrightarrow[i → ∞]{} ∀m ∈ ℕ, + d(f_i,f) \xrightarrow[i → \infty ]{} 0 \gdw + \norm{f_i-f}_{C(K_m)} \xrightarrow[i → \infty ]{} ∀m ∈ ℕ, \] was ja gerade gleichmäßige Konvergenz auf jeder Kompakten Menge $K ⊂ \Omega$ bedeutet. Damit ist Stetigkeit der Folgenglieder $(f_i)_{i ∈ ℕ} ⊂ C(\Omega)$ impliziert Stetigkeit der Grenzfunktion $f ∈ C(\Omega)$, da Stetigkeit nur eine lokale Eigenschaft ist. @@ -1471,24 +1476,24 @@ Wir werden in der Übung sehen, dass $C(\Omega)$ mit dieser Metrik $d_{C(\Omega) \begin{beispiel-nn}[Räume differenzierbarer Funktionen] \begin{enumerate} \item - Betrachte die Menge $C^\ell(K) = \{ f: K → ℝ, D^α f$ existiert und ist stetig für$|α| < \ell \}$ der $\ell$-mal stetig differenzierbaren Funtktionen auf einer kompakten Menge $K ⊂ ℝ^n$ mit $\ell ∈ ℕ_0$ - Dabei ist $α = (α_1,…,α_n) ∈ ℕ_0^n$ ein Multiindex, $|α| = \sum_{i=1}^n α_i$ und + Betrachte die Menge $C^\ell(K) = \{ f: K → ℝ, D^\alpha f$ existiert und ist stetig für$|\alpha | < \ell \}$ der $\ell$-mal stetig differenzierbaren Funtktionen auf einer kompakten Menge $K ⊂ ℝ^n$ mit $\ell ∈ ℕ_0$ + Dabei ist $\alpha = (\alpha _1,…,\alpha _n) ∈ ℕ_0^n$ ein Multiindex, $|\alpha | = \sum_{i=1}^n \alpha _i$ und \[ - D^α f = \frac{∂^{|α|} f}{∂x_1^{α_1}\cdots∂x_n^{α_n}}. + D^\alpha f = \frac{∂^{|\alpha |} f}{∂x_1^{\alpha _1}\cdots∂x_n^{\alpha _n}}. \] Dann wird $C^\ell(K)$ mit der Norm \[ - \norm{f}_{C^\ell(K)} = \max_{|α| \le l} \max_{x ∈ K} | D^α f(x)| + \norm{f}_{C^\ell(K)} = \max_{|\alpha | \le l} \max_{x ∈ K} | D^\alpha f(x)| \] zu einem Banachraum. Die meisten Eigenschaften sind klar, die Vollständigkeit folgt unmittelbar aus der Vollständigkeit von $C(K)$ Konvergenz in $C^\ell(K)$ bedeutet gerade gleichmäßige Konvergenz aller partiellen Ableitungen bis zur Ordnung $\ell$ auf ganz $K$. \item Sei $\Omega ⊂ ℝ^n$ offen und - $\C^\ell(\Omega) = \{ f: \Omega → ℝ, D^α f$ existiert und ist stetig für$|α| < \ell \}$ + $\C^\ell(\Omega) = \{ f: \Omega → ℝ, D^\alpha f$ existiert und ist stetig für$|\alpha | < \ell \}$ der Raum der $\ell$-mal stetig differenzierbaren Funtktionen auf $\Omega$ mit $\ell ∈ ℕ_0$. $C^\ell(\Omega)$ wird mit der Metrik \[ - d(f,g) := \sum_{m ∈ ℕ} 2^{-m} \frac{p_{m,l}(f-g)}{1+p_{m,l}(f-g)}, \quad p_{m,l}(f) = \max_{|α| \le \ell} \norm{D^α f}_{C(K_m)}, + d(f,g) \coloneq \sum_{m ∈ ℕ} 2^{-m} \frac{p_{m,l}(f-g)}{1+p_{m,l}(f-g)}, \quad p_{m,l}(f) = \max_{|\alpha | \le \ell} \norm{D^\alpha f}_{C(K_m)}, \] wobei die $K_m$ Ausschöpfungen von $\Omega$ mit kompakten Mengen sind, zu einem Fréchetraum. Konvergenz in $C^\ell(\Omega)$ bedeutet gerade gleichmäßige Konvergenz aller partiellen Ableitungen bis zur Ordnung $\ell$ auf jedem Kompaktum, das in $\Omega$ enthalten ist. @@ -1497,10 +1502,10 @@ Wir werden in der Übung sehen, dass $C(\Omega)$ mit dieser Metrik $d_{C(\Omega) Wir betrachten nun einige Unterräume von $\C^\ell(\Omega)$: \begin{enumerate}[label=(\roman*)] \item - $\C^\ell_B(\Omega) = \{ f: \Omega → ℝ, D^α f$ existiert, ist beschränkt und ist stetig für$|α| < \ell \}$ + $\C^\ell_B(\Omega) = \{ f: \Omega → ℝ, D^\alpha f$ existiert, ist beschränkt und ist stetig für$|\alpha | < \ell \}$ wird zum normierten Raum mit \[ - \norm{f}_{C^\ell_B(\Omega)} = \max_{|α| \le l} \sup_{x ∈ \Omega} | D^α f(x)| + \norm{f}_{C^\ell_B(\Omega)} = \max_{|\alpha | \le l} \sup_{x ∈ \Omega} | D^\alpha f(x)| \] Zwar gilt $C^\ell_B(\Omega) ⊂ C^\ell(\Omega)$ (als Mengen), jedoch besitzt $C^\ell_B(\Omega)$ nicht die Relativtopologie von $\C^\ell(\Omega)$, wie wir in einer Übung sehen werden. \begin{definition} @@ -1508,7 +1513,7 @@ Wir werden in der Übung sehen, dass $C(\Omega)$ mit dieser Metrik $d_{C(\Omega) \item Für $\Omega ⊂ ℝ^n$ offen und $f: \Omega → ℝ$ heißt \[ - \supp f := \cl{ \{ x ∈ \Omega, f(x) \ne 0 \}} + \supp f \coloneq \cl{ \{ x ∈ \Omega, f(x) \ne 0 \}} \] der \emph{Träger} oder \emph{Support} von $f$. \item @@ -1533,16 +1538,16 @@ Wir werden in der Übung sehen, dass $C(\Omega)$ mit dieser Metrik $d_{C(\Omega) \end{enumerate} \item Sei $\Omega ⊂ ℝ^n$ offen und - $C^∞(\Omega) = \{ f: \Omega → ℝ, D^αf $ existiert und ist stetig für alle $α ∈ ℕ_0^n \} = \bigcap_{\ell ∈ ℕ}C^\ell(\Omega)$. + $C^\infty (\Omega) = \{ f: \Omega → ℝ, D^\alpha f $ existiert und ist stetig für alle $\alpha ∈ ℕ_0^n \} = \bigcap_{\ell ∈ ℕ}C^\ell(\Omega)$. Wir definieren die Topologie wieder über eine Metrik durch Seminormen \[ - d(f,g) := \sum_{m ∈ ℕ} 2^{-m} \frac{p_{m}(f-g)}{1+p_{m}(f-g)}, \quad p_{m}(f) = \max_{|α| \le m} \norm{D^α f}_{C(K_m)}. + d(f,g) \coloneq \sum_{m ∈ ℕ} 2^{-m} \frac{p_{m}(f-g)}{1+p_{m}(f-g)}, \quad p_{m}(f) = \max_{|\alpha | \le m} \norm{D^\alpha f}_{C(K_m)}. \] Mit dieser Metrik wird $C^\ell(\Omega)$ zum Fréchetraum. - Konvergenz in $C^∞(\Omega)$ bedeutet gerade gleichmäßige Konvergenz aller partiellen Ableitungen auf jedem Kompaktum, das in $\Omega$ enthalten ist. + Konvergenz in $C^\infty (\Omega)$ bedeutet gerade gleichmäßige Konvergenz aller partiellen Ableitungen auf jedem Kompaktum, das in $\Omega$ enthalten ist. Auch dieser Raum ist nicht normierbar mit einem analogem Argument wie bei den stetigen Funktionen auf $\Omega$. \item - Sei $\Omega ⊂ ℝ^n$ offen und $C_0^∞(\Omega) = \{ f ∈ C^∞(\Omega) : \supp f ⊂⊂ M \}$ der \emph{Raum der Testfunktionen}. + Sei $\Omega ⊂ ℝ^n$ offen und $C_0^\infty (\Omega) = \{ f ∈ C^\infty (\Omega) : \supp f ⊂⊂ M \}$ der \emph{Raum der Testfunktionen}. Ein Beispiel für so eine Funktion ist \[ f(x) = @@ -1552,40 +1557,40 @@ Wir werden in der Übung sehen, dass $C(\Omega)$ mit dieser Metrik $d_{C(\Omega) \end{cases}, \] wobei $\Omega = B^{\norm\cdot_\infty}_2(0)$, $|\cdot| = \norm\cdot_2$ und $c ∈ ℝ$ konstant. - Offensichtlich ist $C_0^∞(\Omega) ⊂ C^∞(\Omega)$. - Wenn man auf $C_0^∞(\Omega)$ jedoch die Spurtopologie wählt, bekommt man später Probleme (bestimmte Funktionale auf $C_0^∞(\Omega)$ sind nicht mehr stetig, wie wir in einer Übungsaufgabe sehen werden. - Man nennt Funktionale auf $C_0^∞(\Omega)$ auch Distributionen). - Außerdem wäre der $C_0^∞(\Omega)$ mit dieser Metrik nicht vollständig -- der Träger der Grenzfunktion muss nicht mehr beschränkt sein. + Offensichtlich ist $C_0^\infty (\Omega) ⊂ C^\infty (\Omega)$. + Wenn man auf $C_0^\infty (\Omega)$ jedoch die Spurtopologie wählt, bekommt man später Probleme (bestimmte Funktionale auf $C_0^\infty (\Omega)$ sind nicht mehr stetig, wie wir in einer Übungsaufgabe sehen werden. + Man nennt Funktionale auf $C_0^\infty (\Omega)$ auch Distributionen). + Außerdem wäre der $C_0^\infty (\Omega)$ mit dieser Metrik nicht vollständig -- der Träger der Grenzfunktion muss nicht mehr beschränkt sein. \begin{definition-nn} Sei $M ⊂ X$ und $X$ ein linearer Raum. Dann heißt \[ - \conv (M) := \{ x: ∃α_i > 0, x_i ∈ M, i ∈ \{1,…,k\}: \sum_{i=1}^k α_i = 1, \sum_{i=1}^k α_i x_i = x \} + \conv (M) \coloneq \{ x: ∃\alpha _i > 0, x_i ∈ M, i ∈ \{1,…,k\}: \sum_{i=1}^k \alpha _i = 1, \sum_{i=1}^k \alpha _i x_i = x \} \] die \emph{konvexe Hülle} von $M$. \end{definition-nn} - Aus Gründen, die erst später zu verstehen sind, wählt man auf $C^∞_0(\Omega)$ folgende lokalkonvxe Topologie: + Aus Gründen, die erst später zu verstehen sind, wählt man auf $C^\infty _0(\Omega)$ folgende lokalkonvxe Topologie: Setze \[ - p(\xi) := \sum_{k ∈ ℕ} 2^{-k} \frac{\norm \xi _{C^k(\Omega)}}{1 + \norm \xi _{C^k(\Omega)}}, \quad \xi ∈ C_0^∞(\Omega) + p(\xi) \coloneq \sum_{k ∈ ℕ} 2^{-k} \frac{\norm \xi _{C^k(\Omega)}}{1 + \norm \xi _{C^k(\Omega)}}, \quad \xi ∈ C_0^\infty (\Omega) \] Sei $(D_j)_{j ∈ ℕ}$ eine Ausschöpfung von $\Omega$ mit offenen Mengen, also $D_j ⊂ D_{j+1}, D_j ⊂⊂ \Omega, \bigcup_{j ∈ ℕ} D_j = \Omega$. Eine mögliche Wahl wäre beispielsweise $D_j = K_j^\circ$, wobei die $K_j$ wie oben sind. - Für $ε = (ε_j)_{j ∈ ℕ} ∈ ℝ^∞, ε_j > 0$ für alle $ℕ$ definieren wir eine Umgebungsbasis der $0 ∈ C_0^∞(\Omega)$ durch alle Mengen + Für $\epsilon = (\epsilon _j)_{j ∈ ℕ} ∈ ℝ^\infty , \epsilon _j > 0$ für alle $ℕ$ definieren wir eine Umgebungsbasis der $0 ∈ C_0^\infty (\Omega)$ durch alle Mengen \[ - U_ε := \conv \left[ \bigcup_{j ∈ ℕ} \{ \xi ∈ C^∞_0 : p(\xi) < ε_j \} \right] ⊂ C_0^∞(\Omega). + U_\epsilon \coloneq \conv \left[ \bigcup_{j ∈ ℕ} \{ \xi ∈ C^\infty _0 : p(\xi) < \epsilon _j \} \right] ⊂ C_0^\infty (\Omega). \] - mit $ε = (ε_j)_{j ∈ ℕ} ∈ ℝ^∞, ε_j > 0$ und endliche Schnitte davon. Andere Umgebungen umgeben sich durch Translation. - Die so definierte Topologie nennen wir $\T_\D$ und den Raum $C_1^∞(\Omega)$ auch $\D(\Omega)$. + mit $\epsilon = (\epsilon _j)_{j ∈ ℕ} ∈ ℝ^\infty , \epsilon _j > 0$ und endliche Schnitte davon. Andere Umgebungen umgeben sich durch Translation. + Die so definierte Topologie nennen wir $\T_\D$ und den Raum $C_1^\infty (\Omega)$ auch $\D(\Omega)$. Es stellt sich heraus, dass diese Topologie tatsächlich unabhängig von der gewählten Ausschöpfung ist. Außerdem ist $(\D(\Omega),\T_\D)$ ein topologischer linearer Raum, das heißt, die Vektorraumoperationen sind stetig. \begin{lemma}[Charakterisierung offener Mengen in $\D(\Omega)$] Es gilt \[ - O ∈ \T_\D \iff ∀ ξ ∈ O ∃ ε=(e_j)_{j ∈ ℕ} ∈ ℝ^∞, e_j > 0: e+U_ε ⊂ O. + O ∈ \T_\D \iff ∀ ξ ∈ O ∃ \epsilon =(e_j)_{j ∈ ℕ} ∈ ℝ^\infty , e_j > 0: e+U_\epsilon ⊂ O. \] Das heißt, die Topologie $\T_\D$ und die Topologie \[ - \tilde T_\D = \{ O ⊂ C_0^∞(\Omega): ∀ ξ ∈ O ∃ ε = (ε_j)_{j ∈ ℕ} ∈ ℝ^∞, ε_j > 0: ε+ U_ε ⊂ O \} + \tilde T_\D = \{ O ⊂ C_0^\infty (\Omega): ∀ ξ ∈ O ∃ \epsilon = (\epsilon _j)_{j ∈ ℕ} ∈ ℝ^\infty , \epsilon _j > 0: \epsilon + U_\epsilon ⊂ O \} \] sind gleich. \end{lemma} @@ -1593,53 +1598,53 @@ Wir werden in der Übung sehen, dass $C(\Omega)$ mit dieser Metrik $d_{C(\Omega) Übung. \end{proof} \begin{korollar} - Die Mengen $U_∈$ sind bereits eine Umgebungsbasis der Null. + Die Mengen $U_\epsilon$ sind bereits eine Umgebungsbasis der Null. Nach Definition sind sie aber auch Konvex, das heißt $(\D(\Omega),\T_\D)$ ist ein lokalkonvexer Hausdorff-Raum. \end{korollar} \begin{satz} - $ξ_m \xrightarrow[m → ∞]{} 0 \gdw$ + $ξ_m \xrightarrow[m → \infty ]{} 0 \gdw$ \[ \begin{cases} (i), & \text{Es existiert $D$ offen mit $D ⊂⊂ \Omega$ und - $ξ_m ∈ C_0^∞(D)$ für alle $m ∈ ℕ$} \\ + $ξ_m ∈ C_0^\infty (D)$ für alle $m ∈ ℕ$} \\ (ii), & \text{Für jedes $k ∈ ℕ$ gilt: - $\norm{ξ_m}_{C^k(\cl{\Omega})} \xrightarrow[m → ∞]{} 0$} + $\norm{ξ_m}_{C^k(\cl{\Omega})} \xrightarrow[m → \infty ]{} 0$} \end{cases} \] \end{satz} \begin{proof} - Zeige nur „$\Longleftarrow$“. Sei dazu $(ξ_m)_{m ∈ ℕ}$ eine Folge mit (i) und (ii). + Zeige nur „$\Leftarrow$“. Sei dazu $(ξ_m)_{m ∈ ℕ}$ eine Folge mit (i) und (ii). Wähle nun $D_j$ von oben mit $D ⊂ D_j$ ($j$ ist fest). - Sei nun $ε=(ε_i)_{i ∈ ℕ}, ε_i > 0$ gegeben. Dann müssen wir zeigen, dass für alle $m > m_0$ schon $ξ_m ∈ U_ε$ gilt. - Zunächst sind nach (i) $ξ_m ∈ C^∞_0(D_j)$ . + Sei nun $\epsilon =(\epsilon _i)_{i ∈ ℕ}, \epsilon _i > 0$ gegeben. Dann müssen wir zeigen, dass für alle $m > m_0$ schon $ξ_m ∈ U_\epsilon $ gilt. + Zunächst sind nach (i) $ξ_m ∈ C^\infty _0(D_j)$ . Außerdem gilt \[ - p(\xi_m) \le \underbrace{\sum_{k=1}^N 2^{-k} \frac{ \norm{\xi_m}_{C^k(\cl \Omega)} } {1+ \norm{\xi_m}_{C^k(\cl \Omega)} }}_{\text{wegen (i)} < ε_j/2 \text{ für $m \ge m_0(ε_j,N)$}} + \underbrace{\sum_{k=N+1} 2^{-k}}_{<ε_j/2 \text{ für $n$ groß genug}} < ε_j. + p(\xi_m) \le \underbrace{\sum_{k=1}^N 2^{-k} \frac{ \norm{\xi_m}_{C^k(\cl \Omega)} } {1+ \norm{\xi_m}_{C^k(\cl \Omega)} }}_{\text{wegen (i)} < \epsilon _j/2 \text{ für $m \ge m_0(\epsilon _j,N)$}} + \underbrace{\sum_{k=N+1} 2^{-k}}_{<\epsilon _j/2 \text{ für $n$ groß genug}} < \epsilon _j. \] \end{proof} \item Betrachten wir nun Lebesgue-integrierbare Funktionen. - Bereits eingeführt wurden die Räume $\L^p(\Omega)$ und $L^p(\Omega)$, $0 < p < ∞$, wobei $\Omega ⊂ ℝ^n$ offen. - Diese sind für $1 \le p < ∞$ normiert, und für $0 < p < 1$ quasi-normiert. - Für $p = ∞$ setzen wir + Bereits eingeführt wurden die Räume $\L^p(\Omega)$ und $L^p(\Omega)$, $0 < p < \infty $, wobei $\Omega ⊂ ℝ^n$ offen. + Diese sind für $1 \le p < \infty $ normiert, und für $0 < p < 1$ quasi-normiert. + Für $p = \infty $ setzen wir \[ - \L^∞(\Omega) := \{ f: \Omega → ℝ ∪ \{ -∞, ∞ \}, f \text{ messbar und fast überall beschränkt} \}. + \L^\infty (\Omega) \coloneq \{ f: \Omega → ℝ ∪ \{ -\infty , \infty \}, f \text{ messbar und fast überall beschränkt} \}. \] Damit haben wir offenbar \[ - C(\Omega) ∩ B(\Omega) ⊂ \L^∞(\omega). + C(\Omega) ∩ B(\Omega) ⊂ \L^\infty (\omega). \] Sei \[ - \norm f _{\L^∞(\Omega)} := \supess_{t ∈ \Omega} |f(t)| := \inf_{M ⊂ \Omega \text{ NM}} \sup_{t ∈ \Omega \setminus M} |f(t)|. + \norm f _{\L^\infty (\Omega)} \coloneq \supess_{t ∈ \Omega} |f(t)| \coloneq \inf_{M ⊂ \Omega \text{ NM}} \sup_{t ∈ \Omega \setminus M} |f(t)|. \] - Dann gilt für $f ∈ \L^∞(\Omega)$ + Dann gilt für $f ∈ \L^\infty (\Omega)$ \[ \norm f = 0 \gdw f = 0 \text{ fast überall} \] - Mit $N := \{ f ∈ \L^∞(\Omega) : \norm f = 0 \}$ wird + Mit $N \coloneq \{ f ∈ \L^\infty (\Omega) : \norm f = 0 \}$ wird \[ - L^∞(\Omega) := \left( \L^∞(\Omega)/N, \norm\cdot_{L^∞(\Omega)} \right) + L^\infty (\Omega) \coloneq \left( \L^\infty (\Omega)/N, \norm\cdot_{L^\infty (\Omega)} \right) \] zu einem normiertem Raum. \end{enumerate} @@ -1648,21 +1653,21 @@ Wir werden in der Übung sehen, dass $C(\Omega)$ mit dieser Metrik $d_{C(\Omega) { \LARGE Vorlesung vom Donnerstag, 9. November fehlt (genauso wie vermutlich alle weiteren Donnerstagsvorlesungen ab jetzt)} -Seien $f_n → f ∈ L^p(\Omega), h ∈ C_0^∞(\Omega)$. +Seien $f_n → f ∈ L^p(\Omega), h ∈ C_0^\infty (\Omega)$. Dann \[ - \lim_{n → ∞} ∫_Ω f_n(t) h(t) dt = ∫_Ω f(t) h(t) dt, + \lim_{n → \infty } ∫_\Omega f_n(t) h(t) dt = ∫_\Omega f(t) h(t) dt, \] denn \begin{align*} - ∫_Ω (f_n(t) - f(t)) h(t) dt &\le ∫_{\supp h} M |f_n(t) - f(t)| dt \\ + ∫_\Omega (f_n(t) - f(t)) h(t) dt &\le ∫_{\supp h} M |f_n(t) - f(t)| dt \\ & \stackrel{\mathclap{\text{Hölder}}}{\le} \; M [ \supp(h)]^{1/q} \norm{f_n-f}_{L^p(\Omega)} → 0. \end{align*} \section{Beschränkte und kompakte Mengen in metrischen linearen Räumen} Wir wissen bereits nach dem Satz von Heine-Borel, dass eine Teilmenge $K ⊂ ℝ^n$ genau dann kompakt ist, wenn sie abgeschlossen und beschränkt ist. -Beschränktheit bedeutet hier in einer (beliebigen, da alle äquivalent) Norm. +Beschränktheit bedeutet hier Beschränktheit in einer (beliebigen, da alle äquivalent) Norm. Nun wollen wir so ein Konzept für Beschränktheit auch in allgemeinen metrischen (topologischen) linearen Räumen finden. @@ -1673,12 +1678,12 @@ Nun wollen wir so ein Konzept für Beschränktheit auch in allgemeinen metrische \item In einigen metrischen Räumen gilt ohnehin $d(x,0) \le 1$ für alle $x ∈ X$. \item - Ist $d$ eine Metrik auf $X$. Dann ist $\tilde d := \frac d {1+d} \le 1$ eine zu $d$ äquivalente Metrik auf $X$, wie wir in Topologie gesehen haben. + Ist $d$ eine Metrik auf $X$. Dann ist $\tilde d \coloneq \frac d {1+d} \le 1$ eine zu $d$ äquivalente Metrik auf $X$, wie wir in Topologie gesehen haben. \end{enumerate} \end{problem-nn} \begin{definition} - Sei $(X,\T)$ ein topologischer linearer Raum, $B ⊂ X$ heißt \emph{beschränkt}, falls zu jeder offenen Umgebung $U$ von $0 ∈ X$ ein $α > 0$ existiert, so dass $B ⊂ αU = \{αu: u ∈ U\}$, das heißt jede Nullumgebung lässt sich so weit „aufblasen“, dass sie $B$ überdeckt. + Sei $(X,\T)$ ein topologischer linearer Raum, $B ⊂ X$ heißt \emph{beschränkt}, falls zu jeder offenen Umgebung $U$ von $0 ∈ X$ ein $\alpha > 0$ existiert, so dass $B ⊂ \alpha U = \{\alpha u: u ∈ U\}$, das heißt jede Nullumgebung lässt sich so weit „aufblasen“, dass sie $B$ überdeckt. \end{definition} \begin{bemerkung-nn} @@ -1692,33 +1697,33 @@ Nun wollen wir so ein Konzept für Beschränktheit auch in allgemeinen metrische \end{satz} \begin{proof} „⇒“: Sei $k ∈ ℕ$ gegeben. - Setze $r_k := \frac 1 {2^{k+1}}$ und $U := B_{r_k}(0)$. - Da $B$ beschränkt ist, gibt es $α = α_k > 0$, dass + Setze $r_k \coloneq \frac 1 {2^{k+1}}$ und $U \coloneq B_{r_k}(0)$. + Da $B$ beschränkt ist, gibt es $\alpha = \alpha _k > 0$, dass \begin{align*} - & B ⊂ αU = α B_{r_k}(0) \\ - \iff & α^{-1} B ⊂ B_{r_k} (0) \\ - \iff d(α^{-1} x, 0) < r_k ∀ x ∈ B + & B ⊂ \alpha U = \alpha B_{r_k}(0) \\ + \gdw & \alpha ^{-1} B ⊂ B_{r_k} (0) \\ + \gdw &d(\alpha ^{-1} x, 0) < r_k ∀ x ∈ B \end{align*} - Dann gilt schon $p_k(x) \le M_k := α_k$ für alle $x ∈ B$, denn + Dann gilt schon $p_k(x) \le M_k \coloneq \alpha _k$ für alle $x ∈ B$, denn \[ - \frac 1 {2^{k+1}} = r_k > d(α_k^{-1} x, 0 - \ge 2^k \frac {p_k(α_k^{-1}x)}{1+p_k(α_k^{-1} x)} - = 2^{-k} \frac{α_k^{-1} p_k(x)}{1+α_k^{-1} p_k(x)}. + \frac 1 {2^{k+1}} = r_k > d(\alpha _k^{-1} x, 0 + \ge 2^k \frac {p_k(\alpha _k^{-1}x)}{1+p_k(\alpha _k^{-1} x)} + = 2^{-k} \frac{\alpha _k^{-1} p_k(x)}{1+\alpha _k^{-1} p_k(x)}. \] - Also mit $\eta := α_k^{-1} p_k(x)$ gilt $\frac 1 2 > \frac \eta {1+\eta}$, also $\eta < 1$ oder $p_k(x) \le M_k$ für alle $x ∈ B$. + Also mit $\eta \coloneq \alpha _k^{-1} p_k(x)$ gilt $\frac 1 2 > \frac \eta {1+\eta}$, also $\eta < 1$ oder $p_k(x) \le M_k$ für alle $x ∈ B$. „⇐“: Sei also $p_k(x) \le M_k$ für alle $x ∈ B$ und $k ∈ ℕ$. - Wir müssen nun zeigen, dass es für jedes $r > 0$ ein $α > 0$ gibt mit $B ⊂ αB_r(0)$, also $α^{-1} B ⊂ B_r(0)$. + Wir müssen nun zeigen, dass es für jedes $r > 0$ ein $\alpha > 0$ gibt mit $B ⊂ \alpha B_r(0)$, also $\alpha ^{-1} B ⊂ B_r(0)$. Sei also $r > 0$ gegeben. - Wähle nun $m_0 ∈ ℕ$ mit $\sum_{n=m_0+1}^∞ 2^{-n} < r/2$. - Wähle $α > 0$ mit $\sum_{n=1}^{m_0} 2^{-n} \frac{α^{-1} M_k}{1+α^{-1} M_k} < r/2$. + Wähle nun $m_0 ∈ ℕ$ mit $\sum_{n=m_0+1}^\infty 2^{-n} < r/2$. + Wähle $\alpha > 0$ mit $\sum_{n=1}^{m_0} 2^{-n} \frac{\alpha ^{-1} M_k}{1+\alpha ^{-1} M_k} < r/2$. Dann gilt für alle $x ∈ B$ - \[ - d(α^{-1} x, 0) = - \sum_{n ∈ ℕ} 2^{-n} \frac{α^{-1} p_n(x)}{1+α^{-1} p_n(x)} - \le \sum_{n=1}^{m_0} 2^{-n} \frac{α^{-1} p_n(x)}{1+α^{-1} p_n(x)} + \sum_{n=m_0+1}^∞ 2^{-n} < r/2 + r/2 = r. - \] + \begin{align*} + d(\alpha ^{-1} x, 0) &= + \sum_{n ∈ ℕ} 2^{-n} \frac{\alpha ^{-1} p_n(x)}{1+\alpha ^{-1} p_n(x)} \\ + &\le \sum_{n=1}^{m_0} 2^{-n} \frac{\alpha ^{-1} p_n(x)}{1+\alpha ^{-1} p_n(x)} + \sum_{n=m_0+1}^\infty 2^{-n} < r/2 + r/2 = r. + \end{align*} \end{proof} \begin{korollar} @@ -1733,7 +1738,7 @@ Nun wollen wir so ein Konzept für Beschränktheit auch in allgemeinen metrische \begin{bemerkung} Kugeln $B_r(0)$ in $(X,d)$, wobei $d$ wie in \cref{satz-abzaehlbares-prod-seminormierter-raeume}, sinid also immer unbeschränkt, weil nichttriviale Unterräume $M_{n_0} ⊂ B_r(x)$ existieren. - Insbesondere ist dies gültig in den Räumen $\K^∞, C(Ω), C^\ell(Ω)$ und $C^∞(Ω)$. + Insbesondere ist dies gültig in den Räumen $\K^\infty , C(\Omega), C^\ell(\Omega)$ und $C^\infty (\Omega)$. \end{bemerkung} \begin{definition} @@ -1750,7 +1755,7 @@ Nun wollen wir so ein Konzept für Beschränktheit auch in allgemeinen metrische \end{satz} \begin{beispiel-nn} - Die Räume $\K^∞, C(Ω), C^\ell(Ω)$ und $C^∞(Ω)$ sind nicht lokalbeschränkt, aber lokalkonvex. Somit sind sie auch nicht normierbar. + Die Räume $\K^\infty , C(\Omega), C^\ell(\Omega)$ und $C^\infty (\Omega)$ sind nicht lokalbeschränkt, aber lokalkonvex. Somit sind sie auch nicht normierbar. Auch $L^p(0,1)$ mit $0 < p < 1$ ist nicht lokalkonvex, aber lokalbeschränkt, also nicht normierbar. \end{beispiel-nn} @@ -1772,7 +1777,7 @@ Nun wollen wir so ein Konzept für Beschränktheit auch in allgemeinen metrische \begin{warnung-nn} Metrikkugeln müssen im Allgemeinen nicht kreisförmig sein (obwohl die uns bekannten Kugeln dies sind). - Gegenbeispiel: $X = ℝ$, $d(x,y) := \left| ∫_x^y 1+\ind{ℝ_-}(s)\; ds \right|$. + Gegenbeispiel: $X = ℝ$, $d(x,y) \coloneq \left| ∫_x^y 1+\ind{ℝ_-}(s)\; ds \right|$. \end{warnung-nn} \begin{lemma} @@ -1787,7 +1792,7 @@ Nun wollen wir so ein Konzept für Beschränktheit auch in allgemeinen metrische „⊂“: Sei $x ∈ X$. Setze $β_n = 1/n, n ∈ ℕ$. Dann gilt \[ - β_n x \xrightarrow[n → ∞]{} 0, + β_n x \xrightarrow[n → \infty ]{} 0, \] also $β_n ∈ V$ für $n \ge n_0$. Damit haben wir aber $x ∈ n_0 V$. \end{proof} @@ -1815,7 +1820,7 @@ Nun wollen wir so ein Konzept für Beschränktheit auch in allgemeinen metrische \[ K ⊂ \bigcup_{i=1}^s n_i W \stackrel{(*)}= n_s W, \quad n_1 < n_2 < … < n_s, \] - also folgt die Behauptung mit $α = n_s$. $(*)$ gilt wegen der Kreisförmigkeit und $\left|\frac {n_i}{n_s}\right| \le 1$. + also folgt die Behauptung mit $\alpha = n_s$. $(*)$ gilt wegen der Kreisförmigkeit und $\left|\frac {n_i}{n_s}\right| \le 1$. \end{proof} \begin{bemerkung-nn} Ohne die Hausdorff-Eigenschaft gilt dies nicht. Gegenbeispiel: $X = ℝ$ mit der Klumpentopologie. @@ -1827,7 +1832,7 @@ Nun wollen wir so ein Konzept für Beschränktheit auch in allgemeinen metrische \item In einem topologischen Raum $(X,\T)$ heißt eine Menge $A ⊂ X$ \emph{relativ kompakt}, falls $\cl A$ kompakt ist. \item - In einem metrischen Raum $(X,d)$ heißt eine Menge $A ⊂ X$ \emph{präkompakt}, falls für jedes $ε > 0$ die Menge $A$ von endlich vielen Bällen mit Radius $ε$ überdeckt werden kann. + In einem metrischen Raum $(X,d)$ heißt eine Menge $A ⊂ X$ \emph{präkompakt}, falls für jedes $\epsilon > 0$ die Menge $A$ von endlich vielen Bällen mit Radius $\epsilon $ überdeckt werden kann. \end{enumerate} \end{definition} @@ -1847,8 +1852,8 @@ Nun wollen wir so ein Konzept für Beschränktheit auch in allgemeinen metrische Sei $(x_n)_{n ∈ ℕ} ⊂ A$ eine Cauchy-Folge. Nach (b) besitzt $(x_n)_{n ∈ ℕ}$ einen Häufungspunkt $x^*$. Da $(x_n)_{n ∈ ℕ}$ Cauchy-Folge ist, konvergiert $(x_n)_{n ∈ ℕ}$ schon gegen $x^*$. Damit ist $A$ vollständig. - Angenommen, $A$ wäre nicht präkompakt. Dann gibt es $ε > 0$, so dass $A$ keine endliche Überdeckung mit $ε$-Kugeln besitzt. - Dadurch kann man eine Folge $(x_k)_{k ∈ K}$ definieren, mit $d(x_k,x_j) > ε$ für $k \ne j$. + Angenommen, $A$ wäre nicht präkompakt. Dann gibt es $\epsilon > 0$, so dass $A$ keine endliche Überdeckung mit $\epsilon $-Kugeln besitzt. + Dadurch kann man eine Folge $(x_k)_{k ∈ K}$ definieren, mit $d(x_k,x_j) > \epsilon $ für $k \ne j$. Dann besitzt $(x_k)_{k ∈ K}$ offensichtlich keine Cauchy-Teilfolge, also auch keinen Häufungspunkt. Also $A$ präkompakt. \end{proof} @@ -1867,18 +1872,18 @@ Hier gilt $M = \inf \{ c \ge 0:$ mit $C$ gilt (5) $\}$. $(3) \iff (4)$ klar durch die Charakterisierung von beschränkten Mengen in normierten Räumen und Ausnutzung der Linearität. - $(2) \Rightarrow (4)$. Sei $T$ stetig in $x^*$. Wähle $ε > 0$, so dass $T(\cl B_ε(x^*)) ⊂ B_1(T(x^*))$. + $(2) \Rightarrow (4)$. Sei $T$ stetig in $x^*$. Wähle $\epsilon > 0$, so dass $T(\cl B_\epsilon (x^*)) ⊂ B_1(T(x^*))$. Dann gilt für alle $x ∈ \cl B _1 (0)$ \[ - x^* + ε x ∈ \cl B_ε(x^*) + x^* + \epsilon x ∈ \cl B_\epsilon (x^*) \] - und $T(x^*) + εT(x) = T(x^* + εx) ∈ B_1(T(x^*))$, das heißt $ε T(x) ∈ B_1(0)$ oder $\norm{T(x)}_Y \le \frac 1 {ε} =: M$ + und $T(x^*) + \epsilon T(x) = T(x^* + \epsilon x) ∈ B_1(T(x^*))$, das heißt $\epsilon T(x) ∈ B_1(0)$ oder $\norm{T(x)}_Y \le \frac 1 {\epsilon } =: M$ $(4) \Rightarrow (5)$. Für $x \ne 0$ gilt \[ \norm{T(x)} \le \norm x \norm{T\left( \frac x {\norm x} \right)} \le M \norm x, \] - also gilt die Aussage mit $C := M$. + also gilt die Aussage mit $C \coloneq M$. $(5) \Rightarrow (1)$. Für $x, x_1 ∈ X$ gilt \[ @@ -1916,7 +1921,7 @@ In topologischen linearen Räumen gilt dies jedoch nciht. 3.6.7 \end{satz} \begin{proof} - Nur „$\Leftarrow$“: Nach 6.6 reicht es, Beschränktheit von $T$ zu zeigen, also dass, wenn $B ⊂ X$ beschränkt ist, auch $TN?) ⊂ Y$ beschränkt ist. + Nur „$\Leftarrow$“: Nach 6.6 reicht es, Beschränktheit von $T$ zu zeigen, also dass, wenn $B ⊂ X$ beschränkt ist, auch $T(B) ⊂ Y$ beschränkt ist. $B ⊂ X$ ist genau dann beschränkt, wenn für alle $k ∈ ℕ$ $C_k > 0$ existieren mit $p_k(x) \le C_k$ für alle $x ∈ B$. Nach Voraussetzung ist dann aber auch für alle $x ∈ B$ \[ @@ -1926,8 +1931,8 @@ In topologischen linearen Räumen gilt dies jedoch nciht. \end{proof} \begin{definition} - Seien $X, Y$ topologische lineare Räume. Dann bezeichnet $\L(X, Y) := \{ T: X → Y: T$ linear und stetig $\}$ den \emph{Raum der stetigen (beschränkten) Operatoren}. - Im Spezialfall $Y = \K$ sei $X' := \L(X, \K)$ der \emph{Raum der stetigen Funktionale} oder auch der \emph{Dualraum von $X$}. + Seien $X, Y$ topologische lineare Räume. Dann bezeichnet $\L(X, Y) \coloneq \{ T: X → Y: T$ linear und stetig $\}$ den \emph{Raum der stetigen (beschränkten) Operatoren}. + Im Spezialfall $Y = \K$ sei $X' \coloneq \L(X, \K)$ der \emph{Raum der stetigen Funktionale} oder auch der \emph{Dualraum von $X$}. \end{definition} \begin{bemerkung-nn} \begin{enumerate} @@ -1941,9 +1946,9 @@ In topologischen linearen Räumen gilt dies jedoch nciht. Ist $X$ jedoch normierbar, so folgt aus den Hahn-Banach-Sätzen, dass $X'$ nichttrivial ist. \item Falls $X$ und $Y$ normierte Räume sind, dann wird $\L(X, Y)$ ebenfalls zu einem normierten Raum mit der Operatornorm - \[ - \norm T := \norm T _{\L(X,Y)} := \sup \{\norm x _X \le 1\} \norm {Tx}_Y = \inf \{ C ≥ 0: ∀x ∈ X: \norm {Tx} \le C \norm x \}. - \] + \begin{align*} + \norm T &\coloneq \norm T _{\L(X,Y)} \coloneq \sup \{\norm x _X \le 1\} \norm {Tx}_Y \\ &= \inf \{ C \ge 0: ∀x ∈ X: \norm {Tx} \le C \norm x \}. + \end{align*} Das heißt, wir haben \begin{equation} \label{eq:61} @@ -1970,8 +1975,8 @@ In topologischen linearen Räumen gilt dies jedoch nciht. \begin{proof} Es ist nur noch die Vollständigkeit zu zeigen. Sei dazu $(T_n)_{n ∈ ℕ}$ eine Cauchy-Folge in $\L(X,Y)$. - Das heißt, für jedes $ε > 0$ existiert ein $N_0$ mit $\norm {T_n - T_m} < ε$ für $n, m > N_0$. - Also mit \eqref{eq:61} $\norm {T_n x - T_mx} \le \norm {T_n - T_m} \norm x < ε \norm x$ für alle $x ∈ X$ und $n,m > N_0$. + Das heißt, für jedes $\epsilon > 0$ existiert ein $N_0$ mit $\norm {T_n - T_m} < \epsilon $ für $n, m > N_0$. + Also mit \eqref{eq:61} $\norm {T_n x - T_mx} \le \norm {T_n - T_m} \norm x < \epsilon \norm x$ für alle $x ∈ X$ und $n,m > N_0$. Insbesondere ist $(T_nx)_{n ∈ ℕ}$ eine Cauchy-Folge in $Y$. Da $Y$ vollständig ist, besitzt diese Folge einen Grenzwert $y_x ∈ Y$. Wir definieren eine Abbildung \[ @@ -1980,39 +1985,39 @@ In topologischen linearen Räumen gilt dies jedoch nciht. Dann ist $T$ linear, weil alle $T_n$ linear sind. Also ist nur die Stetigkeit von $T$ und die Konvergenz von $(T_n)_{n ∈ ℕ}$ gegen $T$ zu zeigen. Für die Stetigkeit bekommt man unter Verwendung der Dreicksunglechung direkt \[ - \left| \norm {T_n} - \norm{T_m} \right| \le \norm {T_n - T_m} < ε \quad ∀ n, m ≥ N_0, + \left| \norm {T_n} - \norm{T_m} \right| \le \norm {T_n - T_m} < \epsilon \quad ∀ n, m \ge N_0, \] also eine Cauchyfolge $\left( \norm{T_n} \right)_{n ∈ ℕ}$ in $ℝ$, die wegen der Vollständigkeit von $ℝ$ konvergent, also insbesondere auch beschränkt ist. Damit gibt es $M > 0$ mit $\norm {T_n} \le M$ für alle $n ∈ ℕ$, also mit~\eqref{eq:61} \[ - \norm{Tx} \xleftarrow[n → ∞]{} \norm{T_nx } \le M \norm x, ∀ x ∈ X, + \norm{Tx} \xleftarrow[n → \infty ]{} \norm{T_nx } \le M \norm x, ∀ x ∈ X, \] also die stetigkeit von $T$. Jetzt zur Konvergenz: Für $\norm x \le$ 1 gilt \[ - \norm {T_n x - T_m x } < ε, \quad ∀n, m ≥ N_0, + \norm {T_n x - T_m x } < \epsilon , \quad ∀n, m \ge N_0, \] - also durch Grenzwertbildung $n → ∞$ + also durch Grenzwertbildung $n → \infty $ \[ - \norm {T_n x - T x } < ε, \quad ∀n ≥ N_0, + \norm {T_n x - T x } < \epsilon , \quad ∀n \ge N_0, \] und mit~\eqref{eq:61} \[ - \norm {T_n -T} = \sup_{\norm x \le 1} \norm {T_n x - T_x} < ε, \quad ∀ n ≥ N_0, + \norm {T_n -T} = \sup_{\norm x \le 1} \norm {T_n x - T_x} < \epsilon , \quad ∀ n \ge N_0, \] das heißt $T_n → T$ wie gewünscht. Für den Zusatz haben wir \[ - \norm {S(Tx)} ≤ \norm S \norm {Tx} \le \norm S \norm T \norm x. + \norm {S(Tx)} \le \norm S \norm {Tx} \le \norm S \norm T \norm x. \] - Da das für alle $x ∈ X$ gilt, haben wir $\norm {ST} ≤ \norm S \norm T$. + Da das für alle $x ∈ X$ gilt, haben wir $\norm {ST} \le \norm S \norm T$. \end{proof} \begin{korollar} - Ist $X$ ein Banachraum, dann ist $\L(X) := \L(X,X)$ eine \emph{Banachalgebra}, das heißt ein vollständiger normierter Vektorraum mit einer Multiplikation, so dass für $T, S ∈ \L(X)$ gilt: + Ist $X$ ein Banachraum, dann ist $\L(X) \coloneq \L(X,X)$ eine \emph{Banachalgebra}, das heißt ein vollständiger normierter Vektorraum mit einer Multiplikation, so dass für $T, S ∈ \L(X)$ gilt: \[ \norm {TS} \le \norm T \norm S. \] @@ -2020,27 +2025,27 @@ In topologischen linearen Räumen gilt dies jedoch nciht. \begin{bemerkung} Ist $T ∈ \L(X,Y)$, so ist $\ker T$ als Urbild der abgeschlossenen Menge $\{ 0\}$ stets abgeschlossen in $X$. - Das Bild hingegen $R(T) := \im T$ ist im Allgemeinen jedoch nicht abgeschlossen. + Das Bild hingegen $R(T) \coloneq \im T$ ist im Allgemeinen jedoch nicht abgeschlossen. Wann sind Elemente in $\L(X)$ invertierbar? \end{bemerkung} \begin{satz} - Sei $X$ ein Banachraum und $\T ∈ \L(X)$ mit $\limsup\limits_{m → ∞} \norm{T}^{1/m} < 1$. Dann ist $(\id - T)^{-1} ∈ \L(X)$ und es gilt + Sei $X$ ein Banachraum und $\T ∈ \L(X)$ mit $\limsup\limits_{m → \infty } \norm{T}^{1/m} < 1$. Dann ist $(\id - T)^{-1} ∈ \L(X)$ und es gilt \[ - (\id-T)^{-1} = \ lim_{m → ∞} \sum_{n = 0}^m T^n =: \sum_{n = 0}^∞ T^n ∈ \L(X). + (\id-T)^{-1} = \ lim_{m → \infty } \sum_{n = 0}^m T^n =: \sum_{n = 0}^\infty T^n ∈ \L(X). \] mit Konvergenz in $\L(X)$. \end{satz} \begin{proof} - Wähle $m_0$ und $Θ < 1$ mit $\norm {T^n} < Θ^n$ für $n ≥ m_0$. - Für $S_k \sum_{n=0}^k T^n$ gilt dann für $m_0 \le k < l$ + Wähle $m_0$ und $\Theta < 1$ mit $\norm {T^n} < \Theta ^n$ für $n \ge m_0$. + Für $S_k = \sum_{n=0}^k T^n$ gilt dann für $m_0 \le k < l$ \[ - \norm{ S_l - S_k} = \norm { \sum_{n=k+1}^l T^n} \le \sum_{n=k+1}^l \norm{ T^k} \le \sum_{n=k+1}^l Θ^n < ε, \quad k, l ≥ N_0. + \norm{ S_l - S_k} = \norm { \sum_{n=k+1}^l T^n} \le \sum_{n=k+1}^l \norm{ T^k} \le \sum_{n=k+1}^l \Theta ^n < \epsilon , \quad k, l \ge N_0. \] Damit ist $(S_k)_{k ∈ ℕ}$ eine Cauchy-Folge in $\L(X)$ und somit konvergent. - Sei $S$ der Grenzwert. Dann gilt für jedes $x ∈ X$ auch $S_k x \xrightarrow[k → ∞]{\norm\cdot_{X}} Sx$, also damit ist für alle $x∈ X$ + Sei $S$ der Grenzwert. Dann gilt für jedes $x ∈ X$ auch $S_k x \xrightarrow[k → \infty ]{\norm\cdot_{X}} Sx$, also damit ist für alle $x∈ X$ \[ - (\id - T) Sx = \lim_{k → ∞} (\id -T) S_k x = \lim_{k → ∞} \sum_{n=0}^k (T^n -T^{n-1})x = \lim_{k→∞} x - T^{k+1}x = x. + (\id - T) Sx = \lim_{k → \infty } (\id -T) S_k x = \lim_{k → \infty } \sum_{n=0}^k (T^n -T^{n-1})x = \lim_{k→\infty } x - T^{k+1}x = x. \] Damit ist $(\id -T)S = \id$. Da sich analog $S(\id-T) = \id$ auch zeigen lässt, folgt die Behauptung. \end{proof} @@ -2050,13 +2055,13 @@ In topologischen linearen Räumen gilt dies jedoch nciht. 3.7.6 \end{lemma} \begin{bemerkung-nn} - Mit $Θ = 1$ geht es nicht immer. Gegenbeispiel: Sei $X = C[0,1] ∩ \{ x(0) = + Mit $\Theta = 1$ geht es nicht immer. Gegenbeispiel: Sei $X = C[0,1] ∩ \{ x(0) = 0 \}$ und $M = \{ x ∈ X : g∫_0^1 x(t) dt = 0 \}$. Dann ist $M$ ein abgeschlossener linearer Unterraum, weil $T: X → ℝ, ∫_0^1 \cdot$ stetig ist und somit $M = T^{-1}(\{0\})$ als Urbild einer abgeschlossenen Menge in $ℝ$ abgeschlossen ist. - Angenommen, ($Θ=1$), es existierte ein $x_Θ = x_ ∈ X$ mit $\norm x_1 = $ und $\norm {x-x_1} \ge 1 $ für alle $x ∈ M$. + Angenommen, ($\Theta =1$), es existierte ein $x_\Theta = x ∈ X$ mit $\norm x_1 = $ und $\norm {x-x_1} \ge 1 $ für alle $x ∈ M$. Dann setze \[ - c(y) := \frac{∫_0^1 x_1(t) dt}{∫_0^1 y(t) dt} ∈ ℝ + c(y) \coloneq \frac{∫_0^1 x_1(t) dt}{∫_0^1 y(t) dt} ∈ ℝ \] für alle $y \not\in M$. Man beachte, dass dies wohldefiniert ist. Dann ist $x_1 - c(y)y ∈ M$, also $1 \le \norm{ x_1 - c(y)y - x_1} = |c(y)|\norm y$. @@ -2072,14 +2077,14 @@ In topologischen linearen Räumen gilt dies jedoch nciht. \begin{proof} „⇐“ war Korollar 7.4. - „⇒“. Angenommen, $\dim X = \infty.$ Sei $S^1 := \{ x ∈ X: \norm x = 1\}$. + „⇒“. Angenommen, $\dim X = \infty.$ Sei $S^1 \coloneq \{ x ∈ X: \norm x = 1\}$. Da $S^1$ abgeschlossen und beschränkt ist, ist $S^1$ nach Annahme kompakt. - Wähle $x_1 ∈ S^1$ und $M_1 := \lspan \{ x_1 \} \subsetneq X$. + Wähle $x_1 ∈ S^1$ und $M_1 \coloneq \lspan \{ x_1 \} \subsetneq X$. $M_1$ ist ein abgeschlossener Unterraum nach Korollar 7.5. - Nach Ries existiert ein $x_2 ∈ S_1$ mit $\norm {x_2-x_1} \ge Θ := \frac 1 2 $. - Setze nun $M_2 := \lspan \{x_1,x_2\}$. - Da $M_2$ ein abgeschlossener Unterraum ist, existiert ein $x_3 ∈ S_1$ mit $\norm {x_3 - x} \ge Θ$ für alle $x ∈ M_2$, also insbesondere $\norm {x_3-x_1} \ge Θ = \frac 1 2$ und $\norm {x_3-x_2} \ge Θ = \frac 1 2$. - Iterativ (da $\dim X = ∞ $) existiert $x_n ∈ S_1$ mit $\norm {x_m - x_n} \ge \frac 1 2$ für $m \ge n$. + Nach Ries existiert ein $x_2 ∈ S_1$ mit $\norm {x_2-x_1} \ge \Theta \coloneq \frac 1 2 $. + Setze nun $M_2 \coloneq \lspan \{x_1,x_2\}$. + Da $M_2$ ein abgeschlossener Unterraum ist, existiert ein $x_3 ∈ S_1$ mit $\norm {x_3 - x} \ge \Theta $ für alle $x ∈ M_2$, also insbesondere $\norm {x_3-x_1} \ge \Theta = \frac 1 2$ und $\norm {x_3-x_2} \ge \Theta = \frac 1 2$. + Iterativ (da $\dim X = \infty $) existiert $x_n ∈ S_1$ mit $\norm {x_m - x_n} \ge \frac 1 2$ für $m \ge n$. Somit haben wir eine Folge $(x_n)_{n ∈ ℕ}$ ohne Häufungspunkt in $S^1$ gefunden im Widerspruch zu $S^1$ kompakt. \end{proof} @@ -2091,7 +2096,7 @@ Damit sind in unendlich-dimensionalen normierten Räumen weder die Sphären noch \end{definition} \begin{korollar} - Sei $X$ normiert, $\dim X = ∞$. Dann ist $X$ nicht lokalkompakt. + Sei $X$ normiert, $\dim X = \infty $. Dann ist $X$ nicht lokalkompakt. \end{korollar} \begin{proof} Angenommen, dass doch. Dann gibt es $r > 0$, so dass $S_r = \{ x ∈ X : \norm x = r\} ⊂ \cl U$. @@ -2112,7 +2117,7 @@ Sei wieder $\K = \R$ oder $\K = ℂ$. \item $\langle x, y \rangle = \cl {\langle y, x \rangle}$ für alle $x, y ∈ X$. \item - $\langle x, αy + β z \rangle = α \langle x, y \rangle + β \langle x,z \rangle$ für alle $α, β ∈ \K$, $x,y,z ∈ X$. + $\langle x, \alpha y + β z \rangle = \alpha \langle x, y \rangle + β \langle x,z \rangle$ für alle $\alpha , β ∈ \K$, $x,y,z ∈ X$. \end{enumerate} $(X,\langle -,- \rangle)$ heißt \emph{Skalarproduktraum}, \emph{unitärer Raum} oder \emph{Prähilbertraum}. \end{definition} @@ -2125,7 +2130,7 @@ Sei wieder $\K = \R$ oder $\K = ℂ$. Sei $(X, \langle -,- \rangle)$ ein unitärer Raum. Dann gelten die folgenden Aussagen: \begin{enumerate} \item - Durch $\norm x := \sqrt{\langle x, x \rangle}$ wird eine Norm definiert. + Durch $\norm x \coloneq \sqrt{\langle x, x \rangle}$ wird eine Norm definiert. Dadurch wird jeder unitäre Raum auf natürliche Art und Weise normiert und trägt dadurch die induzierte natürliche Topologie. \item $|\langle x,y \rangle| \le \norm x \norm y$ mit Gleichheit genau dann, wenn $x$ und $y$ linear abhängig (Cauchy-Schwarz-Ungleichung). @@ -2147,14 +2152,14 @@ Sei wieder $\K = \R$ oder $\K = ℂ$. \item Einfaches Nachrechnen unter Verwendung von (b) \item - Für $y = 0$ ist die Behauptung klar. Sei also $y \ne 0, α ∈ ℂ$. + Für $y = 0$ ist die Behauptung klar. Sei also $y \ne 0, \alpha ∈ ℂ$. Dann \[ - \langle x + αy, x+αy \rangle = \langle x, x \rangle + \cl \alpha \langle y, x \rangle + α \langle x,y \rangle + |α|^2 \langle y,y \rangle. + \langle x + \alpha y, x+\alpha y \rangle = \langle x, x \rangle + \cl \alpha \langle y, x \rangle + \alpha \langle x,y \rangle + |\alpha |^2 \langle y,y \rangle. \] - Speziell für $\cl \alpha := - \frac{\langle x,y \rangle}{\langle y,y \rangle}$ ergibt sich + Speziell für $\cl \alpha \coloneq - \frac{\langle x,y \rangle}{\langle y,y \rangle}$ ergibt sich \[ - 0 \le \langle x + αy, x+α+ \rangle = \langle x,x \rangle - \frac{|\langle x,y \rangle^2|}{\langle y,y \rangle} - \frac{|\langle x,y \rangle^2|}{\langle y,y \rangle} + \frac{|\langle x,y \rangle^2|}{\langle y,y \rangle} = \langle x,x \rangle - \frac{|\langle x,y \rangle^2|}{\langle y,y \rangle}. + 0 \le \langle x + \alpha y, x+\alpha + \rangle = \langle x,x \rangle - \frac{|\langle x,y \rangle^2|}{\langle y,y \rangle} - \frac{|\langle x,y \rangle^2|}{\langle y,y \rangle} + \frac{|\langle x,y \rangle^2|}{\langle y,y \rangle} = \langle x,x \rangle - \frac{|\langle x,y \rangle^2|}{\langle y,y \rangle}. \] Durch Umstellen ergibt sich \[ @@ -2163,7 +2168,7 @@ Sei wieder $\K = \R$ oder $\K = ℂ$. Die CSU erhält man durch Wurzel ziehen. Gleichheit gilt genau dann, wenn \[ - \langle x+ α y, x+αy \rangle = 0 \gdw x + αy = 0, + \langle x+ \alpha y, x+\alpha y \rangle = 0 \gdw x + \alpha y = 0, \] also wenn $x$ und $y$ linear abhängig sind. \item @@ -2174,9 +2179,10 @@ Sei wieder $\K = \R$ oder $\K = ℂ$. Addieren dieser Gleichungen für $+$ und $-$ ergibt die Behauptung. \item Es gilt - \[ - \norm {x+y}^2 - \norm{x-y}^2 = (\norm x^2 + 2 \Re \langle x,y\rangle + \norm y^2) - (\norm x^2 - 2 \Re \langle x,y \rangle + \norm y^2) = 4 \Re \langle x,y \rangle. - \] + \begin{align*} + \norm {x+y}^2 - \norm{x-y}^2 &= (\norm x^2 + 2 \Re \langle x,y\rangle + \norm y^2) - (\norm x^2 - 2 \Re \langle x,y \rangle + \norm y^2) \\ + & = 4 \Re \langle x,y \rangle. + \end{align*} Analog haben wir \[ -i \norm{x+iy}^2 + i \norm{x-iy}^2 = … = 4i \Im \langle x,y \rangle, @@ -2203,16 +2209,17 @@ Sei wieder $\K = \R$ oder $\K = ℂ$. \begin{bemerkung} \begin{enumerate} - \item Die Paralellogramgleichung ist also charakteristisch für unitäre Räume. \item - $(C(S),\norm\cdot_∞)$ mit $S ⊂ ℝ^n$ kompakt erfüllt dies nicht. + Die Paralellogrammgleichung ist somit charakteristisch für unitäre Räume. \item - Die Abbildung $\langle -,- \rangle$ in unitären Räumen ist stetig in beiden Komponenten als unmittelbare Konsequenz aus der Stetigkeit der Norm. + $(C(S),\norm\cdot_\infty )$ mit $S ⊂ ℝ^n$ kompakt erfüllt dies nicht. + \item + Die Abbildung $\langle \cdot,\cdot \rangle$ in unitären Räumen ist stetig in beiden Komponenten als unmittelbare Konsequenz aus der Stetigkeit der Norm. \end{enumerate} \end{bemerkung} \begin{definition} - Ein bezüglich der Norm $\norm - := \sqrt{ \langle -,- \rangle}$ vollständiger unitärer Raum $(X,\langle -,- \rangle)$ heißt \emph{Hilbertraum}. + Ein bezüglich der Norm $\norm \cdot \coloneq \sqrt{ \langle \cdot,\cdot \rangle}$ vollständiger unitärer Raum $(X,\langle \cdot,\cdot \rangle)$ heißt \emph{Hilbertraum}. \end{definition} @@ -2226,15 +2233,17 @@ Hier fehlt eine VL. Sei also $\hat y ∈ Y$ mit $x-\hat y \perp Y$, also $x-\hat y \perp (\hat y - y)$ für $y ∈ Y$ beliebig. Dann gilt mit Pythagoras \[ - \norm{x-y}^2 = \norm{x-\hat y + \hat y - y}^2 = \norm{x-\hat y}^2 + \norm{\hat y - y}^2 ≥ \norm{x-\hat y}^2, + \norm{x-y}^2 = \norm{x-\hat y + \hat y - y}^2 = \norm{x-\hat y}^2 + \norm{\hat y - y}^2 \ge \norm{x-\hat y}^2, \] was die Behauptung impliziert. \end{proof} \begin{bemerkung-nn} - Damit gilt im Hilbertraum das Riesz'sche Lemma (3.7.6) mit $Θ = 1$. + Damit gilt im Hilbertraum das Riesz'sche Lemma (3.7.6) mit $\Theta = 1$. Setze dazu - $ x_{Θ=1} := \frac{x-\hat y }{\norm{x-\hat y}} $ - für ein $x \notin Y$. Dann ist $\norm{x_Θ} = 1$ und für alle $z ∈ Y$ gilt $\norm {z-x_Θ}^2 + 2 \Re \langle z,x_Θ \rangle + \norm{x_Θ}^2 ≥ 1 = Θ$. + $ x_{\Theta =1} \coloneq \frac{x-\hat y }{\norm{x-\hat y}} $ + für ein $x \notin Y$. Dann ist $\norm{x_\Theta } = 1$ und für alle $z ∈ Y$ +gilt $\norm {z-x_\Theta }^2 + 2 \Re \langle z,x_\Theta \rangle + \norm{x_\Theta +}^2 \ge 1 = \Theta $. \end{bemerkung-nn} \begin{satz} Es sei $Y$ ein vollständiger Unterraum eines unitären Raums $X$. @@ -2315,17 +2324,17 @@ Zentral in der Hilbertraumtheorie ist der Begriff der Hilbertraumbasis. \begin{enumerate} \item Für alle $x ∈ X$ gilt die Vollständigkeitsrelation \[ - \lim_{n → ∞} \norm{x - \sum_{k=1}^n \langle \hat e_k, x \rangle \hat e_k} = 0 + \lim_{n → \infty } \norm{x - \sum_{k=1}^n \langle \hat e_k, x \rangle \hat e_k} = 0 \] \item Für alle $x, y ∈ X$ ist \[ - \langle x,y \rangle = \sum_{k=1}^∞ \cl{\langle \hat e_k. x \rangle} \langle \hat e_k, y \rangle. + \langle x,y \rangle = \sum_{k=1}^\infty \cl{\langle \hat e_k. x \rangle} \langle \hat e_k, y \rangle. \] \item Für alle $x ∈ X$ gilt die Parseval-Gleichung \[ - \norm{x}^2 = \sum_{k=1}^∞ \left| \langle \hat e_k, x \rangle \right|^2. + \norm{x}^2 = \sum_{k=1}^\infty \left| \langle \hat e_k, x \rangle \right|^2. \] \end{enumerate} \end{definition} @@ -2336,8 +2345,8 @@ Zentral in der Hilbertraumtheorie ist der Begriff der Hilbertraumbasis. \begin{enumerate} \item Statt (a) kann man auch \[ - x = \lim_{n → ∞} \sum_{k=1}^n \langle \hat e_k, x \rangle \hat e_k - = \sum_{k=1}^∞ \langle \hat e_k,x \rangle \hat e_k + x = \lim_{n → \infty } \sum_{k=1}^n \langle \hat e_k, x \rangle \hat e_k + = \sum_{k=1}^\infty \langle \hat e_k,x \rangle \hat e_k \] schreiben. Dies nennt man die Fourier-Reihe von $x$. \item @@ -2364,14 +2373,14 @@ Zentral in der Hilbertraumtheorie ist der Begriff der Hilbertraumbasis. \item Sei $S$ wie oben. Sei $x ∈ X$ mit $x \perp S$. Nach (c) gilt dann \[ - \sum_{k=1}^∞ \big| \underbrace{\langle \hat e_k^∞, x \rangle}_{=0} \big|^2 = \norm x ^2, + \sum_{k=1}^\infty \big| \underbrace{\langle \hat e_k^\infty , x \rangle}_{=0} \big|^2 = \norm x ^2, \] also $\norm x = 0$ und $x = 0$. \item Sei nun $S$ ein abzählbares vollständiges Orthonormalensystem und $X$ ein Hilbertraum. Führe den Beweis indirekt. Angenommen, $S$ wäre keine Hilbertraumbasis. - Dann gelten die Eigenschaften (a)-(c) aus der Definition nicht und wegen der obigen Bemerkung ist dann $Y := \cl{\lspan S} \subsetneq X$. + Dann gelten die Eigenschaften (a)-(c) aus der Definition nicht und wegen der obigen Bemerkung ist dann $Y \coloneq \cl{\lspan S} \subsetneq X$. $Y$ ist also ein abgeschlossener Unterraum von $X$, und da $X$ Hilbertraum ist, damit vollständig. Nach Satz 2.9 ist $X = Y \oplus Y^\perp$. Insbesondere ist also $Y^\perp \ne \{ 0\}$. @@ -2383,7 +2392,7 @@ Zentral in der Hilbertraumtheorie ist der Begriff der Hilbertraumbasis. \end{enumerate} \end{proof} \begin{frage-nn} - Hat jeder Hilbertraum $H$ mit $\dim H = ∞$ ein abzählbares vollständiges ONS (also eine Hilbertbasis)? + Hat jeder Hilbertraum $H$ mit $\dim H = \infty $ ein abzählbares vollständiges ONS (also eine Hilbertbasis)? \end{frage-nn} Die Antwort darauf ist nein, aber falls $H$ zusätzlich separabel ist, dann ist sie ja. Dagegen ist die Existenz eines vollständigen Orthonormalensystems (also eventuell überabzählbar, also keine ONB) kein Problem: @@ -2400,48 +2409,47 @@ Dagegen ist die Existenz eines vollständigen Orthonormalensystems (also eventue Sei $X = L^2(0,2\pi), \K = ℝ$. Dann ist ein VONS in $X$ gegeben durch \[ - S = \left\{ \frac 1 {\sqrt{2π}}\right\} - ∪ \left\{ \frac 1 {\sqrt{π}} \cos(nx) : n ∈ ℕ\right\} - ∪ \left\{ \frac 1 {\sqrt{π}} \sin(nx) : n ∈ ℕ\right\}. + S = \left\{ \frac 1 {\sqrt{2\pi }}\right\} + ∪ \left\{ \frac 1 {\sqrt{\pi }} \cos(nx) : n ∈ ℕ\right\} + ∪ \left\{ \frac 1 {\sqrt{\pi }} \sin(nx) : n ∈ ℕ\right\}. \] In der klassischen Fourieranalysis werden Entwicklungen nach diesem VONS $S$ untersucht. - Man zeigt dort, dass $\lspan S$ bezüglich $\norm\cdot_∞$ dicht liegt in $C_\text{per}([0,2\pi]) = \{ f: ℝ → ℝ: f$ ist stetig und $2π$-periodisch $\}$. - Die Aussage von 2.13(2) und (2.10) liefert nur die Begründung für die Dichtheit von $\lspan S$ in $\norm-_{L^2}$. + Man zeigt dort, dass $\lspan S$ bezüglich $\norm\cdot_\infty $ dicht liegt in $C_{\text{per}}([0,2\pi]) = \{ f: \R → \R: f$ ist stetig und $2\pi $-periodisch $\}$. + Die Aussage von 2.13(2) und (2.10) liefert nur die Begründung +für die Dichtheit von $\lspan S$ in $\norm-_{L^2}$. \item - Durch $(f,g)_μ := ∫_a^b μ(t) f(t) g(t)\; dt $, wobei $μ > 0$ und stetig auf $(a,b)$, ist auf $L^2(a,b)$ ein reelles Skalarprodukt definiert. - Für verschiedene Gewichtsfunktionen $μ$ und verschiedene Wahlen von $(a,b)$ erhält man $μ$-orthogonale Polynomsysteme durch Anwendung des Gram-Schmidt-Verfahrens auf die Monome $\{t^i: i ∈ ℕ_0\}$. + Durch $(f,g)_\mu \coloneq ∫_a^b \mu (t) f(t) g(t)\; dt $, wobei $\mu > 0$ und stetig auf $(a,b)$, ist auf $L^2(a,b)$ ein reelles Skalarprodukt definiert. + Für verschiedene Gewichtsfunktionen $\mu $ und verschiedene Wahlen von $(a,b)$ erhält man $\mu $-orthogonale Polynomsysteme durch Anwendung des Gram-Schmidt-Verfahrens auf die Monome $\{t^i: i ∈ ℕ_0\}$. \begin{enumerate}[label=(\roman*)] \item - $a=-1, b=1$, $μ(t) = 1$ liefert die Legendre-Polynome. + $a=-1, b=1$, $\mu (t) = 1$ liefert die Legendre-Polynome. \item - $a=-1, b=1$, $μ(t) = 1$ liefert die Tschebyscheff-Polynome. - Das stimmt nicht, danke, dass du die Folie so lange gezeigt hast. + $a=-1, b=1$, $\mu (t) = \frac 1 {\sqrt{1-t^2}}$ liefert die Tschebyscheff-Polynome. \item - $a=0, b=∞$, $μ(t) = 1$ liefert die Laguerre-Polynome. - Das stimmt nicht, danke, dass du die Folie so lange gezeigt hast. + $a=0, b=\infty $, $\mu (t) = \exp(-t)$ liefert die Laguerre-Polynome. \item - $a=-∞, b=∞$, $μ(t) = \exp(-t^2)$ liefert die Hermite-Polynome. + $a=-\infty , b=\infty $, $\mu (t) = \exp(-t^2)$ liefert die Hermite-Polynome. \end{enumerate} \item Ist $X$ ein unitärer Raum mit ONB, kann er formal vervollständigt werden: Sei also $(\hat e_k)_{k ∈ ℕ} ⊂ X$ diese ONB, dann ist \[ - H := \left\{ \sum_{k=1}^∞ c_k \hat e_k: (c_k)_{k ∈ ℕ} ∈ \ell^2 \right\} + H \coloneq \left\{ \sum_{k=1}^\infty c_k \hat e_k: (c_k)_{k ∈ ℕ} ∈ \ell^2 \right\} \] ist ein Hilbertraum, den man die Vervollständigung von $X$ nennt. Das Skalarprodukt zwischen $x = \sum_{k ∈ ℕ} c_k \hat e_k$ und $y = \sum_{k ∈ ℕ} d_k \hat e_k$ wird definiert als \[ - \langle x,y \rangle := \sum_{k=1}^∞ \cl{c_k} d_k. + \langle x,y \rangle \coloneq \sum_{k=1}^\infty \cl{c_k} d_k. \] Tatsächlich kann $H$ mit dem Koordinatenraum $\ell^2 = \ell^2(ℕ)$ identifiert werden. Die Abbildung \[ - \Phi: \ell^2(ℕ) → H, (c_k)_{k ∈ℕ} ↦ \sum_{k=1}^∞ c_k \hat e_k + \Phi: \ell^2(ℕ) → H, (c_k)_{k ∈ℕ} ↦ \sum_{k=1}^\infty c_k \hat e_k \] ist linear, bijektiv und normerhaltend wegen der Parsevalgleichung \[ - \norm{x}^2 = \sum_{k=1}^∞ \left| \langle \hat e_k, x \rangle \right|^2. + \norm{x}^2 = \sum_{k=1}^\infty \left| \langle \hat e_k, x \rangle \right|^2. \] Also $\ell^2(ℕ)$ und $H$ isometrisch und insbesondere $H$ vollständig. \end{enumerate} @@ -2450,11 +2458,11 @@ Dagegen ist die Existenz eines vollständigen Orthonormalensystems (also eventue % VL NÄCHSTE WOCHE -Der Satz 4.1 liefert also, dass die Abbildung $J_x: X → X,', y ↦ y'$ definiert +Der Satz 4.1 liefert also, dass die Abbildung $J_x: X → X', y ↦ y'$ definiert durch $y': X → \K, x ↦ \langle y,x \rangle$ bijektiv ist. Wir schreiben nun \[ -\langle \langle J_x(y),x \rangle \rangle = \langle \langle J_x(y),x \rangle \rangle_{X'×X} := J(x)(y)[x] +\lAngle J_x(y),x \rAngle = \lAngle J_x(y),x \rAngle_{X'×X} \coloneq J(x)(y)[x] = \langle y,x \rangle. \] Diese Abbildung ist sesquiliniear, das heißt @@ -2462,24 +2470,24 @@ Diese Abbildung ist sesquiliniear, das heißt J_x (y_1 + y_2) = J_x (y_1) + J_x(y_2), \quad y_1, y_2 ∈ X, \] \[ - J_x(αy) = \cl{\alpha} J_x(y), \quad α ∈ \K, + J_x(\alpha y) = \cl{\alpha} J_x(y), \quad \alpha ∈ \K, \] denn \[ - \langle \langle J_x(αy),x \rangle \rangle = \langle αy, x \rangle = \cl \alpha \langle y, x \rangle = \cl \alpha J_x(y) [x] = \cl \alpha \langle \langle J_x(y), x \rangle \rangle - \langle \langle \cl \alpha J_x(y), x \rangle \rangle, + \lAngle J_x(\alpha y),x \rAngle = \langle \alpha y, x \rangle = \cl \alpha \langle y, x \rangle = \cl \alpha J_x(y) [x] = \cl \alpha \lAngle J_x(y), x \rAngle + \lAngle \cl \alpha J_x(y), x \rAngle, \] also $X \cong X'$ sesquilinear isomorph. Gilt da sauch topologisch? -Die Topologie von $X'$ sei hierbei die von $\L(X, \K)$, also die von der Norm $\norm{y'}_{X',N} = \sup_{\norm{x} ≤ 1}|y'[x]|$ erzeugte. +Die Topologie von $X'$ sei hierbei die von $\L(X, \K)$, also die von der Norm $\norm{y'}_{X',N} = \sup_{\norm{x} \le 1}|y'[x]|$ erzeugte. \begin{satz} $X$ und $X'$ sind Hilberträume und $J_x: X → X'$ ist kanonischer sesquilinearer Isomorphismus, der die Norm erhält, also eine Isometrie. Genauer gilt: \begin{enumerate} \item - $\langle y_1', y_2' \rangle_{X'} := \cl{ \langle y_1, y_2 \rangle_X}$, wobei $J_x(y_1) = y_1', J_x(y_2) = y_2'$, macht $X'$ zum Skalarproduktraum. + $\langle y_1', y_2' \rangle_{X'} \coloneq \cl{ \langle y_1, y_2 \rangle_X}$, wobei $J_x(y_1) = y_1', J_x(y_2) = y_2'$, macht $X'$ zum Skalarproduktraum. \item Die durch $\langle -,- \rangle_{X'}$ induzierte Norm \[ @@ -2497,7 +2505,7 @@ Die Topologie von $X'$ sei hierbei die von $\L(X, \K)$, also die von der Norm $\ \item Beispielsweise ist \[ - \langle α y_1' , y_2' \rangle_{X'} \stackrel{def}{=} \cl{\langle \cl \alpha y_1, y_2 \rangle_X} = \cl{ \alpha \langle y_1, y_2 \rangle_X} = \cl{\alpha} \langle y_1',y_2' \rangle_{X'}, + \langle \alpha y_1' , y_2' \rangle_{X'} \stackrel{\text{def}}{=} \cl{\langle \cl \alpha y_1, y_2 \rangle_X} = \cl{ \alpha \langle y_1, y_2 \rangle_X} = \cl{\alpha} \langle y_1',y_2' \rangle_{X'}, \] die anderen Eigenschaften folgen analog. \item @@ -2505,7 +2513,7 @@ Die Topologie von $X'$ sei hierbei die von $\L(X, \K)$, also die von der Norm $\ \[ \norm{y'}_{X',N} = \sup_{\norm x \le 1} |y'[x]| = \norm{y}_{X} \quad \text{für alle $y ∈ X$}. \] - hierbei ist aber „$\le$“ gerade die Cauchy-Schwarzsche Ungleichung, für „$\ge$“ wähle $x = \frac y {\norm y _{X}}$ für $y \ne 0$ ($y=0$ ist sowieso klar). + hierbei ist aber „$\le$“ gerade die Cauchy"=Schwarzsche Ungleichung, für „$\ge$“ wähle $x = \frac y {\norm y _{X}}$ für $y \ne 0$ ($y=0$ ist sowieso klar). \item nichts zu zeigen. \item @@ -2513,6 +2521,38 @@ Die Topologie von $X'$ sei hierbei die von $\L(X, \K)$, also die von der Norm $\ \end{enumerate} \end{proof} +\section{Separable Hilberträume} +\begin{definition} + Ein metrischer Raum $(X,d)$ heißt \emph{separabel}, wenn es $U ⊂ X$ dicht + und abzählbar gibt. +\end{definition} + +\begin{beispiele} + $ℝ^n, ℂ^n, \ell^2, L^2(\Omega)$ für $\Omega ⊂ ℝ^n$ offen sind separable Hilberträume. +\end{beispiele} + +\begin{satz} + In einem separablen unendlich"=dimensionalen Hilbertraum $(X,\langle -,- \rangle)$ gilt + \begin{enumerate} + \item Jedes ONS in $X$ ist höchstens abzählbar. + \item + Sei $S = (\hat e_i)_{i ∈ ℕ}$ ein VONS in $X$. Dann existiert zu jeder + Folge $\alpha = (\alpha _k)_{k ∈ ℕ} ∈ \ell^2$ genau ein $x ∈ X$ mit $\langle \hat + e_k, x \rangle = \alpha _k, k ∈ ℕ$ (Satz von \emph{Riesz-Fischer}). + \item + $X$ ist isometrisch isomorph zum $\ell^2$. Insbesondere sind + $L^2(\Omega)$ und $\ell^2$ isometrisch isomorph. + \end{enumerate} +\end{satz} + + +\section{Riesz'scher Darstellungssatz und Lax-Milgram} +Für einen topologischen linearen Raum $X$ ist der Dualraum $X' = \{x': X → \K, x' $ linear und stetig $\}$ definiert. +Im Allgemeinen kann auch $X' = \{0\}$ gelten. +Ist $X$ jedoch ein Hilbertraum, so ist stets $X' \ne \{0\}$, denn zu $y ∈ X$ ist durch $y'[x] \coloneq \langle y,x \rangle, x ∈ X$ jeweils ein $y' ∈ X'$ erklärt. +Tatsächlich bekommt man dadurch sogar schon alle Elemente des Dualraums: + + \chapter{Der Satz von Hahn-Banach und seine Konsequenzen} \section{Fortsetzbarkeit linearer Funktionale} @@ -2538,49 +2578,49 @@ Wir fragen uns, ob sich Abbildungen so erweitern, dass gewisse Eigenschaften (wi \end{satz} \begin{proof} Zeigen wir zunächst die Existenz der Fortsetzung. - Da $X_0$ dicht in $X$ ist, existiert zu jedem $x ∈ X$ eine Folge $(x_n)_{n ≥1}$, die ganz in $X_0$ liegt und gegen $x$ konvergiert. + Da $X_0$ dicht in $X$ ist, existiert zu jedem $x ∈ X$ eine Folge $(x_n)_{n \ge1}$, die ganz in $X_0$ liegt und gegen $x$ konvergiert. Wir behaupten, dass $(A_0x_n)_{n ∈ ℕ}$ eine Cauchy-Folge in $Y$ ist. Dazu beachte, dass \[ - \norm{A_0 x_n - A_0 x_m}_{Y} \le \norm{A_0}_{\L(X_0,Y)} \norm{x_n-x_m} \xrightarrow[n,m → ∞]{} 0. - \] - Da $Y$ ein Banachraum ist, ist $(A_0x_n)_{n≥1}$ konvergiert, etwa gegen $y$. - Wir setzen $Ax := y$. - Zunächst ist $A$ wohldefiniert, denn wenn $(z_n)_{n ≥ 1}$ eine weitere Folge mit $\lim_{n → ∞} z_n = x$ ist, dann gilt - $z_n - x_n \xrightarrow[n→∞]{} 0$ und - \[ - \norm{A_0 z_n - y} \le \norm{A_0 z_n - A_0 x_n} + \norm{A_0 x_n - y} - \le - \norm{A_0} \norm{z_n - x_n} + \norm{A_0 x_n - y} \xrightarrow[n→∞]{} 0. + \norm{A_0 x_n - A_0 x_m}_{Y} \le \norm{A_0}_{\L(X_0,Y)} \norm{x_n-x_m} \xrightarrow[n,m → \infty ]{} 0. \] + Da $Y$ ein Banachraum ist, ist $(A_0x_n)_{n\ge1}$ konvergiert, etwa gegen $y$. + Wir setzen $Ax \coloneq y$. + Zunächst ist $A$ wohldefiniert, denn wenn $(z_n)_{n \ge 1}$ eine weitere Folge mit $\lim_{n → \infty } z_n = x$ ist, dann gilt + $z_n - x_n \xrightarrow[n→\infty ]{} 0$ und + \begin{align*} + \norm{A_0 z_n - y} &\le \norm{A_0 z_n - A_0 x_n} + \norm{A_0 x_n - y} \\ + & \le + \norm{A_0} \norm{z_n - x_n} + \norm{A_0 x_n - y} \xrightarrow[n→\infty ]{} 0. + \end{align*} Offensichtlich ist $A$ eine Fortsetzung von $A_0$. Dass $A$ linear ist, ist ebenfalls klar. Zur Stetigkeit ist - \[ - \norm{Ax}_Y = \norm{\lim_{n → ∞} A_0 x_n}_Y = \lim_{n → ∞} \norm{A_0 x_n}_{Y} - \le - \lim_{n → ∞} \norm{A_0}_{\L(X_0,Y)} \norm{x_n}_X = \norm{A_0} \norm{x}. - \] + \begin{align*} + \norm{Ax}_Y &= \norm{\lim_{n → \infty } A_0 x_n}_Y = \lim_{n → \infty } \norm{A_0 x_n}_{Y} \\ + &\le + \lim_{n → \infty } \norm{A_0}_{\L(X_0,Y)} \norm{x_n}_X = \norm{A_0} \norm{x}. + \end{align*} Damit ist $A$ beschränkt, also auch stetig. Es gilt $\norm{A_0}_{\L(X_0,Y)} = \norm{A}_{\L(X,Y)}$: „$\ge$“ ist aus dem Vorherigen klar. Für die andere Ungleichung ist \[ \norm{A}_{L(X,Y)} = - \sup_{\norm{x ≤ 1}, x ∈ X} \norm{Ax}_{Y} - ≥ - \sup_{\norm{x ≤ 1}, x ∈ X_0} \norm{Ax}_{Y} = \norm{A_0}_{\L(X_0,Y)}. + \sup_{\norm{x \le 1}, x ∈ X} \norm{Ax}_{Y} + \ge + \sup_{\norm{x \le 1}, x ∈ X_0} \norm{Ax}_{Y} = \norm{A_0}_{\L(X_0,Y)}. \] Für die Eindeutigkeit sei $B: X → Y$ eine weitere stetige, lineare Fortsetzung von $A_0$. - Wie oben existiert zu jedem $x ∈ X$ eine Folge $(x_n)_{n ∈ ℕ} ⊂ X$ mit $\lim_{n → ∞} x_n = x$. + Wie oben existiert zu jedem $x ∈ X$ eine Folge $(x_n)_{n ∈ ℕ} ⊂ X$ mit $\lim_{n → \infty } x_n = x$. Dann ist \[ Ax_n = A_0 x_n = Bx_n \quad ∀ n ∈ ℕ \] und für $x ∈ X$ \[ - \norm{B_x - A_x} ≤ \norm{B_x - Bx_n} + \norm{Bx_n - Ax_n} + \norm{Ax_n - Ax} \xrightarrow[n→∞]{} 0, + \norm{B_x - A_x} \le \norm{B_x - Bx_n} + \norm{Bx_n - Ax_n} + \norm{Ax_n - Ax} \xrightarrow[n→\infty ]{} 0, \] da $A$ und $B$ stetig sind. Also $Bx = Ax$ für alle $x ∈ X$ und damit $B = A$. \end{proof} @@ -2600,18 +2640,18 @@ Ist $X_0$ nicht dicht in $X$, wird die Fortsetzung schwieriger. Auf dem linearen Raum $X$ über $ℝ$ gebe es eine Abbildung $p: X → ℝ$ mit: \begin{enumerate}[label=(\roman*)] \item - $p(αx) = αp(x)$ für alle $α ≥ 0, x ∈ X$ (positiv homogen) + $p(\alpha x) = \alpha p(x)$ für alle $\alpha \ge 0, x ∈ X$ (positiv homogen) \item - $p(x+y) ≤ p(x) + p(y)$ für alle $x, y ∈ X$ (subadditiv) + $p(x+y) \le p(x) + p(y)$ für alle $x, y ∈ X$ (subadditiv) \end{enumerate} Weiter seine $X_0$ ein linearer Teilraum von $X$ und $f_0 : X_0 → ℝ$ eine lineare Abbildung mit \[ - ∀x ∈ X_0 : f_0(x) ≤ p(x). + ∀x ∈ X_0 : f_0(x) \le p(x). \] Dann gibt es eine lineare Fortsetzung $f: X → ℝ$ von $f_0$, welche die Ungleichung respektiert, das heißt \[ - f|_{X_0} = f_0 \quad \text{und} \quad ∀x ∈ X: f(x) ≤ p(x). + f|_{X_0} = f_0 \quad \text{und} \quad ∀x ∈ X: f(x) \le p(x). \] \end{satz} \begin{bemerkung-nn} @@ -2619,30 +2659,30 @@ Ist $X_0$ nicht dicht in $X$, wird die Fortsetzung schwieriger. \end{bemerkung-nn} \begin{proof} Schritt 1. - Wir setzen $f_0$ auf $X_1 := X_0 \oplus \lspan{x_1}$ für ein $x_1 \not\in X$ (existiert immer solange $X_0 \subsetneqq X$). + Wir setzen $f_0$ auf $X_1 \coloneq X_0 \oplus \lspan{x_1}$ für ein $x_1 \not\in X$ (existiert immer solange $X_0 \subsetneqq X$). Offenbar hat jedes $x ∈X_1$ eine eindeutig Darstellung als - $ y = y + \alpha x_1 $, mit $y ∈ X_0$, $α ∈ ℝ$. + $ y = y + \alpha x_1 $, mit $y ∈ X_0$, $\alpha ∈ ℝ$. Dann ist mit $c ∈ ℝ$ beliebig \[ - f(x) = f(y + α(x_1)) := f_0(y) + αc + f(x) = f(y + \alpha (x_1)) \coloneq f_0(y) + \alpha c \] eine lineare Abbildung $X_1 → ℝ$, die $f_0$ fortsetzt. - Wir müssen $c$ so wählen, dass $f(x) ≤ p(x)$ für alle $x ∈ X_1$, also $f_0(y) + αc \le p(y+αx_1)$ für alle $y ∈ X_0, α ∈ ℝ$. + Wir müssen $c$ so wählen, dass $f(x) \le p(x)$ für alle $x ∈ X_1$, also $f_0(y) + \alpha c \le p(y+\alpha x_1)$ für alle $y ∈ X_0, \alpha ∈ ℝ$. Mit (i) ist diese Bedingung äquivalent zu zwei anderen Bedingungen: \begin{enumerate} \item - Für $a > 0$: $f_0(y/α) + c ≤ p(y/α + x_1)$. + Für $a > 0$: $f_0(y/\alpha ) + c \le p(y/\alpha + x_1)$. \item - Für $α < 0$: $f_0(-y/α) - c ≤ p(-y/α - x_1)$ + Für $\alpha < 0$: $f_0(-y/\alpha ) - c \le p(-y/\alpha - x_1)$ \end{enumerate} - für alle $y ∈ X_0$. Der Fall $α = 0$ ist nach Annahme ohnehin klar. + für alle $y ∈ X_0$. Der Fall $\alpha = 0$ ist nach Annahme ohnehin klar. Um diese Bedingungen erfüllen zu können, muss $c ∈ ℝ$ so gewählt werden, dass \[ - ∀y_1, y_2 ∈ X_0: f_0(y_1) - p(y_1 - x_1) ≤ c ≤ p(y_2 + x_2) - f_0(y_2). + ∀y_1, y_2 ∈ X_0: f_0(y_1) - p(y_1 - x_1) \le c \le p(y_2 + x_2) - f_0(y_2). \] Das ist möglich, da \[ - f_0(y_1) + f_0(y_2) = f_0(y_1+y_2) ≤ p(y_1 + y_2) = p(y_1 - x_1 + y_2 + x_1) ≤ p(y_1 - x_1)+p(y_2+x_1). + f_0(y_1) + f_0(y_2) = f_0(y_1+y_2) \le p(y_1 + y_2) = p(y_1 - x_1 + y_2 + x_1) \le p(y_1 - x_1)+p(y_2+x_1). \] Folglich gilt \[ @@ -2654,13 +2694,13 @@ Ist $X_0$ nicht dicht in $X$, wird die Fortsetzung schwieriger. Finde eine maximale Fortsetzung mit dem Lemma von Zorn. Betrachte dazu \[ - \{: X \supset D_g \supset X_0 → ℝ\}: g|_{X_0} = f_0 ∧ ∀x ∈ D_g: g(x) ≤ p(x) \}. + \{: X \supset D_g \supset X_0 → ℝ\}: g|_{X_0} = f_0 ∧ ∀x ∈ D_g: g(x) \le p(x) \}. \] Diese Menge ordnen wir mit $\succeq$ definiert durch \[ h \succeq g \gdw h \text{ ist Fortsetzung von $g$}. \] - Nach dem Lemma von Zorn existiert eine maximale Fortsetzung $g^*$ von $f_0$ mit $g^*(x) ≤ p(x)$ für alle $x ∈ X$. + Nach dem Lemma von Zorn existiert eine maximale Fortsetzung $g^*$ von $f_0$ mit $g^*(x) \le p(x)$ für alle $x ∈ X$. Wäre $D_{g^*}$ nicht $X$, so verfahre wie in Schritt 1 im Widerspruch zur Maximalität. Damit hat $g^*$ die gewünschten Eigenschaften. \end{proof} @@ -2668,7 +2708,7 @@ Ist $X_0$ nicht dicht in $X$, wird die Fortsetzung schwieriger. \begin{bemerkung-nn} \begin{enumerate} \item - Ohne die Zusatzforderung $f(x) ≤ p(x)$ für alle $x ∈X$ ist die lineare Fortsetzbarkeit trivial. + Ohne die Zusatzforderung $f(x) \le p(x)$ für alle $x ∈X$ ist die lineare Fortsetzbarkeit trivial. \item Eine Fortsetzung für lineare Funktionale $f_0: X_0 → \K = ℂ$ ist analog möglich. % yos IV 4 \end{enumerate} @@ -2681,53 +2721,53 @@ Ist $X_0$ nicht dicht in $X$, wird die Fortsetzung schwieriger. Dann existiert zu jedem $x_0 \not\in M$ ein $f ∈ X'$ mit \[ - f(x_0) > 1 ∧ ∀ x ∈ M: f(x) ≤ 1. + f(x_0) > 1 ∧ ∀ x ∈ M: f(x) \le 1. \] \end{satz} -Die Hyperebene $H = \{ x ∈ X: f(x) = 1 + ε\}$ für $0 < ε < f(x_0) < 1$ trennt also $x_0$ und $M$. +Die Hyperebene $H = \{ x ∈ X: f(x) = 1 + \epsilon \}$ für $0 < \epsilon < f(x_0) < 1$ trennt also $x_0$ und $M$. \begin{proof} - Setze $2r := \inf_{y ∈ M} \norm{y - x_0}$ (positiv, da $M$ abgeschlossen). - Sei $N := \cl{M + \cl{B_r(0)}} = \cl{\{ z = y + u: y ∈ M, u ∈ \cl{B_r(0)}\}} ⊂ X$. + Setze $2r \coloneq \inf_{y ∈ M} \norm{y - x_0}$ (positiv, da $M$ abgeschlossen). + Sei $N \coloneq \cl{M + \cl{B_r(0)}} = \cl{\{ z = y + u: y ∈ M, u ∈ \cl{B_r(0)}\}} ⊂ X$. Dann ist (i) $N$ abgeschlossen und (ii) $\cl{B_r(0)} ⊂ N$, da $0 ∈ M$, insbesondere ist $0 ∈ N^\circ$. (iii) ist $N$ konvex: Es genügt, zu zeigen, dass $A = M + B_r(0)$ konvex ist, denn dann ist auch $\cl A$ konvex. - Sei $x _i = y_i + v_i, y_i ∈ M, v_i ∈ \cl{B_r(0)}, i=1,2$ und $α ∈ (0,1)$. Dann ist + Sei $x _i = y_i + v_i, y_i ∈ M, v_i ∈ \cl{B_r(0)}, i=1,2$ und $\alpha ∈ (0,1)$. Dann ist \[ - αx_1 + (1-α)x_2 = \underbrace{[αy_1 + (1-α)y_2]}_{∈ M} + \underbrace{[αu_1+(1-α)v_2]}_{∈ \cl{B_r(0)}}. + \alpha x_1 + (1-\alpha )x_2 = \underbrace{[\alpha y_1 + (1-\alpha )y_2]}_{∈ M} + \underbrace{[\alpha u_1+(1-\alpha )v_2]}_{∈ \cl{B_r(0)}}. \] (iv) ist $x_0 \not\in N$. - Angenommen, $x_0 ∈ N$. Dann existiert eine Folge $z_n = y_n + u_n$ in $A$ mit $z_n → x_0 (n→∞)$. + Angenommen, $x_0 ∈ N$. Dann existiert eine Folge $z_n = y_n + u_n$ in $A$ mit $z_n → x_0 (n→\infty )$. Dann ist für $n_0$ hinreichend groß \[ - \frac r 2 > \norm{z_{n_0} - x_0} = \norm{y_{n_0 - x_0} + u_{n_0}} ≥ |\underbrace{\norm{y_{n_0-x_0}}}_{≥ 2r} - \underbrace{\norm{u_{n_0}}}_{≤ r}| ≥ r. + \frac r 2 > \norm{z_{n_0} - x_0} = \norm{y_{n_0 - x_0} + u_{n_0}} \ge |\underbrace{\norm{y_{n_0-x_0}}}_{\ge 2r} - \underbrace{\norm{u_{n_0}}}_{\le r}| \ge r. \] Verwende nun das Minkowski-Funktional \[ - p_N(x) := \inf \{ρ > 0: ρ^{-1} x ∈ N\}, \quad x ∈ X. + p_N(x) \coloneq \inf \{ρ > 0: ρ^{-1} x ∈ N\}, \quad x ∈ X. \] Dieses hat die Eigenschaften \begin{enumerate} \item - $p_N(αx) = αp_n(x),\quad α ≥ 0, x ∈ X$ (positiv homogen) + $p_N(\alpha x) = \alpha p_n(x),\quad \alpha \ge 0, x ∈ X$ (positiv homogen) \item - $p_N(x+y) ≤ p_N(x) + p_N(y), \quad x, y ∈ X$ (subadditiv) + $p_N(x+y) \le p_N(x) + p_N(y), \quad x, y ∈ X$ (subadditiv) \item - $p_N(x) ≤ 1 \iff x ∈ N$ + $p_N(x) \le 1 \iff x ∈ N$ \item - Ist zusätzlich $\cl{B_r(0)} ⊂ N$, so gilt $p_nNx) ≤ r^{-1}\norm x$ für alle $x ∈ X$. + Ist zusätzlich $\cl{B_r(0)} ⊂ N$, so gilt $p_nNx) \le r^{-1}\norm x$ für alle $x ∈ X$. \end{enumerate} - Sei nun $X_0 := \lspan\{x_0\}$ und $f_0 : X_0 → ℝ$ linear definiert durch $f_0(x_0) := p_N(x_0)$. - Wir behauptung, dass $f_0 (x) ≤ p_N(x)$ für alle $x = λx_0 ∈ X_0$. - Falls $λ ≥ 0$, so ist $f_0(x) = f_0(λx_0) = λp_N(x_0) = p_N(λx_0) = p_N(x)$. - Falls $λ < 0$, so ist wegen $p_n ≥ 0$ ohnehin $f_0(λx_0) = λp_N(x_0) ≤ 0 ≤ p_N(λx_0)$. + Sei nun $X_0 \coloneq \lspan\{x_0\}$ und $f_0 : X_0 → ℝ$ linear definiert durch $f_0(x_0) \coloneq p_N(x_0)$. + Wir behauptung, dass $f_0 (x) \le p_N(x)$ für alle $x = \lambda x_0 ∈ X_0$. + Falls $\lambda \ge 0$, so ist $f_0(x) = f_0(\lambda x_0) = \lambda p_N(x_0) = p_N(\lambda x_0) = p_N(x)$. + Falls $\lambda < 0$, so ist wegen $p_n \ge 0$ ohnehin $f_0(\lambda x_0) = \lambda p_N(x_0) \le 0 \le p_N(\lambda x_0)$. Da $p_N$ die Bedingungen (i) und (ii) aus Hahn-Banach erfüllt, - gibt es eine lineare Fortsetzung $f$ von $f_0$ mit $f(x) ≤ p_N(x)$ für alle $x ∈ X$. + gibt es eine lineare Fortsetzung $f$ von $f_0$ mit $f(x) \le p_N(x)$ für alle $x ∈ X$. Nun ist $f$ stetig, also $f ∈ X'$, denn für alle $x ∈ X$ gilt \begin{multline*} - |f(x) = \max\{f(x), -f(x)\} = \max\{f(x),f(-x)\} ≤ \max\{p_N(x),p_N(-x)\} \\ - ≤ \max\left\{\frac{\norm{x}}{r},\frac{\norm{-x}}{r}\right\} = \frac{\norm x}{r}. + |f(x) = \max\{f(x), -f(x)\} = \max\{f(x),f(-x)\} \le \max\{p_N(x),p_N(-x)\} \\ + \le \max\left\{\frac{\norm{x}}{r},\frac{\norm{-x}}{r}\right\} = \frac{\norm x}{r}. \end{multline*} Außerdem erfüllt $f$ die Gleichung 3.1 (?), denn @@ -2736,27 +2776,27 @@ Die Hyperebene $H = \{ x ∈ X: f(x) = 1 + ε\}$ für $0 < ε < f(x_0) < 1$ tren \] und für $x ∈ M ⊂ N$ gilt \[ - f(x) ≤ p_N(x) ≤ 1. + f(x) \le p_N(x) \le 1. \] \end{proof} \section{Einbettung von $X$ in seinen Bidualraum} Zunächst zur Motivation: Sei $X$ ein normierter linearer Raum. Dann existiert $X'$ und ist ein Banachraum. -Aber dann existiert auch $X'' := (X')'$ und ist ebenfalls ein Banachraum. -Unser ziel wird es nun sein, $X$ in $X''$ einzubetten. +Aber dann existiert auch $X'' \coloneq (X')'$ und ist ebenfalls ein Banachraum. +Unser Ziel wird es nun sein, $X$ in $X''$ einzubetten. \begin{definition} Die kanonische Abbildung $J_0: X → X''$ ist definiert durch \[ - J_0(x) [x'] = \langle \langle J_0(x), x' \rangle \rangle_{X''×X'} := \langle \langle x', x \rangle \rangle_{X'×X} = x'[x] ∈ \K + J_0(x) [x'] = \lAngle J_0(x), x' \rAngle_{X''×X'} \coloneq \lAngle x', x \rAngle_{X'×X} = x'[x] ∈ \K \] für $x ∈ X, x' ∈ X'$. Offensichtlich gilt für $x ∈ X$ fest $J_0(x): X' → \K$ linear, aber $J_0(x)$ ist auch stetig bzw beschränkt: Dazu ist \[ - |J_0(x)[x']| = | \langle \langle x',x \rangle \rangle ≤ \norm{x'}_{X'} \underbrace{\norm{x}_X}_{=: M}. + |J_0(x)[x']| = | \langle \langle x',x \rAngle \le \norm{x'}_{X'} \underbrace{\norm{x}_X}_{=: M}. \] Also ist $J_0(x) ∈ X''$, also insbesondere $J_0$ wohldefiniert. Wegen der linearität von $J_0$ in $x$ schreiben wir statt $J_0(x)$ auch $J_0 x$. @@ -2774,29 +2814,28 @@ Unser ziel wird es nun sein, $X$ in $X''$ einzubetten. Zur Injektivität: Seien $x_1, x_2 ∈ X$ mit $J_0x_1 = J_0x_2$. Dann ist für jedes $x' ∈ X'$ \[ - \langle \langle x',x_1 \rangle \rangle = J_0 x_1[x'] = J_0x_2[x'] = \langle \langle x', x_2 \rangle \rangle, + \lAngle x',x_1 \rAngle = J_0 x_1[x'] = J_0x_2[x'] = \lAngle x', x_2 \rAngle, \] also wegen Linearität von $x'$ \[ - \langle \langle x', x_1-x_2 \rangle \rangle = 0. + \lAngle x', x_1-x_2 \rAngle = 0. \] Mit Folgerung 2.3(1) folgt $x_1-x_2 = 0$. Zur Isometrieeigenschaft bleibt zu zeigen: $\norm{J_0x} = \norm{x}$ für alle $x ∈ X''$. - „≤“: Aus (4.1) folgt bereits + „$\le$“: Aus (4.1) folgt bereits \[ - \norm{J_0(x)}_{X''} = \sup_{\norm{x'} ≤ 1} |J_0(x)[x'] ≤ \norm{x}_X. + \norm{J_0(x)}_{X''} = \sup_{\norm{x'} \le 1} |J_0(x)[x'] \le \norm{x}_X. \] - „≥“: Zu $x_0 ∈ X$ existiert nach Korollar 2.1 ein $x_0' ∈ X'$ mit + „$\ge$“: Zu $x_0 ∈ X$ existiert nach Korollar 2.1 ein $x_0' ∈ X'$ mit $\norm{x_0'}_{X'} = 1$ und $x_0'[x_0]= \norm{x_0}$. Also folgt \[ - \underbrace{|J_0x_0[x_0']|}_{≤ \norm{J_0x_0}_{X''}} = \langle \langle x_0', x_0 \rangle \rangle = \norm{x_0}. + \underbrace{|J_0x_0[x_0']|}_{\le \norm{J_0x_0}_{X''}} = \lAngle x_0', x_0 \rAngle = \norm{x_0}. \] - Da $x_0$ beliebig war, gilt $\norm{J_0x}_{X''} ≥ \norm{x}$. + Da $x_0$ beliebig war, gilt $\norm{J_0x}_{X''} \ge \norm{x}$. \end{proof} - \begin{definition} Ein Banachraum $X$ heißt \emph{reflexiv}, wenn $J_0$ surjektiv ist, also $X$ und $X''$ isomorph sind vermöge $J_0$. \end{definition} @@ -2831,9 +2870,9 @@ Unser ziel wird es nun sein, $X$ in $X''$ einzubetten. \end{bemerkung-nn} \begin{definition} - Eine Folge $(x_n)_{n ∈ ℕ}$ in einem normierten Raum $X$ heißt \emph{schwach konvergent} gegen $x ∈ X$ (in Zeichen: $x_n \rightharpoonup x$ für $n → ∞$), wenn + Eine Folge $(x_n)_{n ∈ ℕ}$ in einem normierten Raum $X$ heißt \emph{schwach konvergent} gegen $x ∈ X$ (in Zeichen: $x_n \xrightharpoonup[n → \infty ]{} x$), wenn \[ - \lim_{n → ∞} x'[x_n] = x'[x] + \lim_{n → \infty } x'[x_n] = x'[x] \] für alle $x' ∈ X'$ gilt. \end{definition} @@ -2845,12 +2884,12 @@ Unser ziel wird es nun sein, $X$ in $X''$ einzubetten. \begin{beispiel-nn} Für $(\hat e_i)_{i ∈ ℕ}$ Hilbertraumbasis in einem separablem Hilbertraum $X$ gilt \[ - \hat e_i \rightharpoonup 0 ∈ X (i → ∞) + \hat e_i \rightharpoonup 0 ∈ X (i → \infty ) \] \end{beispiel-nn} \begin{bemerkung-nn} - $(\hat e_i)_{i ∈ ℕ}$ ist nicht konvergent in der Normtopologie, die Folge ist noch nicht mal Cauchy, insbesondere ist $\norm{\hat e_i - 0} \not \to 0 (i → ∞)$. + $(\hat e_i)_{i ∈ ℕ}$ ist nicht konvergent in der Normtopologie, die Folge ist noch nicht mal Cauchy, insbesondere ist $\norm{\hat e_i - 0} \not\rightarrow 0 (i → \infty )$. \end{bemerkung-nn} \begin{proof} @@ -2858,12 +2897,13 @@ Unser ziel wird es nun sein, $X$ in $X''$ einzubetten. \[ X' = \{ x' : x' ∈ X'\} = \{ J_X(y) : y ∈ X\}. \] - Zu zeigen ist $\lim\limits_{i → ∞}x'[\hat e_i] = x'[0]$ für alle $x' ∈ X'$, also äquivalent - $\lim\limits_{i → ∞} J_x(y)[\hat e_i] = J_x(y)[0]$ für alle $y ∈ X$ bzw. $\lim\limits_{i → ∞} \langle y, \hat e_i \rangle = \langle y, 0 \rangle$ für alle $y ∈ X$. + Zu zeigen ist $\lim\limits_{i → \infty }x'[\hat e_i] = x'[0]$ für alle $x' ∈ X'$, also äquivalent + $\lim\limits_{i → \infty } J_x(y)[\hat e_i] = J_x(y)[0]$ für alle $y ∈ X$ bzw. $\lim\limits_{i → \infty } \langle y, \hat e_i \rangle = \langle y, 0 \rangle$ für alle $y ∈ X$. - Sei also $y ∈ X$ fest gewählt. Dann ist $y = \sum_{i=1}^∞α_i \hat e_i$ mit $α_i = \langle \hat e_i, y \rangle$. - Es gilt $\sum_{i=1}^∞ |α_i|^2 < ∞$ (vgl Def 4.2.12). - Damit folgt $α_i = \langle \hat e_i, y \rangle → 0 (i → ∞)$, weil $α ∈ \ell^2$. Damit folgt die Schache Konvergenz von $(\hat e_i)_{i ∈ ℕ}$. + Sei also $y ∈ X$ fest gewählt. Dann ist $y = \sum_{i=1}^\infty \alpha _i \hat e_i$ mit $\alpha _i = \langle \hat e_i, y \rangle$. + Es gilt $\sum_{i=1}^\infty |\alpha _i|^2 < \infty $ (vgl Def 4.2.12). + Damit folgt $\alpha _i = \langle \hat e_i, y \rangle → 0 (i → \infty )$, weil $\alpha ∈ \ell^2$. + Damit folgt die Schwache Konvergenz von $(\hat e_i)_{i ∈ ℕ}$. \end{proof} @@ -2879,11 +2919,7 @@ Unser ziel wird es nun sein, $X$ in $X''$ einzubetten. \end{satz} - - - -\end{document} %%% Local Variables: %%% mode: latex -%%% TeX-master: "funkana" +%%% TeX-master: "funkana-ebook" %%% End: \ No newline at end of file diff --git a/latexmkrc b/latexmkrc index 8354309..9dbd136 100644 --- a/latexmkrc +++ b/latexmkrc @@ -1,4 +1,4 @@ -#$pdflatex = 'lualatex --recorder --synctex=1 --shell-escape -halt-on-error %O %S && cp %D ./pdf/%R.pdf'; -$pdflatex = 'pdflatex --recorder --synctex=1 --shell-escape -halt-on-error %O %S && cp %D ./pdf/%R.pdf'; +$pdflatex = 'xelatex --recorder --synctex=1 --shell-escape -halt-on-error %O %S && cp %D ./pdf/%R.pdf'; +#$pdflatex = 'pdflatex --recorder --synctex=1 --shell-escape -halt-on-error %O %S && cp %D ./pdf/%R.pdf'; $pdf_mode = 1; $out_dir = "build"; \ No newline at end of file diff --git a/motivation.tex b/motivation.tex new file mode 100644 index 0000000..b7c631b --- /dev/null +++ b/motivation.tex @@ -0,0 +1,68 @@ +\section*{Motivation} \markboth{}{Motivation} +In der klassischen Analyis haben wir Funktionen im $\K^n$, wobei $\K$ entweder $ℝ$ oder $ℂ$ ist, untersucht. +Dabei war das Betrachten von Eigenschaften wie Konvergenz, Stetigkeit und Differenzierbarkeit sehr nützlich. +Die Funktionalanalysis beschäftigt sich nun mit vergleichbaren Problemen in üblicherweise unendlich"=dimensionalen Funktionenräumen. +Hierfür werden wir versuchen, die aus der klassischen Analysis bekannten Untersuchungsmethoden zu verallgemeinern. +Doch zunächst ein paar Probleme, für deren Lösung man die Funktionalanalysis benötigt. + +\begin{problem-nn} + Ein klassisches Beispiel aus der Variationsrechnung: + Wir wollen die Funktion + \[ + f(u) = \int_0^\pi |u'(x)|^2 dx + \] + unter den Nebenbedingungungen $u(0) = u(\pi ) = 0$ und $\int_0^\pi |u(x)|^2 dx = 1$ minimieren. + In der klassischen Analysis haben wir für Minimierungsprobleme mit Nebenbedingungungen Lagrange-Multiplikatoren genutzt. + Im unendlich"=dimensionalen Fall ist das jedoch nicht so einfach. + Wir betrachten $f : Y → ℝ$ wie oben, wobei $Y$ eine Teilmenge des unendlich"=dimensionalen Funktionenraums + \[ + X = \left\{ u ∈ C^1[0,\pi ]: u(0) = u(\pi ) = 0 \right\} + \] + ist, die durch + \[ + Y = \left\{ u ∈ X: \int_0^\pi |u(x)|^2 dx = 1 \right\} + \] + gegeben ist. + Zwar ist $Y$ (in der $\L^2([0,\pi ])$-Metrik) beschränkt und abgeschlossen, jedoch nicht kompakt. +\end{problem-nn} +\begin{problem-nn} + Sei $\mathcal T = \{ 1, \cos t, \sin t, \cos (2t), \sin (2t), … \} = + \{\phi_i\}_{i ∈ ℕ}$. Dann ist bekanntlich + \[ + \langle \phi_i, \phi_j \rangle = ∫_0^{2\pi } φ_i(t) φ_j(t) dt = 2\pi \delta _{i,j}, + \] + wobei $\delta _{i,j}$ das Kronecker-Delta bezeichne. + Also lässt sich durch Normierung ein Orthonormalsystem aus $\mathcal T$ gewinnen. + Jetzt fragen wir uns, ob sich jede $2\pi $-periodische Funktion $u$ bezüglich eines geeigneten Konvergenzbegriffs in eine Reihe $u = \sum_{i ∈ ℕ} \alpha _i φ_i$ mit $\alpha _i ∈ ℝ$ entwickeln können. + Bereits bekannt ist, dass das für das entsprechende endlich-dimensionale Problem geht: Sei $T = \{ e_1,…,e_n\}$ die kanonische Standardbasis des $ℝ^n$ + Dann gilt bekanntlich + \[ + \langle e_i, e_j \rangle_{ℝ^n} = \delta _{i,j} + \] + und für jedes $x ∈ ℝ^n$ ist + \[ + x = \sum_{i=1}^n \alpha _i e_i, \quad \alpha _i = \langle x, e_i \rangle_{ℝ^n}. + \] + Wir fragen uns nach den Zusammenhängen zwischen den Problemen im endlich- und unendlich"=dimensionalen. +\end{problem-nn} +\begin{problem-nn} + Das Biegemoment eines Trägers kann man als Randwertaufgabe (gesucht ist $u: [0,1] → ℝ$, gegeben sind $p,r: [0,1] → ℝ$) + \[ + u''(t) + p(t) u(t) = r(t), \quad u(0) = u(1) = 0 + \] + bestimmen. Mit Hilfte der sogenannten Green'schen Funktion lässt sich diese Randwertaufgabe in eine Integralgleichung + \[ + (T_u)(t) \coloneq ∫_0^1 G(t,s) \big(r(s)-p(s)u(s)\big) ds = u + \] + umwandeln. Das heißt, man sucht einen Fixpunkt eines Integraloperators $T$ in einer geeigneten Menge von Funktionen. +\end{problem-nn} + +Diese Probleme lassen sich mit der klassischen Analysis nicht mehr behandeln. +In der Funktionalanalysis behandeln wir nun im Wesentlichen „Analysis in $\infty$-dimensionalen Räumen“ (meist Funktionenräume). +Das heißt, wir wollen jetzt anstelle des $\K^n$ allgemeinere Räume betrachten, die jodoch immer noch folgende beide Charakteristika aufweisen: +\begin{enumerate} +\item Die lineare Struktur (das heißt, Elemente lassen sich addieren und mit einem Skalar multiplizieren) +\item Die topologische Struktur (also insbesondere ein Konvergenzbegriff) +\end{enumerate} + +Unser Ziel ist es zunächst, die beiden Strukturen zu erarbeiten. \ No newline at end of file diff --git a/pdf/funkana.pdf b/pdf/funkana.pdf deleted file mode 100644 index c8046a5..0000000 Binary files a/pdf/funkana.pdf and /dev/null differ diff --git a/ref.bib b/ref.bib new file mode 100644 index 0000000..7507e59 --- /dev/null +++ b/ref.bib @@ -0,0 +1,153 @@ +@book {MR0450957, + AUTHOR = {Adams, Robert A.}, + TITLE = {Sobolev spaces}, +% NOTE = {Pure and Applied Mathematics, Vol. 65}, + PUBLISHER = {Academic Press, New York-London}, + YEAR = {1975}, +% PAGES = {xviii+268}, + MRCLASS = {46E35}, + MRNUMBER = {0450957}, +MRREVIEWER = {A. Kufner}, +} +@book{alt2002lineare, + title={Lineare Funktionalanalysis: Eine anwendungsorientierte Einf{\"u}hrung}, + author={Alt, H.W.}, + isbn={9783540439479}, + series={Springer Lehrbuch}, + year={2002}, + publisher={Springer} +} +@book {MR0217549, + AUTHOR = {Bachman, George and Narici, Lawrence}, + TITLE = {Functional analysis}, + PUBLISHER = {Academic Press, New York-London}, + YEAR = {1966}, +% PAGES = {xiv+530}, + MRCLASS = {46.00 (47.00)}, + MRNUMBER = {0217549}, +MRREVIEWER = {R. G. Bartle}, +} +@book {MR787404, + AUTHOR = {Deimling, Klaus}, + TITLE = {Nonlinear functional analysis}, + PUBLISHER = {Springer-Verlag, Berlin}, + YEAR = {1985}, +% PAGES = {xiv+450}, + ISBN = {3-540-13928-1}, + MRCLASS = {47-01 (47Hxx 55M25 58-01 58Cxx)}, + MRNUMBER = {787404}, +MRREVIEWER = {Joachim Naumann}, + URL = {https://doi.org/10.1007/978-3-662-00547-7}, +} + +@book {MR1787146, + AUTHOR = {Werner, Dirk}, + TITLE = {Funktionalanalysis}, + PUBLISHER = {Springer-Verlag, Berlin}, + YEAR = {2000}, +% PAGES = {xii+501}, + ISBN = {3-540-67645-7}, + MRCLASS = {46-01 (47-01)}, + MRNUMBER = {1787146}, +MRREVIEWER = {Manfred Wolff}, +} +@book {MR617913, + AUTHOR = {Yosida, K\^osaku}, + TITLE = {Functional analysis}, + SERIES = {Grundlehren der Mathematischen Wissenschaften}, + VOLUME = {123}, + PUBLISHER = {Springer-Verlag, Berlin-New York}, + YEAR = {1980}, +% PAGES = {xii+501}, + ISBN = {3-540-10210-8}, + MRCLASS = {46-01 (47-01)}, + MRNUMBER = {617913}, +} +@book {MR2380292, + AUTHOR = {Heuser, Harro}, + TITLE = {Funktionalanalysis}, + SERIES = {Mathematische Leitf\"aden}, + NOTE = {Theorie und Anwendung}, + PUBLISHER = {B. G. Teubner, Stuttgart}, + YEAR = {2006}, +% PAGES = {696}, + ISBN = {978-3-8351-0026-8; 3-8351-0026-2}, + MRCLASS = {46-01 (47-01)}, + MRNUMBER = {2380292}, + URL = {https://doi.org/10.1007/978-3-8351-9072-6}, +} +@book {MR0365062, + AUTHOR = {Rudin, Walter}, + TITLE = {Functional analysis}, + NOTE = {McGraw-Hill Series in Higher Mathematics}, + PUBLISHER = {McGraw-Hill Book Co., New York-D\"usseldorf-Johannesburg}, + YEAR = {1973}, +% PAGES = {xiii+397}, + MRCLASS = {46-01}, + MRNUMBER = {0365062}, +MRREVIEWER = {F. Smithies}, +} +@book {MR2953760, + AUTHOR = {Meise, Reinhold and Vogt, Dietmar}, + TITLE = {Einf\"uhrung in die {F}unktionalanalysis}, + PUBLISHER = {Vieweg + Teubner Verlag, Wiesbaden}, + YEAR = {2011}, +% PAGES = {x+273}, + ISBN = {978-3-8348-1872-0}, + MRCLASS = {46-02 (46-01 46A04 46A45 46B99 47-01)}, + MRNUMBER = {2953760}, + URL = {https://doi.org/10.1007/978-3-8348-8654-5}, +} +@book {MR564653, + AUTHOR = {Taylor, Angus Ellis and Lay, David C.}, + TITLE = {Introduction to functional analysis}, + PUBLISHER = {John Wiley \&\ Sons, New York-Chichester-Brisbane}, + YEAR = {1980}, +% PAGES = {xi+467}, + ISBN = {0-471-84646-5}, + MRCLASS = {46-01 (47-01)}, + MRNUMBER = {564653}, +} + +@book {MR1347692, + AUTHOR = {Zeidler, Eberhard}, + TITLE = {Applied functional analysis}, + SERIES = {Applied Mathematical Sciences}, + VOLUME = {109}, +% NOTE = {Main principles and their applications}, + PUBLISHER = {Springer-Verlag, New York}, + YEAR = {1995}, +% PAGES = {xvi+404}, + ISBN = {0-387-94422-2}, + MRCLASS = {00A05 (35-01 46-01 47-01 49-01 58-01)}, + MRNUMBER = {1347692}, +MRREVIEWER = {Jean Mawhin}, +} +@book {MR0467224, + AUTHOR = {Wloka, Josef}, + TITLE = {Funktionalanalysis und {A}nwendungen}, +% NOTE = {de Gruyter Lehrbuch}, + PUBLISHER = {Walter de Gruyter, Berlin-New York}, + YEAR = {1971}, +% PAGES = {291}, + MRCLASS = {46-01}, + MRNUMBER = {0467224}, +} +@book {MR969367, + AUTHOR = {Dautray, Robert and Lions, Jacques-Louis}, + TITLE = {Mathematical analysis and numerical methods for science and + technology. {V}ol. 2}, +% NOTE = {Functional and variational methods, +% With the collaboration of Michel Artola, Marc Authier, +% Philippe B\'enilan, Michel Cessenat, Jean Michel Combes, H\'el\`ene +% Lanchon, Bertrand Mercier, Claude Wild and Claude Zuily, +% Translated from the French by Ian N. Sneddon}, + PUBLISHER = {Springer-Verlag, Berlin}, + YEAR = {1988}, +% PAGES = {xvi+561}, + ISBN = {3-540-19045-7}, + MRCLASS = {00A05}, + MRNUMBER = {969367}, + URL = {https://doi.org/10.1007/978-3-642-61566-5}, +} + \ No newline at end of file diff --git a/skript.cls b/skript.cls index 1eddd77..cb2247b 100644 --- a/skript.cls +++ b/skript.cls @@ -9,27 +9,35 @@ \RequirePackage{tikz-cd} \tikzcdset{arrow style=tikz, diagrams={>=stealth}} -\RequirePackage{ifluatex} +\RequirePackage{ifxetex,ifluatex} +\newif\ifxetexorluatex +\ifxetex + \xetexorluatextrue +\else + \ifluatex + \xetexorluatextrue + \else + \xetexorluatexfalse + \fi +\fi -\RequirePackage{csquotes} -\RequirePackage{hyphenat} -\RequirePackage{titlesec} +\RequirePackage{scrlayer-scrpage} \RequirePackage{gitinfo} -\RequirePackage{mathtools} -\RequirePackage{amsmath, amssymb} +\RequirePackage{mathtools} +\RequirePackage{amsmath, amssymb} -\ifluatex - \RequirePackage{polyglossia} +\ifxetexorluatex + \RequirePackage[babelshorthands]{polyglossia} \setdefaultlanguage{german} - \RequirePackage{libertine} + \setmainfont{Libertinus Serif} \RequirePackage[warnings-off={mathtools-colon,mathtools-overbracket}]{unicode-math} - \setmathfont{TeX Gyre Pagella Math} - \setmathfont[range=\setminus]{XITS Math} - \setmathfont[range={\sum}]{TeX Gyre Termes Math} - \setmathfont[range={\int}]{XITS Math} - \setmathfont{Latin Modern Math}[range={cal,bfcal},StylisticSet=1] + \setmathfont[AutoFakeBold]{Libertinus Math} + \setmathfont[AutoFakeBold,range={\rightarrow,\leftarrow,\rightharpoonup,\rightharpoondown,\leftharpoondown,\leftharpoonup}]{texgyretermes-math.otf} + \setmathfont[AutoFakeBold,range={\Vert,\lbrace,\rbrace,\vert}]{xits-math.otf} + \setmathfont[AutoFakeBold,range={\BbbN,\BbbR,\BbbC,\BbbZ,\mathbb,\int}]{XITS Math} + % \setmathfont{latinmodern-math.otf}[range={cal,bfcal}] \else \RequirePackage[ngerman]{babel} \RequirePackage[utf8]{inputenc} @@ -38,37 +46,39 @@ \RequirePackage[libertine]{newtxmath} \RequirePackage[cal=zapfc,bb=boondox]{mathalfa} \RequirePackage[T1]{fontenc} + \def\lAngle{\langle\langle} + \def\rAngle{\rangle\rangle} \fi % fonts \setkomafont{disposition}{\rmfamily} -\RequirePackage{mathtools} -\RequirePackage{amsmath, amsfonts, amssymb} - % fonts -\RequirePackage{textcomp} % für erweiterten Text-Symbolvorrat \RequirePackage{setspace} -\setstretch{1.05} +\setstretch{1.1} \setlength\parskip{4pt} \setlength\parindent{0pt} \RequirePackage[amsmath, thmmarks, framed]{ntheorem} \RequirePackage[framemethod=tikz]{mdframed} -\RequirePackage{versions} -\RequirePackage{hyperref} +\RequirePackage[unicode,colorlinks]{hyperref} \RequirePackage[capitalise, nameinlink]{cleveref} -\RequirePackage{scrpage2} -\RequirePackage{authoraftertitle} - \pagestyle{scrheadings} \clearscrheadfoot \ohead{\headmark} -\cfoot{-- \pagemark~--} +\cfoot{\pagemark} +\ifoot{\tiny Revision\gitVtags: \gitAbbrevHash{} (\gitAuthorDate)} \automark{section} +\RequirePackage[ + backend=biber, + style=alphabetic, + giveninits, + url=true, + ]{biblatex} + \usepackage{enumitem} \setenumerate{label=(\alph*)} @@ -118,8 +128,8 @@ \def\newdef#1#2{\newtheorem{#1}[defsatzusw]{#2}\newtheorem*{#1-nn}{#2}} \theoremseparator{.} \numberwithin{defsatzusw}{section} -% \theoremsymbol{\ensuremath{\diamond}} % kursive schrift +\theorembodyfont{\normalfont} \theoremstyle{mychange} \newthm{satz}{Satz} \newthm{lemma}{Lemma} @@ -148,12 +158,11 @@ \theorembodyfont{\normalfont} \theoremseparator{.} % \theoremsymbol{\scalebox{0.8}{\ensuremath{\blacksquare}}} -\theoremsymbol{\nolinebreak[1]\hspace*{.5em plus 1fill}\ensuremath{\blacksquare}} +\theoremsymbol{\nolinebreak[1]\hspace*{.5em plus 1fill}\scalebox{0.8}{\ensuremath{\blacksquare}}} \newtheorem{proof}{Beweis} \newtheorem{beweis}{Beweis} -\titleformat{\section}{\titlefont\Large}% - {\S\,\thesection}{.66em}{} \renewcommand{\thesection}{\arabic{section}} +\renewcommand\sectionformat{\S \thesection\autodot \hspace{1ex}} \endinput \ No newline at end of file -- cgit v1.2.3-24-g4f1b